LEIK FNP Practice Questions

Pataasin ang iyong marka sa homework at exams ngayon gamit ang Quizwiz!

All of the following clinical findings are considered benign oral findings except: A) A patch of leukoplakia B) Fordyce spots C) Torus palatinus D) Fishtail uvula

A) A patch of leukoplakia An early sign of HIV is a hairy patch of leukoplakia on the tongue.

Which of the following would be classified as a second-degree burn? A) A severe sunburn with blistering B) Burns that involve the subcutaneous layer of skin C) A reddened finger after touching a hot iron D) Burns that involve escha

A) A severe sunburn with blistering Burns: First degree: erythema without blistering, painful; second degree: red skin with superficial blisters, painful; third degree: entire skin layering, involving the subcutaneous tissue and may include soft tissue facia, painless.

While doing a cardiac exam on a 45-year-old male, you note an irregular rhythm with a pulse rate of 110 beats per minute. The patient is alert and is not in distress. What is the most likely diagnosis? A) Atrial fibrillation B) Ventricular fibrillation C) Cardiac arrhythmia D) First-degree right bundle branch block

A) Atrial fibrillation Atrial fibrillation is chaotic electrical activity of the heart, caused by several ectopic foci in the atria without any signs of distress.

You note that your 11-year-old female patient is at Tanner Stage II. You would advise her mother that menarche will probably start in: A) 1 to 2 years B) 2.5 to 3 years C) 4 years D) It is dependent on the girl's genetic makeup

A) 1 to 2 years Menarche normally begins approximately 1-2 years following Tanner Stage II. Average age is 11.8 years old.

The span of the normal adult liver is: A) 15 to 18 cm in the midclavicular line B) 12 to 16 cm in the right midclavicular line C) 2 to 6 cm in the midsternal line D) 4 to 8 cm in the midsternal line

A) 15 to 18 cm in the midclavicular line This range is considered the normal span for adults.

The posterior fontanel should be completely closed by: A) 3 months B) 4 months C) 5 months D) 6 months

A) 3 months The posterior fontanelle normally closes by 3 months of age. The anterior fontanelle closes between 12 and 18 months of age.

One of the developmental milestones for this age group is the ability to draw a stick figure "person" with 2 to 3 separate body parts. What is the age group that this finding is associated with? A) 3 year old B) 4 year old C) 5 year old D) 6 year old

A) 3 year old The ability to draw a person with 2 to 3 separate body parts is one of the developmental milestones to assess in children who are 3 years of age. Others are climbs up stairs with alternating feet without handrails; rides a tricycle; knows name, age, and gender; and can speak in 3-word sentences that can be understood by most strangers.

The most current recommendation from the Joint National Commission on the Evaluation and Treatment of High Blood Pressure in Adults (JNC 8) for the blood pressure goal in diabetics is: A) <140/90 B) <130/85 C) <130/80 D) <125/75

A) < 140/90 JNC 8 bp goals include < 140/90 in patients with diabetes.

According to the guidelines outlined in JNC 8 in those younger than 60 (without comorbitidites), the normal blood pressure should be: A) <140/90 B) <130/85 C) <120/80 D) <110/75

A) <140/90 The JNC VIII Guidelines recommended that bp be less than 140/90 in those less than 60. Those older than 60, BP should be less than 150/90.

An 80-year-old male with hypertension and hyperlipidemia presents with complaints of the quick onset of severe low back pain that is accompanied by abdominal pain which is gradually worsening. The patient appears pale and complains that he does not feel well. During the abdominal exam, the nurse practitioner detects a soft pulsatile mass just above the umbilicus as she palpates this area with her hand. Which of the following conditions is most likely? A) Abdominal aortic aneurysm B) Cauda equina syndrome C) Acute diverticulitis D) Adenocarcinoma of the colon

A) Abdominal aortic aneurysm Elderly males who are ex-smokers are at higher risk for abdominal aortic aneurysm. It is usually asymptomatic and is discovered incidentally during a routine chest x-ray or abdominal ultrasound. Although small aneurysms are usually not detectable during abdominal exams, the larger aneurysms may be palpable during an abdominal exam, but abdominal obesity will obscure the findings. The symptoms in this case point toward a rapidly dissecting aneurysm. The best action in this case is to call 911 stat.

A positive obturator sign might signify which of the following conditions? A) Acute appendicitis B) Acute pancreatitis C) Acute cholecystitis D) Acute hepatitis

A) Acute appendicitis A positive obturator sign may indicate acute appendicitis. The test is performed by having the patient lie down supine, and rotate the hip, using full range of motion. A positive sign is when pain is experienced with movement or flexion of the hip.

An urgent care nurse practitioner is assessing a 45-year-old White woman with a BMI of 32 for complaints of intermittent right upper quadrant abdominal pain over the past few weeks that is precipitated by eating fried foods and peanut butter. On exam, the patient's heart and lungs are normal. There is no pain over the costovertebral angle. During abdominal exam, the bowel sounds are present in all quadrants. While palpating deeply on the right upper quadrant during deep inspiration, the patient complains of severe sharp pain to the nurse practitioner. Which of the following is best described? A) Acute cholecystitis B) Acute appendicitis C) Acute gastroenteritis D) Acute diverticulitis

A) Acute cholecystitis Acute cholecystitis can be diagnosed when deep palpation of the right upper quadrant causes the patient to complain of pain.

Grey-Turner's sign is highly suggestive of which of the following conditions? A) Acute pancreatitis B) Acute appendicitis C) Acute diverticulitis D) Gastric cancer

A) Acute pancreatitis Grey-Turner's sign is the acute onset of bluish discoloration located on the flank area that is caused by bruising. It is usually associated with severe acute pancreatitis, but it can also be found in some cases of ruptured ectopic pregnancy.

Cullen's sign is most commonly associated with which of the following? A) Acute pancreatitis B) Myocardial infarction C) Acute pyelonephritis D) Preeclampsia

A) Acute pancreatitis Cullen's sign is commonly seen in acute pancreatitis. It is a yellowish-blue skin color change around the umbilicus. It is thought to occur due to the pancreatic enzymes that run along the ligament and subcutaneous tis- sues around the umbilicus.

All of the following are possible causes for secondary hypertension except: A) Acute pyelonephritis B) Pheochromocytoma C) Renovascular stenosis D) Coarctation of the aorta

A) Acute pyelonephritis Pheochromocytoma is an adrenal gland tumor in which there is increased production of the hormones adrenaline and noradrena- line, which may cause long-term elevation in blood pressure. Renovascular steno- sis is a narrowing of one or both arteries leading to the kidneys. It can cause severe hypertension and irreversible kidney damage. Coarctation of the aorta is a con- genital heart defect of the aorta; it is a narrowing of the aorta that causes the heart to work harder to get blood to flow through the narrow aortic passageway to other organs, which in turn causes an increase in blood pressure.

A 28-year-old male nurse of Hispanic descent reports a history of a cold that resolved 2 weeks ago except for a dry cough and pain over his right cheek that worsens when he bends down. The patient denies fever. He tells the employee health nurse practitioner that he is very allergic to both cephalexin (Keflex) and erythromycin. The vital signs show that the patient's temperature is 99.2 degrees Fahrenheit, a pulse of 72 beats per minute, and a respiratory rate of 12 breaths per minute. Which of the following conditions is most likely? A) Acute sinusitis B) Acute bronchitis C) Fever secondary to the previous viral URI (upper respiratory infection) D) Munchausen's syndrome

A) Acute sinusitis Acute sinusitis symptoms include cough, facial pain, and low-grade fever.

A 50-year-old woman of Irish descent presents with history of lethargy, feeling weak, nausea, anorexia with diarrhea and abdominal pain. The woman's skin appears tanned, but she denies prolonged sun exposure. During physical exam, the skin appears tan with hyperpigmentation of the nipple area, the gums, and the lips. The electrolyte panel reveals hyperkalemia and hyponatremia. She reports craving salty foods. Which of the following is most likely? A) Addison's disease B) Cushing's disease C) Metabolic syndrome D) Cutaneous drug reaction

A) Addison's disease Addison's disease is also known as primary adrenal insufficiency. The most common cause of damage to the adrenal cortex (the outer layer of the gland) is autoimmune destruction. The adrenal cortex produces glucocorticoids (cortisol) and mineralocorticoids (aldosterone). Aldosterone regulates sodium retention and potassium excretion through the kidneys (affects blood pressure). Electrolyte abnormalities are high potassium and low sodium. In primary disease (Addison's), serum cortisol is low, ACTH is high, and serum aldosterone is low. If the patient is not treated, severe stress (illness, accident) may cause an adrenal crisis ("Addisonian" crisis), which can be fatal.

Clara is a 20-year-old college student who reports to the student health clinic with a laceration in her left hand. She tells the nurse practitioner that she cut her hand while working in her garden. Her last Td booster was 5.5 years ago. Which of the following is correct? A) Administer a booster dose of the Td vaccine B) Administer the Td vaccine and the Td immunoglobulin (HyperTet) C) Administer Td immunoglobulin (HyperTet) only D) She does not need any Td immunoglobulin (Hypertet) or a Td booster

A) Administer a booster dose of the Td vaccine Td booster vaccine is recommended for recent injuries if it has been more than 5 years since the last Td.

The red reflex is elicited by shining a light in the eyes of the infant at an angle with the light about 15 in. away. The nurse practitioner is screening for? A) Cataracts B) Strabismus C) Blindness D) The blinking response

A) Cataracts Screening for cataracts is performed by shining a light into the eyes at an angle, approximately 15 inches away. If the red reflex is not elicited, then cataracts are present.

A multigravida who is at 28 weeks of gestation has a fundal height of 29 cm. Which of the following is the best recommendation for this patient? A) Advise the mother that her pregnancy is progressing well B) Order an ultrasound of the uterus C) Refer her to an obstetrician for an amniocentesis D) Recommend bed rest with bathroom privileges

A) Advise the mother that her pregnancy is progressing well From 20-35 weeks' gestation, fundal height should equal weeks gestation. If greater than 2 cm within the gestational week of pregnancy, further testing should be performed to evaluate fetal growth

A nurse practitioner is doing a funduscopic exam on a 35-year-old female during a routine physical exam. He notices that she has sharp disc margins and a yellowish- orange color in the macular area. The ratio of veins to arteries is 3:2. What is the next most appropriate action? A) Advise the patient that she had a normal exam B) Advise the patient that she had an abnormal exam C) Refer the patient to the emergency room D) Refer the patient to an ophthalmologist

A) Advise the patient that she had a normal exam A normal funduscopic exam would include sharp disc margins, with the macula appearing yellow-orange in color. The ratio comparing the artery to vein width is 2:3.

All of the following agents are used to control the inflammatory changes seen in the lungs of asthmatics except: A) Albuterol inhaler (Proventil) B) Triamcinolone (Azmacort) C) Montelukast (Singulair) D) Cromolyn sodium inhaler (Intal)

A) Albuterol inhaler (Proventil) Proventil inhaler is a short-acting bronchodilator and is used for immediate relief of shortness of breath. It acts quickly to dilate the tubes in the lungs. Medications that have steroids and leukotrienes help the inflamed channels to remain open and clear but take longer to get into the system to work.

Rovsing's sign is associated with which of the following? A) An acute abdomen, such as during a ruptured appendix B) Knee instability C) Damage to the meniscus of the knee D) Acute cholelithiasis

A) An acute abdomen, such as during a ruptured appendix Rovsing's sign identifies an acute abdomen, such as acute appendicitis. Rovsing's maneuver is per- formed with the patient in the supine position, palpating deep into the left lower quadrant of the abdomen and having referred pain to the right lower quadrant.

The Jarisch-Herxheimer reaction is best described as: A) An immune-mediated reaction precipitated by the destruction of a large number of spirochetes due to an antibiotic injection B) Severe chills and elevated blood pressure C) Caused by infection with either Chlamydia trachomatis or gonorrheal infection of the liver capsule D) Associated with certain viral illnesses

A) An immune-mediated reaction precipitated by the destruction of a large number of spirochetes due to an antibiotic injection The Jarisch-Herxheimer reaction is defined as an immune-mediated reaction precipitated by the destruction of a large number of spirochetes due to the antibiotic injection.

When an infant is found to have tufts of fine dark hair on the sacrum, which of the following tests is recommended? A) An ultrasound of the sacrum B) A plain radiograph of the lumbo sacral spine C) No imaging test is necessary D) Genetic testing

A) An ultrasound of the sacrum An infant with tufts of fine dark hair on the sacrum should be evaluated for occult spina bifida. The first imaging test to order is an ultrasound of the lower spine.

A 55-year-old male patient describes to you an episode of chest tightness in his substernal area that radiated to his back while he was jogging. It was relieved immediately when he stopped. Which of the following conditions does this best describe? A) Angina pectoris B) Acute myocardial infarction C) Gastroesophageal reflux disease D) Acute costochondritis

A) Angina pectoris Symptoms of angina pectoris include a pressing, squeezing, or crushing pain, usually in the chest or back, arms, neck, or ear lobes. Weakness, fatigue, and shortness of breath and pain radiating in the arms, shoulders, jaw, neck, and/or back may also occur. Angina pectoris is most commonly brought on by physical exertion, but can also be triggered by stress and other stressors to the body.

Which of the following is recommended by JNC 7 as first-line treatment for hypertension in patients with microalbuminuria? A) Angiotensin-converting enzyme (ACE) inhibitors B) Diuretics C) Calcium channel blockers D) Beta-blockers

A) Angiotensin-converting enzyme (ACE) inhibitors First line of treatment for hypertension in patients with microalbuminuria is ACE inhibitors. ACE inhibitors are renal protective.

A medium-pitched harsh mid-systolic murmur is best heard at the right second ICS of the chest. It radiates into the neck. Which of the following is the correct diagnosis? A) Aortic stenosis B) Pulmonic stenosis C) Aortic regurgitation D) Mitral stenosis

A) Aortic stenosis The murmur associated with aortic stenosis can be auscul- tated as harsh and high pitched in the right second intercostal space; it typically radiates to the carotid arteries and apex.

You can determine a pulse deficit by counting the: A) Apical and radial pulses at the same time, then finding the difference between the two B) Apical pulse first, then the radial pulse, and subtracting to find the difference between the two C) Apical pulse and the femoral pulse at the same time and finding the difference between the two D) Radial pulse first, then counting the femoral pulse, and subtracting to find the difference between the two

A) Apical and radial pulses at the same time, then finding the difference between the two The pulse deficit is determined by counting the apical and radial pulses at the same time, then subtracting to find the difference between the two.

Patients who are being screened for TB and are immunocompromised should be evaluated for anergy. Which of the following is the best description of anergy testing? A) Apply Candida or mumps antigen to the right forearm and the PPD on the left forearm and read results in 48 to 72 hours B) Apply Candida or mumps antigen and PPD on left forearm only and check for a reaction in 24 hours C) Mix the Candida or mumps antigen with the PPD and apply it to both forearms D) Apply the Candida or mumps antigen 24 hours before the PPD on the left forearm

A) Apply Candida or mumps antigen to the right forearm and the PPD on the left forearm and read results in 48 to 72 hours Anergy is when the immune system is unable to perform a healthy, normal immune response when the body is challenged with a particular antigen.

The nurse practitioner notices a gray ring on the edge of both irises of an 80-year- old female. The patient denies visual changes or pain. She reports that she has had the "ring" for many years. Which of the following is the most likely diagnosis? A) Arcus senilis B) Pinguecula C) Peripheral cataracts D) Macular degeneration

A) Arcus senilis Arcus senilis is frequent in older adults. Arcus senilis does not affect vision.

A woman is being evaluated by the nurse practitioner for complaints of dyspareunia. A microscopy slide reveals a large number of atrophic squamous epithelial cells. The vaginal pH is 4.0. There are very few leukocytes and no RBCs are seen on the wet smear. Which of the following is most likely? A) Atrophic vaginitis B) Bacterial vaginosis C) Trichomoniasis D) This is a normal finding

A) Atrophic vaginitis Symptoms of atrophic vaginitis include painful intercourse, atrophic squamous epithelial cells, and a decrease in pH. Vaginal atrophy is caused by lack of or imbalance of estrogen. Normal pH of the vagina is 4.0-5.0 (acidic).

A 13-year-old patient has a throat culture that is positive for strep throat. She reports that her younger brother was recently diagnosed with strep throat and treated. The patient has a severe allergy to penicillin and reports that erythromycin makes her very nauseated. Which of the following antibiotics is the best choice? A) Azithromycin (Zithromax) B) Cephalexin (Keflex) C) Cefuroxime axetil (Ceftin) D) Levofloxacin (Levaquin)

A) Azithromycin (Zithromax) If the patient has a severe penicillin allergy, there is a 10% chance of cross-reactivity to cephalosporins (especially first generation). Because the patient is a child, the levofloxacin is contraindicated. Nausea is a com- mon adverse reaction to erythromycin (it is not an allergic reaction). The best option is to use azithromycin because of its minimal GI adverse effects. Azithromycin has fewer drug interactions compared with other macrolides.

A chest radiograph shows an area of consolidation on the lower lobe. Which of the following conditions is most likely? A) Bacterial pneumonia B) Acute bronchitis C) COPD D) Atypical pneumonia

A) Bacterial pneumonia Consolidation is visible on x-ray when bacterial pneumonia is present. Consolidation is not present in the lungs with bronchitis, COPD, or atypical pneumonia.

What is the most common type of skin cancer? A) Basal cell skin cancer B) Squamous cell skin cancer C) Melanoma D) Actinic keratose

A) Basal cell skin cancer Skin cancer is the most common type of cancer overall, but the most common type of skin cancer is basal cell carcinoma of the skin. Regarding mortality, the skin cancer that causes the most deaths (from skin cancer) is melanomas (65% of skin cancer deaths).

Terazosin (Hytrin), an alpha blocker, is used to treat which of the following conditions? A) Benign prostatic hypertrophy and hypertension B) Chronic prostatitis and atrial fibrillation C) Urinary tract infections and arrhythmias D) Benign prostatic hypertrophy and chronic prostatitis

A) Benign prostatic hypertrophy and hypertension Hytrin is used to treat benign prostatic hypertrophy and hypertension.

Laws governing nurse practitioner authority are determined by the: A) Board of Nursing B) State Legislature C) American Nursing Association D) State Medical Association

A) Board of Nursing Each state has laws establishing a Board of Nursing that regulates the role of the nurse practitioner.

You note bony nodules located at the proximal interphalangeal joints on both the hands of your 65-year-old female patient. Which of the following is most likely? A) Bouchard's node B) Heberden's node C) Osteoarthritic nodules D) Tophi deposits

A) Bouchard's node Bony nodules at the proximal interphalangeal joints of the hands are called Bouchard's nodes. Heberden's nodes are nodules on the distal interphalangeal joints. Tophi deposits are seen with gout, where high levels of uric acid in the blood occur and cause nodules in the joint that can eventually destroy the bone. Osteoarthritic nodules develop in the joints of the hands.

A 14-year-old teen is worried that she has not started to menstruate like most of her friends. During the gynecological examination, the nurse practitioner tells the mother, who is in the room with the patient, that her daughter is starting Tanner Stage II. What are the physical exam findings during this stage? A) Breast buds and some straight pubic hair B) Fully developed breasts and curly pubic hair C) Breast tissue with the areola on a separate mound with curly pubic hair D) No breast tissue and no pubic hair

A) Breast buds and some straight pubic hair Tanner Stage II in females includes breast buds and few straight, fine pubic hairs.

The nurse practitioner is educating a new patient with Raynaud's phenomenon about lifestyle changes that have been found helpful in decreasing exacerbations of the disorder. Which of the lifestyle recommendations is false? A) Caffeine-containing drinks and foods such as chocolate have been found helpful with decreasing the incidence of exacerbations B) Wear gloves or mittens during cold weather and be careful when handling frozen foods C) Stop smoking and exercise at least 3 times a week D) Decrease emotional stress and lifestyle stressors

A) Caffeine-containing drinks and foods such as chocolate have been found helpful with decreasing the incidence of exacerbations Lifestyle changes associated with decreasing exacerbations are wearing gloves or mittens during cold weather, taking care with handling frozen foods (wear gloves) and avoiding vasoconstricting agents (caffeine, smoking, cocaine, amphetamines) and emotional stress. Exercise and reducing lifestyle stress are recommended. Usually involves the fingers and/or toes due to severe arteriolar vasospasm causing ischemia. During an exacerbation, the fingers change color into white, blue, and red (think of the American flag as a reminder). Raynaud's is classified either as primary (Raynaud's disease) or secondary (Raynaud's phenomenon). Individuals with secondary Raynaud's have a higher risk of autoimmune disorders such as scleroderma, Sjogren syndrome, and systemic lupus erythematosus. Affects mostly young women (ages 15 to 30 years).

Which of the following antihypertensive medications has beneficial effects for an elderly White female with osteoporosis? A) Calcium channel blocker B) Angiotensin-converting enzyme (ACE) inhibitor C) Beta-blocker D) Diuretic

A) Calcium channel blocker Calcium channel blockers act by blocking the cal- cium channels in the heart muscle and the blood vessels, thereby keeping more calcium in the bones.

The cones in the retina of the eye are responsible for: A) Central vision B) Peripheral vision C) Night vision D) Double vision

A) Central vision The cones of the eyes are responsible for central vision.

A 35-year-old woman is complaining of gradual weight gain, lack of energy, and amenorrhea. The urine pregnancy test is negative. The CBC shows a hemoglobin of 13.5 g and MCV 84. The nurse practitioner suspects that the patient may have hypothyroidism. The TSH is 10 mU/L. Which of the following is the next step in the evaluation? A) Check the thyroid profile B) Check the total T3 and T4 levels C) Check for antithyroid peroxidase antibodies D) Recheck the TSH in 4 to 6 months

A) Check the thyroid profile The upper limit of the serum TSH level is about 5.0 mU/L (range of 0.5 to 5.0). With an elevated TSH of 10, it is important to rule out hypothyroidism. The next step in this patient's evaluation is to order a thyroid profile test. Serum assays measure bound and unbound (free) forms of thyroxine (T4) and triiodothyronine (T3). Classic findings of hypothyroidism are a low total T4, low T3-resin uptake (THBI), and low free T4 index.

Some pharmacologic agents may cause confusion in the elderly. Which of the following pharmacologic agents is most likely to cause confusion in this population? A) Cimetidine (Tagamet), digoxin (Lanoxin), diphenhydramine (Benadryl) B) Acetaminophen (Tylenol), aspirin (Bayer), indomethacin (Indocin) C) Sucralfate (Carafate), docusate sodium (Surfak), psyllium (Metamucil) D) Cephalexin (Keflex), amoxicillin (Amoxil), clarithromycin (Biaxin)

A) Cimetidine (Tagamet), digoxin (Lanoxin), diphenhydramine (Benadryl) Medications that commonly cause confusion in the elderly include cimetidine, lanoxin, and diphenhydramine.

An adult patient was recently discharged from the hospital with a prescription of clindamycin. The patient reports that he took his last dose yesterday. He presents in the primary care clinic with complaints of the recent onset of watery diarrhea from 10 to 15 times a day with abdominal cramping. He denies fever and chills. Which of the following conditions is most likely in this patient? A) Clostridium difficile-associated diarrhea (CDAD) B) Giardiasis C) Pseudomembranous colitis D) Irritable bowel syndrome

A) Clostridium difficile-associated diarrhea (CDAD) An important risk factor for Clostridium difficile-associated diarrhea (CDAD) and C. difficile colitis is antibiotic therapy and hospitalization. Almost any antibiotic can cause the condition, but the most common are clindamycin, cephalosporins, and fluoroquinolones. Diarrhea can occur during therapy as well as after therapy (5 to 10 days; up to 10 weeks). Pseudomembranous colitis is a complication of C. difficle colitis.

The S1 heart sound is caused by: A) Closure of the atrioventricular valves B) Closure of the semilunar valves C) Opening of the atrioventricular valves D) Opening of the semilunar valves

A) Closure of the atrioventricular valves Closure of the AV valves causes the S1 sound heard when auscultating heart sounds.

A 35-year-old smoker is being evaluated for birth control choices. The patient has a history of PID along with an embolic episode after her last pregnancy. Which of the following methods of birth control would you recommend? A) Condoms and the vaginal sponge (Today Sponge) B) Estrogen patches C) Intrauterine device D) Depo-Provera (depot medroxyprogesterone)

A) Condoms and the vaginal sponge (Today Sponge) Contraindications for hormonal contraception include: Migraine headaches; cigarette smoking or obesity in women older than 35 years; history of thromboembolic disease; hypertension or vascular disease if over 35 years of age; systemic lupus erythematosus with vascular disease, nephritis, or antiphospholipid antibodies; breastfeeding (may use progestin-only pills); hypertriglyceridemia; CAD; CHF; and strokes.

All of the following findings are associated with the secondary stage of an infection by the organism Treponema pallidum except: A) Condyloma acuminata B) Maculopapular rash of the palms and soles C) Lymphadenopathy D) Condyloma lata

A) Condyloma acuminata Findings consistent with the diagnosis of syphilis, caused by the Treponema pallidum organism, include maculopapular rash of the palms and soles, lymphadenopathy, and condyloma lata. Condyloma acuminata (genital warts) are caused by the human papilloma virus (HPV) and spread to oth- ers by skin-to-skin contact.

What is the name of the physiologically active compound that is derived from soybeans? A) Isoflavones B) Estrogen C) Progesterone D) Resveratrol

A) Isoflavones Soy isoflavones mimic the action of estrogen in the body. It is derived from soybeans and soybean products (soy milk, tofu). Resveratrol is found in red grapes and red wine. It is thought to help prolong life and is considered one of the reasons that the Mediterranean diet increases longevity

A possible complication from Bell's palsy is: A) Corneal ulceration B) Acute glaucoma C) Inability to swallow D) Loss of sensation in the affected side

A) Corneal ulceration Due to the paralysis caused by the seventh cranial nerve damage, the eyelid on the affected side may not close voluntarily. This leads to dryness, which can in turn result in ulceration.

Which chronic illness disproportionately affects the Hispanic population? A) Diabetes mellitus B) Hypertension C) Alcohol abuse D) Skin cancer

A) Diabetes mellitus Diabetes mellitusis 2-3 times higher in Mexican Americans versus non-Hispanic Americans.

Mr. Jones, who has been on pravastatin (Pravachol) 20 mg at bedtime for the past few months, complains of lately feeling extremely fatigued. The patient also noticed that his urine has been a darker color during the past 2 weeks. Which of the following is the best treatment plan to follow? A) Discontinue his pravastatin and order a liver function profile B) Continue the pravastatin but on half the dose C) Schedule him for a complete physical exam D) Schedule him for a liver function profile

A) Discontinue his pravastatin and order a liver function profile Side effects of statin drugs include myalgia, fatigue, and elevated liver enzymes. Routine labs should include liver enzymes to assess the effects on the liver.

A 30-year-old male patient refuses to take his afternoon dose of pills. The nurse tells him of the possible consequences of his action, but the patient still refuses to cooperate. Which of the following is the best course for the nurse to follow? A) Document in the patient's record his behaviors and the action taken by the nurse B) Reassure the patient that he will be fine after taking the medicine C) Document only the patient behavior D) Document only the nurse's action

A) Document in the patient's record his behaviors and the action taken by the nurse If a patient is non-compliant, documentation of the patient behavior and the actions taken by the nurse both must be documented.

Which of the following is recommended treatment for erythema migrans or early Lyme disease? A) Doxycycline (Vibramycin) 100 mg PO BID x 21 days B) Ciprofloxacin (Cipro) 250 mg PO BID x 14 days C) Erythromycin (E-mycin) 333 mg PO TID x 10 days D) Dicloxacillin 500 mg PO BID x 10 days

A) Doxycycline (Vibramycin) 100 mg PO BID x 21 days The CDC-recom- mended treatment for erythema migrans or Lyme disease is doxycycline 100 mg PO BID x 21 days.

All of the following measures have been found to help lower the risk of osteoporosis except: A) Drinking organic juice B) Eating low-fat dairy foods C) Performing weight-bearing exercises D) Vitamin D supplementation

A) Drinking organic juice Vitamin D levels must be sufficient for the body to absorb calcium. Eating foods high in vitamin D and calcium along with calcium and vitamin D supplements are advised to protect the bones and prevent bone loss. Performing weight-bearing exercises daily also increases bone strength.

An elderly Hmong, who is from Thailand, is seen by the nurse practitioner for a follow-up visit. He is accompanied by his eldest daughter. The patient presented 6 weeks ago with complaints of the recent onset of morning headaches. The patient was diagnosed with Stage 2 hypertension and prescribed hydrochlorothiazide, 125 mg tablet once a day. On this visit, Mr. Nguyen tells the nurse practitioner that the new medicine cured the headache and he stopped taking the medicine. What is the best plan to follow during this visit? A) Educate the patient about hypertension, how the medicine works on his body, and the importance of taking his pill daily B) Reassure the patient that he can resume his prescription medicine again the next morning C) Tell the patient that you will lower the dose of the hydrochlorothiazide to 12.5 mg daily D) Speak to the patient in a loud voice and confront him about his behavior

A) Educate the patient about hypertension, how the medicine works on his body, and the importance of taking his pill daily When Hmong (Thailand, Burma, Vietnamese) see a medical doctor for a symptom, they expect to be treated and "cured" of their illness. When the symptoms disappear, many will stop taking the medicine. When medication is to be taken on a long-term basis, it is important to educate the patient (and the patient's family) about the disease (i.e., hypertension), how the medicine works on the body, and the reason why he has to take his medicine daily. Hmong and Vietnamese are very polite and consider speaking in a loud voice, staring, or confrontation to be rude behavior.

The mother of an 8-year-old boy reports the presence of a round red rash on the child's left lower leg. It appeared 1 week after the child returned from visiting his grandparents, who live in Massachusetts. During the skin exam, the maculopapular rash is noted to have areas of central clearing making it resemble a round target. Which of the following is best described? A) Erythema migrans B) Rocky Mountain spotted fever C) Meningococcemia D) Larva migrans

A) Erythema migrans Erythema migrans is an annular lesion that enlarges and with time has central clearing. This is commonly caused by a bite from an infected tick and can cause Lyme disease if not treated appropriately. Lyme disease is most commonly seen in the northeastern United States.

Which of the following would you recommend on an annual basis for an elderly type 2 diabetic? A) Eye exam with an ophthalmologist B) Follow-up visit with a urologist C) Periodic visits to an optometrist D) Colonoscopy

A) Eye exam with an ophthalmologist Elderly patients with type 2 diabetes should have a dilated eye exam done annually by an ophthalmologist. It is also recommended to see a podiatrist once or twice a year. Preventive care also includes receiving a flu shot annually, receiving a pneumovax vaccine if over 60 years of age, and taking a baby aspirin of 81 mg each day.

A 25-year-old male with schizophrenia comes in for a routine annual physical. He is a heavy smoker and has a BMI of 28. The patient has been on olanzapine (Zyprexa) for 10 years. Regarding the patient's prescription, which of the following laboratory tests is recommended for monitoring for adverse effects of atypical antipsychotics? A) Fasting blood glucose, fasting lipid profile, and weight B) Urinalysis, serum creatinine, 24-hour urine for protein and creatinine clearance C) Liver function tests only D) CBC with differential, liver function tests, weight

A) Fasting blood glucose, fasting lipid profile, and weight Patients on atypical antipsychotics commonly gain weight and are at risk for obesity, hyperglycemia, and type 2 diabetes. Zyprexa will increase lipids (cholesterol, LDL, and triglycerides). Atypical antipsychotics also increase the risk of death among frail elders and older adults living in nursing homes

The majority of serum alpha fetoprotein is produced by the: A) Fetal liver B) Mother's liver C) Placenta D) Fetal neural tube

A) Fetal liver Serum alpha fetoprotein is produced by the fetal liver.

All of the following statements are correct regarding the Td vaccine except: A) Fever occurs in up to 80% of the patients B) A possible side effect is induration on the injection site C) The Td is given every 10 years D) The DPT and DT should not be given beyond the seventh birthday

A) Fever occurs in up to 80% of the patients Side effects of the Td vaccine include induration at the injection site. Td is given in adults every 10 years. The DPT and DT should not be given beyond 7 years of age. Fever may occur, but studies do not support 80% of patients having fever.

Which of the following conditions is associated with three stages of rashes? A) Fifth disease B) Erythema infectiosum C) Varicella D) Rocky Mountain spotted fever

A) Fifth disease Fifth disease has three stages: Prodromal stage: Begins with symptoms of an upper respiratory infection, such as low-grade fever, headache, chills, and malaise. Second stage: A red rash appears on the cheeks, known as the "slapped cheek" rash. Usually resolves in 2-3 days. Third stage: The rash moves to the arms and legs and becomes a "lacy"-appearing rash that is flat/purple in appearance. May last for a few weeks.

In the majority of children, the first permanent teeth start to erupt at the age of 6 years. Which of the following are the first permanent teeth to erupt in this time period? A) First molars B) Second molars C) Lower or upper incisors D) Canines

A) First molars The first molars are the first permanent teeth to develop; they appear at approximately 6 years of age

A patient has recently been diagnosed with migraine headache with aura. She wants advice on what she should eliminate from her diet to reduce her risk of headaches. The best response is to advise this patient to avoid: A) Foods with aspartame (Equal) because they can trigger migraines B) Foods with monosodium glutamate because they promote diuresis C) Foods with garlic because they promote hypertension D) Foods with nitrates because they do not induce flushing

A) Foods with aspartame (Equal) because they can trigger migraines Dietary triggers for migraine headaches include foods that contain aspartame, tyramine, nitrites, MSG, and/or red wine. These foods are thought to cause a change in the blood vessels and increased blood flow to the brain.

An adolescent male presents to the nurse practitioner for a wellness exam. The patient's face is long and narrow with a prominent forehead and chin, and he has large ears. The mother reports that her son has intellectual disabilities and autistic disorder. Which of the following conditions is being described? A) Fragile X disorder B) Marfan's syndrome C) Turner's syndrome D) Down syndrome

A) Fragile X disorder Fragile X syndrome is the most common form of inherited intellectual disability. The disorder is associated with a higher incidence of autism (especially boys). Males are affected more severely than females. The facial features can vary. The classic facie is a long, narrow face with a prominent forehead and chin and large ears. Other features include hyperlaxity of the joints, flat feet, high arched palate, and others. Definitive diagnosis is by genetic testing.

All of the following statements reflect inadequate breast milk production except: A) Full-term infant is at birth weight by the second week of life B) Less than 6 wet diapers per day or less than 4 stools per day C) Infant is nursing fewer than 8 times per 24-hour period D) Weight loss of > 10% of birth weight

A) Full-term infant is at birth weight by the second week of life The full- term infant should be back to birth weight at 2 weeks of age. The infant should be nursing every 2-4 hours and should wet 6-10 diapers a day (24 h).

Erythromycin 200 mg with sulfisoxazole 600 mg suspension (Pediazole) is contraindicated in which of the following conditions? A) G6PD deficiency anemia B) Lead poisoning C) Beta thalassemia minor D) B12 deficiency anemia

A) G6PD deficiency anemia Glucose-6-phosphate dehydrogenase (G6PD) defi- ciency is a hereditary condition that occurs when the red blood cells break down, causing hemolysis, due to a missing or lack of a sufficient enzyme that is needed to help the red blood cells work efficiently. Certain foods and medications may trigger this reaction and hemolysis will occur. Some of these medications include antimalarial drugs, aspirin, nitrofurantoin, NSAIDs, quinidine, quinine, and sulfa medications.

A 40-year-old female is positive for anti-hepatitis C virus (HCV). Which test is appropriate for follow-up? A) HCV RNA B) HCV antibodies C) HCV core antigen D) Hepatitis C surface antigen

A) HCV RNA HCV RNA is performed following a positive anti-hepatitis C virus test.

Females with polycystic ovarian syndrome are at higher risk for: A) Heart disease and breast cancer B) Uterine fibroids and ovarian cancer C) Premature menopause D) PID (pelvic inflammatory disease)

A) Heart disease and breast cancer Females with PCOS are at higher risk of developing heart disease, stroke, and breast cancer.

Which of the following tests would you recommend to patients to confirm the diagnosis of beta thalassemia or sickle cell anemia? A) Hemoglobin electrophoresis B) Bone morrow biopsy C) Peripheral smear D) Reticulocyte count

A) Hemoglobin electrophoresis Patients with the diagnosis of beta thalassemia and/or sickle cell anemia would be screened using hemoglobin electrophoresis to identify the blood disorder.

Which of the following maneuvers is contraindicated in patients with acute prostatitis? A) Massaging the infected prostate B) Serial urine samples C) Rectal exams D) Palpation of the epididymis

A) Massaging the infected prostate Massaging the infected prostate is contraindicated for acute prostatitis because the massage can spread the bacteria from the prostate into the blood system and lead to sepsis.

A middle-aged Black male complains of a history of outbreaks of painful large nodules and pustules on both his axillae that resolve after treatment with antibiotics. On physical examination, the nurse practitioner notices large red nodules that are tender to palpation. In addition, several pustules are present along with multiple scars on the skin. The nurse practitioner advises the patient that the condition is caused by a bacterial infection of the sweat glands in the axillae. Which of the following conditions is being described? A) Hidradenitis suppurativa B) Severe nodular acne C) Granuloma inguinale D) Cat scratch fever

A) Hidradenitis suppurativa Hidradenitis suppurativa is a chronic skin condition, commonly found in the apocrine glands in the axilla and/or groin, that causes painful nodules under the skin. These abscesses tend to open and drain fluid and pus. Significant scarring of the skin may result from these outbreaks.

A small abscess on a hair follicle on the eyelid is called: A) Hordeolum B) Pterygium C) Pinguecula D) Ptosis

A) Hordeolum Hordeolum is a painful, acute bacterial infection of the hair follicle on the eyelid. Pinguecula is a thickening of the bulbar conjunctiva, located on the inner and outer margins of the cornea. Pterygium is a thickening of the conjunctiva located on the nasal or temporal cornea. Pinguecula and pterygium are both caused by the UV light of long-term sun exposure. Sunglasses with UV protection are recommended to prevent damage to the conjunctiva. Ptosis is the drooping of the upper eyelid.

A positive Chvostek's sign is associated with: A) Hypocalcemia B) Hypernatremia C) Hypokalemia D) Hyperkalemia

A) Hypocalcemia Chvostek's sign is contraction of the facial muscles when the facial nerve is tapped briskly in front of the ear (anterior to the auditory canal). Low calcium levels cause tetany and neuromuscular disturbances. Acute hypocalcemia with symptoms (tetany, weakness, arrhythmias) should be referred to the emergency department. Conditions such as acute or chronic renal failure, Vitamin D or magnesium deficiency, or acute pancreatitis increase the risk of hypocalcemia

What is the name of the immune process that is responsible for anaphylactic reactions? A) IgE-mediated reaction B) Serious allergic reaction C) Antibody reaction D) Atopic reaction

A) IgE-mediated reaction Anaphylaxis is an immunoglobulin E (IgE)-mediated reaction (also known as IgE-mediated type 2). IgE immediate reactions such as anaphylaxis trigger mast cell degranulation and release of potent mediators such as histamines, leukotrienes, and prostaglandins, which immediately induce con- striction of smooth muscle, swelling, vasodilation, and other pathologic changes in the body that may be fatal.

All of the following are considered selective serotonin reuptake inhibitors (SSRIs) except: A) Imipramine (Elavil) B) Fluoxetine (Prozac) C) Sertraline (Zoloft) D) Paroxetine (Paxil CR)

A) Imipramine (Elavil) Prozac, Zoloft, and Paxil CR are selective serotonin reuptake inhibitors. Elavil is a tricyclic antidepressant.

A 44-year-old female who is undergoing treatment for infertility complains of not having a menstrual period for a few months. The night before, she started spotting and is now having cramp-type pains in her pelvic area. Her blood pressure (BP) is 160/80, the pulse rate is 110, and she is afebrile. Her labs reveal a mild anemia with mild leukocytosis. On pelvic exam, the uterine fundus is found above the symphy- sis pubis. The cervical os is dilated at 3 cm. Which of the following is most likely? A) Inevitable abortion B) Threatened abortion C) Incomplete abortion D) Acute pelvic inflammatory disease

A) Inevitable abortion Inevitable abortion is defined as vaginal bleeding with pain and cervical dilation and/or effacement. Threatened abortion is defined as vaginal bleeding with absent or minimal pain and a closed, long, and thick cervix. Incomplete abortion is moderate to diffuse vaginal bleeding, with the passage of tissue and painful uterine cramping or contractions. Acute pelvic inflammatory disease is a sudden onset of inflammation and pain that affects the pelvic area, cervix, uterus, and ovaries, which is caused by infection.

The Phalen test is used to evaluate: A) Inflammation of the median nerve B) Rheumatoid arthritis C) Degenerative joint changes D) Chronic tenosynovitis

A) Inflammation of the median nerve The Phalen test is used to identify inflammation of the median nerve seen with carpal tunnel syndrome. Phalen's test is performed by examining the wrist with full flexion of the wrist for 60 seconds. Positive findings are reproduction of the symptoms such as numbness and tin- gling of the thumb, index finger, and middle finger areas.

All of the following vaccines are contraindicated in pregnant women except: A) Influenza B) Mumps C) Varicella D) Rubella

A) Influenza Influenza vaccine is recommended for all pregnant patients for prevention, due to the decrease in immune status during pregnancy. The vaccine is safe to use during pregnancy. The other vaccines are live viruses and are contraindicated during pregnancy.

Joe, a 20-year-old Asian male, reports pain in his right knee after twisting it playing soccer. The injured knee locks up when he attempts to straighten his leg. Which of the following is most likely? A) Injury to the meniscus of the right knee B) Injury to the patella of the right knee C) Injury to the ligaments of the right knee D) Rupture of the quadriceps tendon

A) Injury to the meniscus of the right knee Pain in the knee after twisting it and "locking up" while attempting to straighten the leg is most consistent with an injury to the meniscus.

What is the significance of a positive Lachman sign? A) Instability of the affected knee caused by damage (i.e.,rupture) to the anterior cruciate ligament (ACL) of the knee B) Posterior cruciate ligament laxity which may cause locking of the affected knee C) Achilles tendon rupture D) Patellar tendon rupture

A) Instability of the affected knee caused by damage (i.e., rupture) to the anterior cruciate ligament (ACL) of the knee A positive Lachman sign is highly suggestive of damage to the anterior cruciate ligament of the knee. The anterior drawer sign may also be positive. There is laxity of the abnormal knee joint (compared with the normal knee). The Lachman test or maneuver is considered more sensitive for ACL damage compared with the anterior drawer test.

Which of the following foods would you advise a new mother to introduce to her 6-month-old infant first? A) Iron-fortified rice cereal B) Plain rice cereal C) Iron-fortified pureed chicken meat D) Plain pureed carrots

A) Iron-fortified rice cereal At 6 months of age, it is recommended that the infant be fed iron-fortified rice cereal. Introducing only one new food at a time is recommended; in case an allergy does develop, you will be able to identify which food caused the allergy.

Atrophic macular degeneration of the aged (AMD) is the leading cause of blindness in the elderly in the United States Which of the following statements is correct? A) It is a slow or sudden painless loss of central vision B) It is a slow or sudden painless loss of peripheral vision C) It is an occlusion of the central retinal vein causing degeneration of the macular area D) It is commonly caused by diabetic retinopathy

A) It is a slow or sudden painless loss of central vision Macular degeneration causes the loss of central vision.

When Molluscum contagiosum is found on the genital area of children, which of the following is the best explanation? A) It should raise the suspicion of child sexual abuse B) It is not considered a sexually transmitted disease C) It is caused by atypical bacteria D) It is caused by the poxvirus and will resolve on its own

A) It should raise the suspicion of child sexual abuse Molluscum contagiosum is spread by skin-to-skin contact. Lesions found in the genital area of young children should be evaluated for suspicion of child sexual abuse.

You are reviewing the bilirubin level on a 3-day-old full-term neonate. You note that it is 10 mg/dL. The infant has a slight yellow color to his skin, mucous membranes, and sclera. The infant is feeding well, is not irritable, and has 8 to 10 wet diapers per day. Which of the following is a true statement? A) Keep on monitoring the infant's bilirubin level until it returns back to normal in about 1 week B) Recommend that the infant be treated with phototherapy 10 minutes a day until the bilirubin level is back down to a normal range C) Refer the infant to a neonatologist as soon as possible D) Refer the infant to the neonatal intensive care unit

A) Keep on monitoring the infant's bilirubin level until it returns back to normal in about 1 week Bilirubin is excreted through the urine and feces. Increased fluids and wetting 8-10 diapers a day is sufficient fluid intake/excretion to help bring down the bilirubin level. Levels should continue to be monitored and should improve in approximately 1 week.

A patient who recently returned from a vacation in Latin America complains of a severe headache and stiff neck that are accompanied by a high fever for the past 12 hours. While examining the patient, the nurse practitioner flexes both the patient's hips and legs and then tells the patient to straighten them against resistance. The name of this test is: A) Kernig's maneuver B) Brudzinski's maneuver C) Murphy's sign D) Homan's sign

A) Kernig's maneuver Kernig's maneuver is performed by having the patient flex both hips and legs and then straighten the legs against resistance, testing for meningitis. Flexion of hip/knees is a positive sign for meningitis. Brudzinski's maneuver is performed by placing the patient's hands behind his head, and gently tucking chin to chest. Murphy's sign is elicited by having the patient inspire with the tips of the examiner's fingers placed on the right upper quadrant, at the liver border, under the ribs. Pain on inspiration is suggestive of cholecystitis. Homan's sign is flexion of the foot, causing pain in the posterior calf area, suggestive of a DVT.

The Lachman maneuver is used to detect which of the following? A) Knee instability B) Nerve damage of the knee due to past knee injuries C) The integrity of the patellar tendon D) Tears on the meniscus of the knee

A) Knee instability The Lachman maneuver is a test performed to assess for knee instability. Knee instability indicates a tear of the anterior cruciate ligament.

A red, raised serpiginous-shaped rash is noted by the nurse practitioner on the right foot of a 4-year-old child brought in for a preschool physical by the mother. The child complains of severe itch and keeps scratching the lesion. The mother reports that the child frequently played in the yard without shoes or sandals on. Which of the following is most likely? A) Larva migrans B) Erythema migrans C) Tinea pedis D) Insect bites

A) Larva migrans Larva migrans affects the intestines of dogs/cats and is caused by the eggs of parasites (worms) from the animals' intestine. Children are at high risk of developing this if they come in contact with dirt that is contaminated with dog/cat feces. Eating foods that are grown in contaminated soil and/or raw liver are other means of transfer of the infection. After the eggs hatch, the parasite can migrate to other organs of the body if left untreated

Human papilloma virus (HPV) infection of the larynx has been associated with: A) Laryngeal neoplasia B) Esophageal stricture C) Cervical cancer D) Metaplasia of esophageal squamous cells

A) Laryngeal neoplasia HPV infection of the larynx has been associated with laryngeal neoplasia; HPV infection of the cervix is associated with cervical cancer.

Which of the following is considered an abnormal result on a Weber test? A) Lateralization to one ear B) No lateralization in either ear C) Air conduction lasts longer than bone conduction D) Bone conduction lasts longer than air conduction

A) Lateralization to one ear The Weber test is performed by placing a tuning fork on top of the head. A normal Weber test is when no lateralization occurs. Lateralization to one ear occurs with hearing impairment.

The complications of untreated gout include: A) Loss of joint mobility and renal failure B) Loss of joint mobility and liver failure C) An increased risk of urinary tract infections D) Bladder cancer

A) Loss of joint mobility and renal failure Complications of untreated gout are loss of joint mobility and renal failure. High uric acid levels can also lead to kidney stones.

The most common cause of cancer deaths in males is: A) Lung cancer B) Prostate cancer C) Colon cancer D) Skin cance

A) Lung cancer Lung cancer is the most common cause of cancer deaths in men. Prostate cancer and colon cancer are the second and third causes of cancer death in men.

The following abnormal lab results may be seen in patients with acute mononucleosis except: A) Lymphocytosis and/or atypical lymphocytes B) Positive EBV titers for immunoglobulin (Ig) M and Ig G C) Elevated liver function tests D) Elevated creatinine and BUN

A) Lymphocytosis and/or atypical lymphocytes Common symptoms of mononucleosis include positive EBV titers, elevated liver enzymes, and elevated creatinine and BUN.

Which of the following is considered a spiritual illness by Latinos/Hispanics that can cause symptoms such as loss of appetite, crying, diarrhea, and weakness or death among infants and small children? A) Mal ojo or mal de ojo B) Chronic nightmares C) Trabajo D) Malo

A) Mal ojo or mal de ojo Mal ojo or mal de ojo is a spiritual illness that can cause symptoms such as loss of appetite, crying, diarrhea, colic, fear, weakness, or death. A curandero or curandera is usually consulted and does spiritual cleansing of the patient. It may take several cleansings (limpia) to cure the patient. Trabajo means "work" and malo means "bad"; these are being used as distractors.

Patient confidentiality is breached when: A) Medical information is given to a spouse B) Records are subpoenaed C) Reports are sent to the Public Health Department D) Records are released to insurance companies

A) Medical information is given to a spouse Patient confidentiality is breached when medical information is given to a spouse or any other individual without consent of the patient.

A 19-year-old male athlete complains of acute knee pain after a football game. The nurse practitioner elicits McMurray's sign, which is positive on the patient's injured knee. This is a test for: A) Meniscal injury B) Inflammation of the knee joint C) Osteophytes of the knee joint D) Tenosynovitis

A) Meniscal injury With an acute knee injury, the knee should be assessed using McMurray's sign. A positive McMurray's sign indicates a meniscal injury. Inflammation of the knee, osteophytes, and tenosynovitis would not elicit a positive McMurray's sign.

A 25-year-old woman complains of dysuria, severe vaginal pruritis, and a malodorous vaginal discharge. Pelvic examination reveals a strawberry-colored cervix and frothy yellow discharge. Microscopic exam of the discharge reveals mobile organisms that have flagella. The correct pharmacologic therapy for the condition is: A) Metronidazole (Flagyl) B) Ceftriaxone sodium (Rocephin) C) Doxycycline hyclate (Vibramycin) D) Clotrimazole (Gyne-Lotrimin)

A) Metronidazole (Flagyl) Trichomoniasis symptoms include dysuria, severe vaginal pruritis, and malodorous vaginal discharge. Wet prep will show trichomonads that are pear-shaped and have several flagella (whiplike tails) at one end. CDC recommendation for treatment is metronidazole.

Epidemiologic studies show that Hashimoto's disease occurs most commonly in: A) Middle-aged to older women B) Smokers C) Obese individuals D) Older men

A) Middle-aged to older women Hashimoto's disease commonly occurs in middle-aged to older women.

Medicare Part A will pay for all of the following services except: A) Minor surgery in a walk-in surgical center B) Plastic surgery to repair facial damage from a burn C) Kidney transplantation D) Medical supplies and drugs that are used while the patient is in the hospital

A) Minor surgery in a walk-in surgical center Medicare Part A will pay for medically necessary inpatient care and supplies. Therefore, any type of surgery that is done in outpatient settings such as a walk-in surgical center will not be reimbursed. If plastic surgery is medically necessary (i.e., plastic surgery to repair facial damage from a burn), then it will be reimbursed. Costs for organ transplantation such as kidney transplants are reimbursed.

Stella, a new mother, complains to you that she has been feeling irritable and jittery almost daily for the past few months. She complains of frequent palpitations and more frequent bowel movements along with weight loss. Her BP is 160/70, her pulse is 110, and she is afebrile. All of the following conditions should be considered in the differential diagnosis for this patient except: A) Mitral regurgitation B) Graves' disease C) Generalized anxiety disorder D) Illicit drug use

A) Mitral regurgitation Signs and symptoms of mitral regurgitation do not include frequent bowel movements with weight loss.

A nurse practitioner is taking part in a community outreach program for a local hospital. Most of her audience has a diagnosis of hypertension. They are all interested in learning more about a proper diet. When discussing potential sources of potassium and magnesium, which of the following is the best advice? A) Most fruits and vegetables B) Whole grains and sausages C) Processed corned beef and yogurt D) Mushrooms and sauerkraut

A) Most fruits and vegetables Fruits and vegetables are higher in potassium and magnesium.

Orchitis is caused by which of the following? A) Mumps virus B) Measles virus C) Chlamydia trachomatis D) Chronic urinary tract infections that are not treated adequately

A) Mumps virus Orchitis is caused by the mumps virus.

Precocious puberty is defined as: A) Onset of puberty before age 8 in girls and age 9 in boys B) Onset of puberty before age 9 in girls and age 10 in boys C) Onset of puberty before age 10 in girls and age 11 in boys D) Onset of puberty at an earlier age than the child's parent(s)

A) Onset of puberty before age 8 in girls and age 9 in boys The definition of precocious puberty is the onset of puberty before age 8 in girls and age 9 in boys.

Jenny, a 21-year-old, complains to you of a 1-week episode of dysuria, frequency, and a strong odor to her urine. This is her second episode of the year. What is the most appropriate follow-up for this patient? A) Order a urinalysis and urine for culture and sensitivity (C&S) and treat the patient with antibiotics B) Order a urine C&S and hold treatment until you get the results from the lab C) Treat the patient with a 7-day course of antibiotics and order a urine for culture and sensitivity (urine C&S) now and after she completes her antibiotics D) Treat the patient with a stronger drug such as ofloxacin (Floxin) for 10 days

A) Order a urinalysis and urine for culture and sensitivity (C&S) and treat the patient with antibiotics The best treatment for this patient is to order the urinalysis and urine C&S to identify the organism causing the infection. Treatment may begin while waiting for the culture results. However, occasionally treatment may have to be changed after the culture/sensitivity results return, due to resistance to the antibiotic used.

Stella works in the housekeeping department of a hospital. She presents to the employee health clinic with a complaint of a needlestick to her left thumb. The needle was in one of the garbage bags from the emergency room. The patient had a little bleeding that stopped spontaneously. Which of the following is the next step? A) Order an enzyme-linked immunosorbent assay (ELISA) test as soon as possible B) Recommend a tetanus booster in 1 week C) Offer the patient hepatitis B immunoglobulin D) Order a chest x-ray

A) Order an enzyme-linked immunosorbent assay (ELISA) test as soon as possible Employee health clinic protocols for needlesticks recommend ordering an ELISA test as soon as possible to establish baseline blood work for the employee.

A second triple screen on a 35-year-old primigravida reveals abnormally low lev- els of the alpha fetoprotein and estriol and high levels of human chorionic gonad- otropin. Which of the following interventions is the best choice for this patient? A) Order an ultrasound B) Order a computed tomography (CT) scan of the abdomen C) Order a 24-hour urine for protein clearance D) Assess for a history of illicit drug or alcohol use

A) Order an ultrasound Abnormally low levels of alpha fetoprotein and estriol and high levels of human chorionic gonadotropin are abnormal during pregnancy. An ultrasound should be ordered to further evaluate the fetus for characteristics of Down syndrome and/or fetal demise.

A 60-year-old female truck driver presents to the outpatient urgent care clinic of a hospital complaining of the worsening of her low back pain the past few days. She describes the pain as "sharp and burning" and points to the left buttock. She reports that the pain started on the mid-buttock of the left leg and recently started to go down the lateral aspect of the leg toward the top of the foot. During the physical exam, the ankle jerk and the knee jerk reflex is 1+ on the affected leg and 2+ on the other leg. The pedal, posterior tibialis, and popliteal pulses are the same on both legs. Which of the following tests should the nurse practitioner consider for this patient? A) Order both a plain radiograph and computed tomography (CT) scan of the spine as soon as possible B) Write a prescription for ibuprofen 800 mg PO QID with a muscle relaxant and advise the patient to follow up with her primary care provider within 3 days C) Refer the patient to an orthopedic surgeon D) Ordering an imaging study of the spine is premature because the majority of low back pain cases resolve within 10 to 12 weeks

A) Order both a plain radiograph and computed tomography (CT) scan of the spine as soon as possible The patient has a severe case of sciatica that is worsening quickly (may progress to cauda equina). In addition, older age and an abnormal neurologic exam are red flags for potentially serious underlying pathology (osteoporosis fracture, cancer, infection, spondylolisthesis). Because the patient's symptoms are worsening, a CT scan is the best choice. A plain radiograph can only detect bony pathology, but a CT scan can detect nerve root compression, herniated disc, cancer, and spinal stenosis (narrowing of the spinal canal). In addition, the patient needs to follow up with a neurologist as soon as possible.

The atypical antipsychotic drugs have many adverse effects. Which of the following side effects is most likely to be seen with this drug class? A) Orthostatic hypotension and sedation B) Malignant hypertension and headache C) Skin hyperpigmentation and alopecia D) Severe anxiety and increased appetite

A) Orthostatic hypotension and sedation Orthostatic hypotension and sedation are common side effects of atypical antipsychotics such as olanzapine (Zyprexa), quetiapine (Seroquel), and risperidone (Risperdal). It is also a common side effect of the older antipsychotics like haloperidol (Haldol). Antipsychotics do not cause severe anxiety and hyperphagia (increased appetite). They lower anxiety and cause sedation, sleepiness, anorexia, and hypotension and increase the risk of sud- den death in frail elders

The positive signs of pregnancy include: A) Palpation of the fetus and auscultation of the fetal heart tones by the nurse practitioner B) Palpation of the fetus and a positive quantitative serum pregnancy test C) Fetal heart tones and a positive quantitative serum pregnancy test D) Fetal heart tones and feeling of movement of the baby by the mother

A) Palpation of the fetus and auscultation of the fetal heart tones by the nurse practitioner Presumptive signs of pregnancy are symptoms experienced by the woman, such as amenorrhea, breast tenderness, nausea/vomiting, fatigue, and increased urinary frequency. Probable signs of pregnancy are signs detected by the examiner, such as an enlarged uterus. Positive signs of pregnancy are direct evidence of pregnancy such as audible fetal heart tones or cardiac activity on ultrasound.

Which of the following medications is indicated for the treatment ofobsessive-compulsive disorder? A) Paroxetine (Paxil CR) B) Haldoperidol (Haldol) C) Lorazepam (Xanax) D) Imipramine (Elavil

A) Paroxetine (Paxil CR) Antidepressants are the most common medications used for OCD. Those antidepressants that are approved for OCD by the Food and Drug Administration (FDA) include clomipramine (Anafranil), fluvox- amine (Luvox), fluoxetine (Prozac), paroxetine (Paxil, Pexeva), and sertraline (Zoloft).

A 30-year-old chef complains of pruritic hives over her chest and arms but denies difficulty swallowing or breathing. She reports a family history of allergic rhinitis and asthma. Which of the following interventions is most appropriate? A) Perform a complete and thorough history B) Prescribe an oral antihistamine such as diphenhydramine 25 mg PO QID C) Give an injection of epinephrine 1:1000 intramuscularly stat D) Call 911

A) Perform a complete and thorough history Prior to prescribing medications, a complete and thorough history must be performed to determine possible causes of hives. The patient denied difficulty with swallowing and breathing, so there was no medical emergency to require calling 911.

A test called the visual fields by confrontation is used to evaluate for: A) Peripheral vision B) Central distance vision C) Narrow-angle glaucoma D) Accommodation

A) Peripheral vision The visual fields of confrontation test is used to evaluate peripheral vision. The Snellen chart is used to measure central distance vision. A tonometer is used to assess for glaucoma. The ophthalmoscope is used to assess for cataracts.

Human chorionic gonadotropin (hCG) is produced by the: A) Placenta B) Hypothalamus C) Anterior pituitary D) Ovaries

A) Placenta HCG is produced by the placenta.

All of the following foods are best avoided by individuals with celiac sprue except: A) Rice cereal B) Blueberry muffins C) Organic wheat bread D) Rye bread

A) Rice cereal Patients can eat any food except those that contain the protein gluten. Foods containing wheat, barley, and rye should be avoided.

A 10-year-old male who was recently accepted into his school's soccer team has a history of exercise-induced asthma. The child wants to know when he should take his albuterol inhaler. The nurse practitioner would advise the patient: A) Premedicate himself 20 minutes prior to starting exercise B) Wait until he starts to exercise before using the inhaler C) Premedicate 60 minutes before starting exercise D) Wait until he finishes his exercise before using his inhaler

A) Premedicate himself 20 minutes prior to starting exercise Exercise-induced asthma is best controlled by using the Proventil inhaler (bronchodilator) approximately 20 minutes prior to exercise, to prevent vasospasm of the bronchioles and shortness of breath with exercise. These bronchodilators usually last approximately 4 hours. They also work quickly to open up the bronchioles if an acute attack/shortness of breath occurs

A charitable foundation plans to build a community youth center in a large urban area with a history of gang violence. What type of health prevention activity is being done in this area? A) Primary prevention B) Secondary prevention C) Tertiary prevention D) Health prevention

A) Primary prevention A community youth center with good staffing can be an effective method of drawing the children out of the streets into a safer environment. It can reduce the risk of children becoming victims of gang violence. In addition, staff members can serve as role models or mentors for the older children.

All of the following are included in the criteria used to diagnose patients with AIDS except: A) Profound fatigue B) Thrush C) Kaposi's sarcoma D) Hairy leukoplakia of the tongu

A) Profound fatigue Criteria used to diagnose AIDS include hairy leukoplakia of the tongue, Kaposi's sarcoma, and thrush.

Which of the following is recommended as first-line treatment for essential tremor? A) Propranolol (Inderal) B) Phenytoin (Dilantin) C) Amitriptyline (Elavil) D) Fluoxetine (Prozac)

A) Propranolol (Inderal) Propranolol (Inderal) is approved for "treatment" of essential tremor. It helps control the symptoms. Essential tremors are permanent and cannot be cured. Before prescribing, order an EKG. Do not use beta- blockers if a patient has second- or third-degree heart block or chronic lung disease.

A 75-year-old patient has a history of benign prostatic hypertrophy (BPH). During the physical exam, which of the following clinical findings during the prostatic exam is correct? A) Prostate feels firm and uniformly enlarged B) Prostate feels boggy and enlarged C) Prostate feels harder than normal D) Presence of tender nodules

A) Prostate feels firm and uniformly enlarged The prostate should feel firm and will be uniformly enlarged. A boggy and warm prostate is present with acute prostatitis.

Which bacterium is the most common pathogen seen in otitis externa infections? A) Pseudomonas aeroginosa B) Streptococcus pyogenes C) Haemophilus influenza D) Moraxella catarrhalis

A) Pseudomonas aeroginosa The most common bacterium is Pseudomonas. The second most common bacterium is Staphylococcus aureas. Polymyxin and neomycin combination ear drops (Cortisporin) are the first-line treatment for otitis externa. Other ear drops that are also effective are the quinolone ear drops (ofloxacin, ciprofloxacin topical drops).

A 55-year-old woman who has type 2 diabetes is concerned about her kidneys. She has a history of 3 urinary tract infections within the past 8 months. She denies dysuria and frequency at this visit. Which of the following is the best initial course to follow? A) Recheck the patient's urine and order a urine for culture and sensitivity B) Order an IVP (intravenous pyelogram) C) Advise the patient to follow up with a urologist D) Evaluate the patient for a possible kidney infection

A) Recheck the patient's urine and order a urine for culture and sensitivity A urinary tract infection is defined as the presence of 100,000 organisms per mL of urine in asymptomatic patients or greater than 100 organisms per mL or urine with pyuria (more than 7 WBCs/mL) in a symptomatic patient

Lifestyle modifications are an important aspect in the treatment of hypertension. Which of the following statements is incorrect? A) Reduce intake of sodium, potassium, and calcium B) Reduce intake of sodium and saturated fats C) Exercise at least three to four times per week D) Maintain an adequate intake of potassium, magnesium, and calcium

A) Reduce intake of sodium, potassium, and calcium Lifestyle modifications for hypertension include exercise 3-4 times a week, diet modifications of reduced intake of sodium and saturated fats, and adequate dietary intake of potassium, magnesium, and calcium

Which of the following is the best course of treatment for this patient? A) Refer him to an orthopedic specialist B) Refer him to a chiropractor C) Advise him that the clicking noise will resolve within 2 to 4 weeks D) AdvisehimtouseanAcebandagewrapduringthefirst2weeksforkneesup- port and to see you again for reevaluation

A) Refer him to an orthopedic specialist Referral to orthopedics is advised for evaluation of the need for treatment and surgery. Delaying referral and treatment can be detrimental if the diagnosis is not correct.

A 28-year-old male nurse tells the employee health nurse practitioner that he was treated for a UTI twice the previous year. The patient denies fever, flank pain, or urethral discharge during the visit. Which of the following is the best follow-up for this patient? A) Refer the patient to a urologist B) Prescribe the patient ofloxacin (Floxin) for 2 weeks instead of 1 week C) Advise the patient that he needs to void every 2 hours when awake D) Refer the patient to the local ED, because he has a very high risk of sepsis

A) Refer the patient to a urologist It is recommended that frequent UTIs in males be referred to a urologist for evaluation and treatment.

An 87-year-old male reports to the nurse practitioner that his grandson locks him in the bedroom when the grandson goes out of the house and sometimes with- holds food from him if he does not give the grandson spending money. The patient appears frail, with poor grooming, and has a strong odor of urine on his clothing. Which of the following is the best action for the nurse practitioner to take? A) Report the patient's grandson for elder abuse to protective health services of the state B) Call the grandson from the waiting room and educate him about the importance of proper grooming for his grandfather C) Advise the grandson that if the patient reports the same complaints the next time he is seen by the nurse practitioner, he will be reported for elder abuse to the state authorities D) Advise the patient that he should call his son as soon as possible

A) Report the patient's grandson for elder abuse to protective health services of the state Speaking with the grandson and warning him about elder abuse and reporting may result in harm to the patient and/or refusal to return to the clinic in the future for follow-up of the patient. Option D is a vague answer (call his son as soon as possible for what?).

The nurse practitioner examines a 4-week-old boy whose mother reports that he has cried for at least 3 hours a day at the same time of day since birth. What is the main goal in the clinical evaluation of this infant? A) Rule out any physiologic cause for the crying spells B) Make sure that the infant is well clothed C) Evaluate the environment D) Order laboratory and diagnostic testing

A) Rule out any physiologic cause for the crying spells For infants that cry for several hours during the day, ruling out a physiological problem that may be causing the distress is recommended.

The bell of the stethoscope is best used for auscultation of which of the following? A) S3 and S4 and low-pitched tones B) S3 and S4 only C) S1 and S2 and high-pitched tones D) S1 and S2 only

A) S3 and S4 and low-pitched tones The bell of the stethoscope is used to auscultate the S3, S4, and low-pitcehd tones. Remember the word "below" (Bell-Low).

Which of the following drugs is most likely to cause sexual dysfunction in males? A) SSRIs B) ACE inhibitors C) Amphetamines D) Atypical antidepressants

A) SSRIs A common side effect of SSRIs (e.g., Prozac, Paxil, Zoloft) is sexual dysfunction in males. For depressed males, atypical antidepressants such as bupropion (Wellbutrin) cause less sexual dysfunction.

A 40-year-old nurse complains of a new onset of back pain secondary to her job on the medical-surgical floor of a hospital. She reports lifting some obese patients while working the previous night shift. She reports to the worker's compensation clinic where she was referred. She describes the pain as starting in her right but- tocks area and radiating down the back of her thigh. It becomes worse when she sits down for long periods. You would suspect: A) Sciatica B) Acute muscle spasm C) Cauda equina syndrome D) Acute muscle strain

A) Sciatica Sciatica is defined as pain that begins in the buttock area and radiates down one leg. Other symptoms include weakness and tingling sensation. Acute muscle spasm and strain do not cause tingling down the leg. Cauda equina syndrome is an emergent issue, in which there is neurological involvement and patient complaints of weakness and loss of bladder and bowel control.

On auscultation of the chest, a split S2 is best heard at: A) Second intercostal space, right sternal border B) Second intercostal space, left sternal border C) Fifth intercostal space, midclavicular line D) Fourth intercostal space, left sternal border

A) Second intercostal space, right sternal border A split S2 is best heard at the second ICS, right of the sternal border.

Which of the following laboratory values may be found elevated alone on the liver function panel of patients who are alcohol abusers? A) Serum GGT (gamma glutamyl transaminase) B) Serum ALT (alanine aminotransferase) C) Serum bilirubin D) Blood urea nitrogen

A) Serum GGT (gamma glutamyl transaminase) The serum gamma glutamyl transaminase alone will be elevated with alcohol abusers.

If left untreated, Zollinger-Ellison syndrome can cause which of the following? A) Severe ulceration of the stomach or duodenum B) Toxic megacolon C) Chronic diarrhea D) Malabsorption of fat-soluble vitamins

A) Severe ulceration of the stomach or duodenum Untreated Zollinger-Ellison syndrome can lead to severe ulceration of the stomach or duodenum. Zollinger- Ellison syndrome occurs when tumors (gastrinomas) in the intestine, pancreas, or lymph nodes near the pancreas produce excessive amounts of gastrin, which in turn will increase the amount of acid produced by the stomach. High amounts of acid in the stomach will produce ulceration of the stomach or duodenum.

Your newly diagnosed diabetic patient reports to you that she had severe hives and swollen lips when she took Bactrim for a bladder infection 2 months ago. Which of the following statements is correct? A) She cannot take any pills in the sulfonylurea class B) She can take some of the pills in the sulfonylurea class C) She can take any of the pills in the sulfonylurea class D) None of the above

A) She cannot take any pills in the sulfonylurea class If an allergic reaction occurs with Bactrim, the patient must not take any medications in the sulfonylurea class.

A 15-year-old White male is brought in by his father for a physical exam. He is concerned that his son is "too short" for his age. The father reports that when he was the same age, he was much taller. His son wants to try out for the football team, but the father is concerned because his son might be "too short" to join. Which of the follow- ing physical exam findings is worrisome? A) Small smooth testicles with no pubic or facial hair B) Smooth testicles with rugated scrotum that is a darker color than his normal skin color C) Smooth testicles with coarse and curly pubic hair D) Straight pubic and axillary hair with a long thin penis

A) Small smooth testicles with no pubic or facial hair Small, smooth testicles with no pubic or facial hair (Tanner Stage I) is a worrisome finding at the age of 14 years because it signifies that the boy is not in the pubertal stage yet. The average age of onset of puberty among boys is 12 years (range 10 to 14 years). The maximum growth spurt in boys occurs about 2 years after the onset of puberty. Boys start about 1 year later than girls and continue to grow until their early 20s (college).

Carol, a 30-year-old type 2 diabetic, is on regular insulin and lente insulin in the morning and in the evening. She denies changes in her diet or any illness, but recently started attending aerobic classes in the afternoon. Because of her work- outs, her blood sugars have dipped below 50 mg/ dL very early in the morning. Her fasting blood sugar before breakfast is now elevated and higher than normal. Which of the following is best described? A) Somogyi phenomenon B) Dawn phenomenon C) Raynaud's phenomenon D) Insulin resistance

A) Somogi phenomenon The Somogi phenomenon is when nocturnal hypoglycemia (2 a.m to 3 a.m.) stimulates the liver to produce glucagon to raise the blood sugar. The fasting blood glucose levels will be elevated from this glucagon production.

The most common type of skin malignancy is: A) Squamous cell cancer B) Basal skin cancer C) Melanoma D) Dysplastic nevi

A) Squamous cell cancer Squamous cell cancer is the most frequent type of skin cancer. It can occur in normal skin or inflamed skin. Common areas involved are those which are exposed to the sun or UV light. The earliest form of squamous cell cancer is known as Bowen's disease, in which the cancer has not involved the tissue nearby.

The bacterium responsible for the highest mortality in patients with community- acquired pneumonia is: A) Streptococcus pneumoniae B) Mycoplasma pneumoniae C) Moraxella catarrhalis D) Haemophilus influenzae

A) Streptococcus pneumoniae Steptococcus pneumoniae is known for being responsible for the highest mortality in patients with community-acquired pneumonia.

Which of the following drug classes is recommended for the treatment of post- herpetic neuralgia? A) TCAs (tricyclic antidepressants) B) SSRIs (selective serotonin reuptake inhibitors) C) Atypical antidepressants D) Benzodiazepines

A) TCAs (tricyclic antidepressants) Tricyclic antidepressants and anticonvulsants are recommended for post-herpetic neuralgia. These medications may help with the neuropathic pain.

A 30-year-old male with a history of gout is walking to the examination room and the nurse practitioner notices that he is limping. When the patient sits down, the nurse practitioner notes that the metatarsophalangeal joint of the great toe is very swollen and is bright red. The patient reports that he was attending a party the night before and drank. The patient is requesting a prescription to treat his painful toe. The nurse practitioner prescribes the patient indomethacin (Indocin) 50 mg TID PRN and colchicine. Regarding colchicine, which of the following instructions is correct? A) Take 1 pill every hour to every 2 hours until relief is obtained or adverse gastrointestinal effects occur, such as abdominal pain, nausea, or diarrhea B) Take 1 pill every hour until relief is obtained up to 24 hours C) Take 1 pill every 4 to 6 hours until the pain is relieved D) Take 2 to 3 pills QID until relief is obtained or adverse GI effects occur, such as abdominal pain, nausea, or diarrhea

A) Take 1 pill every hour to every 2 hours until relief is obtained or adverse gastrointestinal effects occur, such as abdominal pain, nausea, or diarrhea Colchicine acts as an anti-inflammatory and helps to suppress gouty attacks. It is usually taken as 1 tablet (0.6 mg) every 1 to 2 hours until relief is obtained (or adverse gastrointestinal effects occur, such as abdominal pain, nausea, or diarrhea). Prescribe the patient only 10 tablets at a time (do not refill) during a flare-up. The maximum dose is 6 mg/day. Many patients will develop GI adverse effects even before the pain is relieved. Colchicine can also be taken daily in small amounts for prophylaxis.

A 14-year-old female who is sexually active is brought in by her mother for an immunization update. According to the mother, her daughter had 1 dose of hepatitis B vaccine. Which of the following vaccines would you administer at this visit? A) Td and hepatitis B B) DTaP (diphtheria, tetanus, acellular pertussis) and hepatitis B C) Hepatitis B only D) MMR (measles, mumps, rubella) and Td

A) Td and hepatitis B The CDC recommends Td and hepatitis B for 14-year-old females. It is recommended for her to continue the hepatitis series, which includes a total of 3 injections.

Which of the following clinical findings can mimic a case of testicular torsion but is not considered an emergent condition? A) The "blue dot" sign B) One swollen testicle with yellow-colored penile discharge C) An acute onset of dysuria and frequency D) Avaricocele

A) The "blue dot" sign The "blue dot" sign is located underneath the skin of the testicle and appears as a round, blue to purple mass. It is not an emergent condi- tion. Also known as the torsion of testicular appendage.

The Pap smear result for a 20-year-old sexually active college student who used condoms inconsistently reveals a large number of white blood cells and blood along with inflammatory changes. During the speculum exam, the nurse practitioner who examined the patient noticed that the patient's cervix bled very easily (friable) and a small amount of purulent discharge was present on the cervical surface. No cervical motion tenderness was noted during the bimanual vaginal exam. What is the next step in the management of this patient? A) The NP needs to do cervical cultures to verify gonorrhea B) Prescribe metronidazole vaginal cream for the patient over the phone C) Call the patient and tell her she needs a repeat Pap smear in 6 months D) Advise her to use a Betadine douche at bedtime x 3 days

A) The NP needs to do cervical cultures to verify gonorrhea Cultures should be taken at the time of the Pap smear, as the patient may not return for later diagnostic testing.

The Pap smear result on a 20-year-old sexually active student who uses condoms inconsistently shows a large amount of inflammation. Which of the following is the best follow-up action? A) The NP needs to do cervical cultures to verify the presence of gonorrhea B) Prescribe metronidazole vaginal cream for the patient over the phone C) Call the patient and tell her she needs a repeat Pap smear in 6 months D) Advise the patient to use a Betadine douche at bedtime x 3 days

A) The NP needs to do cervical cultures to verify the presence of gonor- rhea Cultures should be taken at the time of the Pap smear, as the patient may not return for later diagnostic testing.

Nurse practitioners and clinical nurse specialists derive their legal right to practice from: A) The Nurse Practice Act of the state where they practice B) The laws of the state where they practice C) The Medicare statute D) The Board of Nursing in the state where they practice

A) The Nurse Practice Act of the state where they practice The Nurse Practice Act provides nurses with the legal right to practice nursing. The State Board of Nursing is the agency that enforces the Nurse Practice Act. The Medicare statute provides the funds for paying for health services at the age of 65 years and older.

When evaluating the blood pressure on both the arms and legs of an infant who has a diagnosis of coarctation of the aorta, which of the following is the correct finding? A) The blood pressure is higher in the arms than in the legs B) Only the diastolic blood pressure is higher in the legs than in the arms C) The blood pressure is higher in the legs than the arms D) The blood pressure is lower in both arms than in the legs

A) The blood pressure is higher in the arms than in the legs In coarctation of the aorta, blood pressure is higher in the arms than in the legs due to the narrowing in the aorta. Blood pressure must rise to get adequate blood flow to the lower extremities; therefore, the blood pressure above the coarctation rises to compensate for this.

RhoGAM's mechanism of action is: A) The destruction of Rh-positive fetal RBCs that are present in the mother's circulatory system B) The destruction of maternal antibodies against Rh-positive fetal RBCs C) The stimulation of maternal antibodies so that there is a decreased risk of hemolysis D) The destruction of maternal antibodies against fetal RBCs

A) The destruction of Rh-positive fetal RBCs that are present in the mother's cirulatory system RhoGAM is given to mothers with Rh-negative blood when the fetus has Rh-positive blood. RhoGAM protects the mother from developing antibodies by destroying the Rh-positive fetal RBCs in the mother's blood system.

All of the following are factors important in determining the peak expiratory flow volume except: A) Weight B) Height C) Age D) Gender

A) Weight Peak expiratory flow volume is determined by using height, gender, and age.

A 30-year-old female complains of having no period for the last 12 weeks. She is sexually active and has been using condoms inconsistently. The patient has a his- tory of irregular menstrual cycles and severe dysmenorrhea. The urine pregnancy test result is positive. Which of the following is a true statement regarding this pregnancy? A) The fundus of the uterus should be at the level of the symphysis pubis B) The cervix should be dilated about 0.5 in. at this time of gestation C) "Quickening" starts during this period D) Hegar's sign is present during this period of pregnancy

A) The fundus of the uterus should be at the level of the symphysis pubis At 12 weeks gestation, the fundus of the uterus should be located approximately at the symphysis pubis.

A patient who is on warfarin sodium (Coumadin) is advised to avoid eating large amounts of leafy green vegetables because: A) The high Vitamin K levels will increase bleeding time B) They have too much ascorbic acid, which can interact with the medicine C) The high fiber content will decrease the absorption of the Coumadin D) The vitamins in the vegetables will bind and inactivate Coumadin

A) The high Vitamin K levels will increase bleeding time Foods containing Vitamin K may enhance the anticoagulation effect of Coumadin, thus increasing bleeding time.

A positive Coombs test on an Rh-negative pregnant woman means: A) The mother has autoantibodies against Rh-positive red blood cells (RBCs) B) The fetus has autoantibodies against maternal RBCs C) The mother does not have Rh factor against fetal RBCs D) The fetus does not have RBC autoantibodies

A) The mother has autoantibodies against Rh-positive red blood cells (RBCs) The mother's autoantibodies can attack the fetus's Rh-positive red blood cells and cause destruction of these cells, which can cause severe anemia and com- plications in the fetus. Today this is preventable with the administration of anti- RhD immunoglobulin (Rho(D) immune globulin) to an Rh-negative mother at 28 weeks gestation and after birth if the newborn is Rh positive.

During a sports physical, you note that the vision of an 18-year-old male athlete is 20/30 in both eyes. Which of the following statements is true? A) The patient can see at 20 ft what a person with normal vision can see at 30 ft B) The patient can see at 30 ft what a person with normal vision can see at 20 ft C) The patient cannot engage in contact sports D) The patient needs to be referred to an ophthalmologist

A) The patient can see at 20 ft what a person with normal vision can see at 30 ft When vision results are 20/30 in both eyes, this means that the patient can see at 20 ft what a person with normal vision can see at 30 ft.

During the physical exam of a 60-year-old adult, the nurse practitioner performs an abdominal exam. The nurse practitioner is checking the left upper quadrant of the abdomen. During percussion, an area of dullness is noted beneath the lower left ribcage. Which of the following is a true statement regarding the spleen? A) The spleen is not palpable in the majority of healthy adults B) The spleen is 8 cm to 10 cm in the left midaxillary line at its longest axis C) The spleen is 2 cm to 6 cm between the ninth to eleventh ribs on the left midaxillary line D) The splenic size varies depending on the patient's gender

A) The spleen is not palpable in the majority of healthy adults The spleen is located on the left upper quadrant of the abdomen under the diaphragm and is protected by the lower ribcage. In the majority of adults, it is not palpable. The spleen's longest axis is from 11 cm to 20 cm. Any spleen larger than 20 cm is enlarged. The best test for evaluating splenic (or hepatic) size is the abdominal ultrasound. Disorders that can cause splenomegaly include mononucleosis, sickle cell disease, congestive heart failure, bone marrow cancers (myeloma, leukemia), and several other diseases.

Research nomenclature assigns which of the following symbols to indicate a sub- population or a subgroup within the total population? A) The symbol "n = " B) The symbol "N = " C) The symbol "p = " D) The symbol "P = "

A) The symbol "n = " The correct symbol to indicate a subpopulation of a sample is the small letter "n." For example, if a research study has a total population of 100 (N = 100), but it is divided into 2 equal groups, then each group has 50 subjects (n = 50 for each group)

Which of the following is the correct statement regarding the size of the arterioles and veins on the fundi of the eye? A) The veins are larger than the arterioles B) The arterioles are larger than the veins C) The arterioles are half the size of the veins D) The veins and the arterioles are equal in size

A) The veins are larger than the arterioles On funduscopic exam of the eye you will see that the veins are larger in size than the arterioles.

All of the following statements are true regarding domestic abuse except: A) There is no delay in seeking medical treatment B) The pattern of injuries is inconsistent with the history reported C) Injuries are usually in the "central" area of the body instead of the extremities D) Pregnant women have a higher risk of domestic abuse

A) There is no delay in seeking medical treatment When assessing for domes- tic abuse, the most common body area that is abused is the "swim-suit" area, which is usually covered by clothing. Suspect abusive relationships when the history is inconsistent with the injury. Most victims do not seek medical attention until after several episodes of violence. Studies have shown that the incidence of battery escalates during pregnancy.

Café-au-lait spots look like tan-to-light brown stains that have irregular borders. They can be located anywhere on the body. Which of the following is a correct statement? A) They are associated with neurofibromatosis or von Recklinghausen's disease B) They may be considered as precancerous after a biopsy C) They are more common in darker-skinned children D) They are associated with Wilson's disease

A) They are associated with neurofibromatosis or von Recklinghausen's disease Café-au-lait spots are caused by an increase in melanin content, often with the presence of giant melanosomes. They have irregular borders and vary in color from light to dark brown. Neurofibromatosis causes tumors to grow in the nervous system, and these tumors commonly cause skin changes that are seen as café-au-lait spots.

Thiazide diuretics have been shown to have a beneficial effect on the bones. Hypertensive women with osteopenia or osteoporosis benefit from thiazide diuretics. What is the mechanism of action for its effect on the bones? A) Thiazide diuretics decrease calcium excretion by the kidneys and stimulate osteoclast production B) Thiazide diuretics increase both calcium and magnesium retention by the kidneys C) Thiazide diuretics increase bone mineral density (BMD) D) Thiazide diuretics influence electrolyte excretion by the kidneys

A) Thiazide diuretics decrease calcium excretion by the kidneys and stimulate osteoclast production This positive side effect of thiazides results in a decrease in calcium bone loss and an increase in the bone mineral density.

Pete J., a 20-year-old White male, is being seen for a physical exam by the nurse practitioner. He complains of pruritic macerated areas in his groin for the past 2 weeks. Which of the following is the most likely? A) Tinea cruris B) Tinea corporis C) Tinea capitis D) Tinea pedis

A) Tinea cruris Tinea cruris is a fungal infection found in the groin area. Tinea corporis involves the body, tinea capitis involves the head, and tinea pedis involves the feet.

Fetal TORCH infections can cause microcephaly, mental retardation, hepatospleno- megaly, and intrauterine growth retardation. The acronym TORCH stands for: A) Toxoplasma gondii, other infections, rubella, cytomegalovirus, and herpes B) Toxic shock syndrome, ocular infections, rubella, cytomegalovirus, and herpes zoster C) Tetanus, ophthalmic infections, roseola, cancer, and head abnormalities D) Toxins, other infections, roseola, candidiasis, and head abnormalities

A) Toxoplasmagondii, other infections, rubella, cytomegalovirus, and herpes The acronym TORCH stands for Toxoplasma gondii, other infections, rubella, cytomegalovirus, and herpes. Although several of the conditions listed in the other answer options can also cause fetal problems, they are not included in the TORCH acronym.

A 56-year-old man complains of episodes of lancinating pain that shoots up to his right cheek when he eats or drinks. He has stopped drinking cold drinks because of the pain. Which of the following is most likely? A) Trigeminal neuralgia B) Cluster headache C) Acute sinusitis D) Sinus headache

A) Trigeminal neuralgia Pain shooting up the right cheek with food or drink is seen with trigeminal neuralgia.

Which of the following drugs can increase the risk of bleeding in patients who are on anticoagulation therapy with warfarin sodium (Coumadin)? A) Trimethoprim/sulfamethoxazole (Bactrim DS) B) Carafate (Sucralfate) C) Losartan (Cozaar) D) Furosemide (Lasix)

A) Trimethoprim/sulfamethoxazole (Bactrim DS) Coumadin interacts with Bactrim and will increase the risk of bleeding; therefore, it is contraindicated.

Which of the following is an accurate description of eliciting for Murphy's sign? A) Upon deep inspiration by the patient, palpate firmly in the right upper quad- rant of the abdomen below the costovertebral angle B) Bend the patient's hips and knees at 90 degrees, then passively rotate hip exter- nally, then internally C) Ask the patient to squat, then place the stethoscope on the apical area D) Press into the abdomen deeply, then release it suddenly

A) Upon deep inspiration by the patient, palpate firmly in the right upper quad- rant of the abdomen below the costovertebral angle Murphy's sign is positive if pain occurs when performing RUQ palpation while the patient takes a deep inspiration, causing the patient to abruptly stop the deep breath due to pain. As the liver is palpated, it will descend and push up against the inflamed gallbladder and cause sharp pain (cholecystitis).

Jason, an 8-year-old with type 1 diabetes, is being seen for a 3-day history of fre- quency and nocturia. He denies flank pain and is afebrile. The urinalysis result is negative for blood and nitrites but is positive for a large amount of leukocytes and ketones. He has a trace amount of protein. Which of the following is the best test to order initially? A) Urine for culture and sensitivity B) 24-hour urine for protein and creatinine clearance C) 24-hour urine for microalbumin D) An intravenous pyelogram

A) Urine for culture and sensitivity An 8-year-old male patient with the diagnosis of diabetes has a high risk of urinary tract infections. A large amount of leukocytes in the urinalysis is abnormal and he has been having symptoms of frequency and nocturia for the past 3 days. The urine culture would be ordered because he has a high risk of infection. The urine culture and sensitivity (C&S) is the best evaluation for diagnosing a urinary tract infection.

A 30-year-old primigravida is diagnosed with a possible threatened abortion. The result of the urine pregnancy test is positive. Which of the following statements is true regarding a threatened abortion? A) Vaginal bleeding and cramping are present, but the cervix remains closed B) Vaginal bleeding and cramping are present along with a dilated cervix C) The fetus and placenta are all expelled D) The products of conception and the placenta remain inside the uterus along with a dilated cervix

A) Vaginal bleeding and cramping are present, but the cervix remains closed Threatened abortion is defined as vaginal bleeding and cramping with- out the presence of cervical dilation.

A new mother is planning on breastfeeding her newborn infant for at least 6 months. She wants to know whether she should give the infant vitamins. Which of the following vitamin supplements is recommended by the American Academy of Pediatrics (APA) latest guideline during the first few days of life? A) Vitamin D drops B) Multi vitamin drops C) It is not necessary to give breastfed infants vitamin supplements because breast milk contains enough vitamins and minerals that are necessary for the infant's growth and development D) Folic acid drops

A) Vitamin D drops According to the APA, all infants should be given Vitamin D supplementation within the first few days of life. Mothers who plan to breastfeed their infants should be taught how to use Vitamin D drops. Infant formula is supplemented with Vitamin D (and many other vitamins, minerals, and omega-3 oil), so there is no need to give it separately.

Which of the following is the best method for diagnosing candidiasis in the primary care setting? A) Wet smear B) Tzanck smear C) KOH (potassium hydroxide) smear D) Clinical findings only

A) Wet smear The wet prep smear is best used to diagnose candidiasis in the primary care setting. To perform the wet prep, the vaginal smear is placed on a slide, then a drop of saline solution is applied. Organisms viewed under the microscope can include WBCs, clue cells, bacteria, yeast cells, and trichomonads.

Which of the following groups has been recommended to be screened for thyroid disease? A) Women 50 years or older B) Adolescent females C) Elderly males D) School-age children

A) Women 50 years or older Screening for thyroid disease is recommended for women 50 years of age and older.

The best form of aerobic exercise for a patient with severe rheumatoid arthritis is: A) Yoga B) Swimming C) Riding a bicycle D) Passive range of motion

A) Yoga The American College of Rheumatology states that exercise is beneficial for everyone, including those with RA, and currently recommends 150 minutes of moderate-intensity aerobic activity each week. Safe forms of aerobic exercise, such as walking, aerobic dance, and aquatic exercise, help arthritis patients to control weight, and improve sleep, mood, and overall health.

A menopausal woman with osteopenia is attending a dietary education class. Which of the following foods are recommended? A) Yogurt and sardines B) Spinach and red meat C) Cheese and red meat D) Low-fat cheese and whole grain

A) Yogurt and sardines Postmenopausal women are advised to increase their dietary intake of calcium and vitamin D to help protect their bones from osteope- nia/osteoporosis. Foods that are high in calcium and vitamin D include yogurt and sardines.

Erysipelas is an infection of the skin most commonly caused by which of the following class of organisms? A) Streptococci B) Staphylococi C) Gram-negative bacteria D) Fungi

A)Streptococci Erysipelas is a skin infection commonly caused by group A beta- hemolytic streptococci. This infection is usually more superficial than other bacterial infections of the skin, such as cellulitis.

Extreme tenderness and involuntary guarding at McBurney's point is a significant finding for possible: A) Acute cholecystitis B) Acute appendicitis C) Acute gastroenteritis D) Acute diverticulitis

B) Acute appendicitis Symptoms of an acute abdomen, such as appendicitis, include extreme tenderness and involuntary guarding at McBurney's point.

Which of the following laboratory tests is positive in a large number of patients with systemic lupus erythematosus? A) Antinuclear antibody (ANA) B) Rheumatoid factor C) Antiparietal antibody D) Immunoglobulin

Antinuclear antibody (ANA) Screening tests for systemic lupus erythematosus include antinuclear antibody. The rheumatoid factor test is performed to diagnose rheumatoid arthritis. Antiparietal antibody testing is done to evaluate for anti- bodies against the parietal cells. The parietal cells make a substance that the body needs to absorb Vitamin B12. Immunoglobulin testing is done to assess for the amount of antibodies in the blood for a specific disease.

What cholesterol level is classified as borderline? A) 180 to 199 mg/dL B) 200 to 239 mg/dL C) > 240 mg/dL D) > 300 mg/dL

B) 200 to 239 mg/dL Total cholesterol levels should be < 200. The levels 200-239 would be the best answer.

Which of the following individuals is more likely to be affected by alpha thalassemia anemia? A) 53-year-old Greek patient B) 25-year-old Chinese patient C) 62-year-old Russian patient D) 38-year-old African American patient

B) 25-year-old Chinese patient Alpha thalassemia minor/trait or disease is more prevalent among Asians such as Chinese and Filipinos. Beta thalassemia minor/ trait or disease is more common in the countries in the Mediterranean area, such as Greece and Italy.

A 22-year-old male presents to the urgent care clinic with burns caused by a hot oil spill while frying food. He denies facial involvement, dyspnea, or weakness. During the physical exam, the nurse practitioner notices bright red skin with numerous bullae on the right arm and hand and bright red skin on the right thigh and the right lower leg. On a pain scale of 1-10, he reports the pain as 8. Based on the Rule of Nines, what is the TBSA of this patient's burns? A) 36% B) 27% C) 18% D) 9%

B) 27% This patient has a TBSA of 27% and should be referred to the emergency department as soon as possible. Check the "ABCs" and monitor the patient for shock. Do not puncture bullae.

Which of the following individuals is most likely to be at higher risk for osteoporosis? A) 70-year-old female of African ancestry who walks daily for exercise B) 42-year-old obese woman from Cuba who has been taking prednisone 10 mg daily for the last 12 years to control her severe asthma C) 55-year-old Caucasian female who is an aerobics instructor D) 4-year-old Asian female who has been on high-dose steroids for 1 week

B) 42-year-old obese woman from Cuba who has been taking prednisone 10 mg daily for the last 12 years to control her severe asthma Risk factors for osteoporosis include postmenopause, early menopause, use of chronic steroids, smoking, excessive use of alcohol, sedentary lifestyle, insufficient intake of calcium and Vitamin D in the diet, and being an Asian or Caucasian female.

A nursing home resident reports to his physician that his previous roommate was recently started on tuberculosis treatment. A Mantoux test and chest x-ray are ordered for the patient. What is the minimum size of induration considered positive for this patient? A) 3 mm B) 5 mm C) 10 mm D) 15 mm

B) 5 mm To perform the Mantoux test, a dose of 5 tuberculin units in 0.1 mL solution is injected intradermally into the forearm and read 48 to 72 hours later. Results are measured by the size of the induration, not ery- thema. Standard results of 10 mm or less are considered negative for a low- risk population. For a high-risk population, such as a recent exposure, 5 mm of induration or greater is a positive result.

All of the following children are within the parameters of normal growth and development for their age group except: A) A 2-month-old who coos and smiles B) A 14-month-old who understands complex commands C) A 20-month-old who can walk without support D) A 3-year-old who can speak in three- to four-word sentences

B) A 14-month-old who understands complex commands A 14-month-old child should developmentally be able to say "mama" and "dada," know his own name, and know at least 2-4 words. A 2-year-old is able to understand simple commands.

Native Americans view illness or disease as being caused by which of the following? A) Poor blood circulation in the body B) A punishment by the "spirits" for wrongful actions against others or for failure to follow spiritual rules C) An imbalance of the flow of energy in the body D) An imbalance of the hot and cold energy forces in the body

B) A punishment by the "spirits" for wrongful actions against others or for failure to follow spiritual rules Native Americans believe that illness is punishment by the spirits for wrong actions or from failure to follow spiritual rules. Shamans "cure" the illness by performing rituals and using herbal medicines. Medicine pouches that are tied to the patient by a string are believed to help cure the illness. Do not remove such pouches without the patient's (or parent's) permission.

When assessing a patient suspected of having vertigo, which description provided by the patient is most consistent with the diagnosis? A) A sensation of imbalance while walking B) A sensation of spinning or rotating C) A sensation of "passing out" D) A sensation of lightheadedness when changing positions from reclining to standing

B) A sensation of spinning or rotating Vertigo is defined as having a sensation of spinning or rotating

A bulla is defined as: A) A solid nodule less than 1 cm in size B) A superficial vesicle filled with serous fluid greater than 1 cm in size C) A maculopapular lesion D) A shallow ulcer

B) A superficial vesicle filled with serous fluid greater than 1 cm in size A bulla is defined as a superficial vesicle filled with serous fluid greater than 1 cm in size.

A positive psoas and obturator sign is highly suggestive of which of the following conditions? A) Ectopic pregnancy B) Acute appendicitis C) Peritonitis D) Abdominal aortic aneurysm

B) Acute appendicitis Both the psoas and obturator signs are associated with acute appendicitis. When the appendix becomes inflamed or ruptured, the blood and pus irritate the psoas and/or obturator muscles, which are both located in the retroperitoneal area. Both muscles are hip flexors and assist with hip movement.

The nurse practitioner would test the obturator and iliopsoas muscle to evaluate for: A) Cholecystitis B) Acute appendicitis C) Inguinal hernia D) Gastriculcer

B) Acute appendicitis. Signs and symptoms of an acute abdomen include invol- untary guarding, rebound tenderness, boardlike abdomen, and a positive obtu- rator and psoas sign. A positive obturator sign occurs when pain is elicted by internal rotation of the right hip from 90 degrees hip/knee flexion. The psoas sign is positive when pain occurs with passive extension of the thigh while the patient is lying on his/her side with knees extended, or when pain occurs with active flexion of his/her thigh at the hip.

A homeless 47-year-old male with a history of injection drug use (IDU) and alcohol abuse presents to the Public Health Clinic with a recent history of fever, night sweats, fatigue, and weakness. The patient has recently noticed some thin red streaks on his nailbed and red bumps on some of his fingers that hurt. During the cardiac exam, the nurse practitioner hears a Grade 3/6 murmur over the mitral area.The subcutaneous red-purple nodules are tender to palpation.The thin red lines on the nailbeds resemble subungual splinter hemorrhages. Which of the following conditions is most likely? A) Pericarditis B) Acute bacterial endocarditis C) Rheumatic fever D) Viral cardiomyopathy

B) Acute bacterial endocarditis Bacterial endocarditis is also known as infective endocarditis (IC). It is a serious bacterial infection of the heart valves and the endocardial surface. The bacteria most commonly involved are Staphylococcus and Streptococcus species. Subcutaneous red painful nodules on the finger pads are called Osler's nodes. Subungual splinter hemorrhages on the nailbeds are caused by microemboli. Janeway's lesions are caused by bleeding under the skin (usually located on the palms and the soles) and are painless red papules and macules. Other findings are conjunctival hemorrhages, petechaie, cardiac fric- tion rubs, arrhythmias, murmurs, and others. Three blood cultures obtained at separate sites 1 hour apart are used to identify the causative organism. Some of the risk factors are damaged prosthetic valves, history of rheumatic fever and injection drug use

Which of the following diagnoses is most likely in this patient? A) Rhabdomyolysis B) Acute drug-induced hepatitis C) Acute mononucleosis D) A major depressive episode

B) Acute drug-induced hepatitis Liver enzymes, such as ALT and AST, can be elevated with the use of medications, such as statin medications. High liver enzymes can cause an acute drug-induced hepatitis.

A concerned new mother reports to you that her son, who is 3 years of age, is not toilet trained yet. Which of the following is an appropriate reply? A) Recommend a referral to a pediatric urologist B) Advise the mother that her child is developing normally C) Recommend a bed-wetting alarm D) Recommend a voiding cystogram

B) Advise the mother that her child is developing normally Toilet training begins at approximately 2 years of age and may take 1-2 years to complete. Males who are not toilet trained by 3 years of age may still be developing normally.

The mother of a 4-week-old infant is concerned that her infant's eyes are crossed for a few seconds occasionally. The nurse practitioner would: A) Recommend referral to a pediatric ophthalmologist B) Advise the mother that this is a normal finding in infants up to 2 months of age C) Recommend that multivitamin supplements be given to the infant daily D) Educate her on how to patch the infant's eye every 4 hours

B) Advise the mother that this is a normal finding in infants up to 2 months of age Infants' eyes commonly cross over at times, and this is a normal finding up to 2 months of age.

A mother brings in her 6-year-old daughter to see the nurse practitioner (NP). She complains that the school nurse found a few nits on her daughter's hair. The mother states that the school has a "no nits" policy regarding head lice and the child cannot go back to school until all the nits have been removed. The child was treated with permethrin shampoo (Nix) twice about 3 months ago. During the physical exam, the NP sees a few nits that are about 2 inches away from the scalp. The child denies itchiness on her scalp. Which of the following is the best action for the NP to follow? A) Prescribe lindane (Kwell) for the child because she may have head lice that are resistant to permethrin B) Advise the mother to use a nit comb after spraying the child's hair with white distilled vinegar, waiting for 15 minutes, and then rinsing the child's hair C) Advise the mother to re-treat the girl with permethrin cream instead of shampoo D) Reassure the mother that the nits will probably drop off after a few weeks

B) Advise the mother to use a nit comb after spraying the child's hair with white distilled vinegar, waiting for 15 minutes, and then rinsing the child's hair According to the CDC, nits that are more than 1⁄4 inch from the child's scalp are usually not viable. The child also does not have an itchy scalp. One method of removal is to soak the child's head with distilled vinegar (and then rinse after), which will break down the protein of the nit casings, making it easy to comb them out of the hair.

Precocious puberty is defined as the onset of secondary sexual characteristics before the age of: A) Age 7 in girls and age 8 in boys B) Age 8 in girls and age 9 in boys C) Age 9 in girls and age 10 in boys D) Age 9 for both girls and boys

B) Age 8 in girls and age 9 in boys Precocious puberty is defined as onset of secondary sexual characteristics by the age of 8 years in girls and 9 years in boys.

Metronidazole (Flagyl) produces the disulfiram (Antabuse) effect when combined with alcoholic drinks or medicine. You would educate the patient to avoid: A) Alcoholic drinks during the time she takes the medicine B) Alcoholic drinks 1 day before, during therapy, and a few days after therapy C) Alcoholic drinks after she takes the medicine D) There is no need to avoid any food or drink.

B) Alcoholic drinks 1 day before, during, therapy and a few days after therapy Alcohol is contraindicated during the use and up until 3 days following the administration of Flagyl. Severe side effects may occur, such as tachycardia, flushing, tingling sensations, nausea, and vomiting.

Which of the following effects is seen in every woman using Depo-Provera (medroxyprogesterone injection) for more than 5 years? A) Melasma B) Amenorrhea C) Weight loss D) Headaches

B) Amenorrhea One common side effect seen in women who have been taking Depo-Provera for more than 5 years is amenorrhea. It is a progesterone hormone that causes cessation of periods.

A mother of a 7-year-old boy tells the family nurse practitioner that his teacher has complained to her of her son's frequent episodes of daydreaming. While the child is at home, the mother reports that sometimes her son seems not to hear her. The child "blanks out" for a short period of time. Which of the following is most likely? A) A partial seizure B) An absence seizure (petit mal seizure) C) A grand mal seizure D) A jacksonian seizure

B) An absence seizure (petit mal seizure) Petit mal seizure is a seizure that usually lasts less than 15 seconds, and the child appears to not be listening. The child appears to be "blacked out" or daydreaming when this occurs.

The "gold standard" for the diagnosis of active Helicobacter pylori infection of the stomach or duodenum is: A) A Helicobacter pylori titer B) An endoscopy with tissue biopsy C) An upper GI series D) A urea breath test

B) An endoscopy with tissue biopsy The "gold standard" for diagnosing Helicobacter pylori infection is to perform tissue biopsy via endoscopy.

All of the following factors are not associated with an increased risk of osteopenia in teenage girls except: A) Drinking one glass of low-fat milk daily B) Anorexia nervosa C) Participation in sports D) A normal BMI (basal metabolic index)

B) Anorexia nervosa Anorexia nervosa increases the risk for osteopenia in teenage girls due to the poor intake of foods that are high in calcium and Vitamin D.

When initially treating an adult for acute bronchitis, which of the following should the nurse practitioner be least likely to order? A) Expectorants B) Antibiotics C) Bronchodilators D) Antitussives

B) Antibiotics Acute bronchititis is normally a viral infection, so expectorants, bronchodilators, and antitussives would be prescribed for the cough. Antibiotics are not effective against viral infections.

You notice a medium-pitched harsh systolic murmur during an episodic exam. It is best heard at the right upper border of the sternum. What is most likely? A) Mitral stenosis B) Aortic stenosis C) Pulmonic stenosis D) Tricuspid regurgitation

B) Aortic stenosis Aortic stenosis is best heard at the right upper border of the sternum, radiating to the neck with a medium-pitched systolic murmur. Mitral stenosis is heard at the apex of the heart and sounds like a low-pitched diastolic rumbling murmur. It is suggested that this murmur is heard best with the bell side of the stethoscope.

A 45-year-old man fell asleep while smoking in his bedroom and started a fire. According to the patient, he refused to go to the emergency department because he had only minor burns. About 12 hours later, he presents to a walk-in urgent care center complaining of a new cough that is productive of saliva with clear mucus with small carbonaceous black particles. His brows appear singed. Which of the following is the priority when evaluating this patient? A) Perform a medical history, including prescription, OTC, and herbal medicines B) Assess the patient for respiratory distress C) Evaluate the patient for asthma and atopy D) Use the Rule of Nines to evaluate the total body surface area (TBSA)

B) Assess the patient for respiratory distress Assess the patient for respiratory distress ASAP. Follow the "ABCs" and assess the patient for any life-threatening symptoms. Smoke inhalation lung injury is the main cause of death in thermal burn victims.

A new patient is being interviewed by the nurse practitioner. The patient reports that she had a gastrectomy procedure 5 years ago to treat severe obesity. Currently, her BMI is 25 and the patient denies complications from the procedure. The nurse practitioner is aware that the patient is at higher risk for which of the following disorders? A) Folate deficiency anemia B) B12-deficiency anemia C) Iron-deficiency anemia D) Normocytic anemia

B) B12-deficiency anemia Intrinsic factor is made by the parietal cells, which are located on the fundus of the stomach. Intrinsic factor is needed to effectively absorb Vitamin B12 (dairy, meat). Because the gastric fundus is damaged in these patients, they are at higher risk of B12-deficiency anemia (MCV greater than 100).

A 30-year-old female who is sexually active complains of a large amount of milk- like vaginal discharge for several weeks. A microscopy slide reveals a large amount of squamous epithelial cells that have blurred margins. Very few white blood cells are seen. The vaginal pH is at 6.0. What is most likely? A) Trichomonas infection B) Bacterial vaginosis C) Candidal infection D) A normal finding

B) Bacterial vaginosis Bacterial vaginosis is a bacterial infection of the vagina. Signs and symptoms include copious off-white to gray discharge with foul odor without vaginal erythema or irritation. Wet prep will show positive for clue cells. When performing a wet prep, the "whiff test" will be positive for a strong "fishy" odor when vaginal discharge is mixed with one drop of KOH.

The first-line treatment consideration for managing acute alcohol withdrawal delirium includes: A) Intubation B) Benzodiazepines C) Avoidance of physical restraints to decrease agitation D) Antipsychotics

B) Benzodiazepines Acute alcohol withdrawal delirium is managed with benzodiazepines.

Which of the following classes of antihypertensive drugs should a patient be weaned off slowly to avoid the risk of severe rebound hypertension? A) Diuretics B) Beta-blockers C) ACE inhibitors D) Calcium channel blockers

B) Beta-blockers Reverse rebound hypertension can occur if beta-blockers are abruptly stopped. Recommendations include weaning off the beta-blockers when changing medications.

Chadwick's sign is characterized by: A) Softening of the cervix B) Blue coloration of the cervix and vagina C) Softening of the uterine isthmus D) Nausea and vomiting during the first trimester of pregnancy

B) Blue coloration of the cervix and vagina Chadwick's sign is defined as a bluish discoloration of the cervix and vagina. These changes are caused by the increased vascularity and congestion in the pelvic area during pregnancy.

An infant who does not have a history of reactive airway disease and allergy has both inspiratory and expiratory wheezing accompanied by fever and profuse clear nasal discharge. Which of the following is most likely? A) Tracheobronchitis B) Bronchiolitis C) Croup D) A small foreign body that is lodged on the left main bronchus

B) Bronchiolitis Symptoms of tracheobronchitis include prominent dry, non- productive cough; later, coughing up phlegm is common. Bronchiolitis is a viral infection caused by RSV that is commonly seen during the winter/spring months in infants and young children. Typical signs/symptoms include fever, inspiratory/expiratory wheezing, with clear drainage. Croup is a viral infection with the classic "barking" cough; the patient may have a runny nose, but typically no fever. When a child swallows a foreign object, choking, wheezing, and shortness of breath may occur, but no fever or clear drainage is present.

A patient who is complaining of a new onset of severe headache is being examined. The patient is instructed to lie down on the examining table while the nurse practitioner flexes his head and neck forward to his chest. The patient reacts by quickly flexing his hip and knee. What is the name of this positive finding? A) Kernig's sign B) Brudzinski's sign C) Rovsing's sign D) Drawer's sign

B) Brudzinski's sign Brudzinski's sign is positive for meningitis when a patient spontaneously flexes his/her hips and knees after lying down and having his/her head and neck flexed forward to the chest.

Which cranial nerve innervates the extraocular muscles of the eyes? A) CN II, CN III, and CN VI B) CN III, CN IV, and CN VI C) CN IV, CN V, and CN VII D) CN V, CN VI, and CN VIII

B) CN III, CN IV, and CN VI The extraocular muscles of the eyes are innervated by cranial nerves III, IV, and VI.

Which cranial nerve is being evaluated when Rinne testing is done? A) CN VII B) CN VIII C) CN IX and X D) CN XI

B) CN VIII The acoustic nerve, cranial nerve VIII, is being evaluated when the Rinne test is performed.

A 67-year-old female with a 50 pack/year history of smoking presents for a routine annual physical examination. She complains of being easily short of breath and is frequently fatigued. Physical examination reveals diminished breath sounds, hyperresonance, and hypertrophied respiratory accessory muscles. Her CBC results reveal that her hematocrit level is slightly elevated. Her pulmonary function test (PFT) results show increased total lung capacity. What is the most likely diagnosis for this patient? A) Bronchogenic carcinoma B) COPD (chronic obstructive pulmonary disease) C) Chronic bronchitis D) Congestive heart failure

B) COPD (chronic obstructive pulmonary disease) Common complaints and signs/symptoms of COPD include shortness of breath, fatigue, chronic cough, wheezing, difficulty performing tasks and speaking, pursed-lip breathing, digital clubbing, diminished breath sounds, use of accessory muscles, and polycythemia due to chronic hypoxia. Pulmonary function tests/ spirometry include forced expira- tory volume, forced vital capacity, and FEV1/FVC ratios.

All of the following describe normal behavior for a 3-year-old child except: A) Speaks in 3- to 4-word sentences that are understood by most strangers B) Can draw a cross C) Can draw a circle D) Can ride a tricycle

B) Can draw a cross Developmental stages in children include the following: 1 year: walk; 2 years: walks up steps with the same foot; 3 years: pedals a tricycle and copies a circle; 4 years: rides a bicycle and copies a cross and draws a person with 2 parts

The mother of a 13-year-old male with Down syndrome is in the family nurse practitioner's office and wants a sports physical done for her son. She reports that he wants to join the football team in his school. You would tell the mother that her son: A) Can play a regular football game as long as he wears maximum protective football gear B) Cannot play some contact sports because of an increased risk of cervical spine injury C) Can play certain contact sports after he has been checked for cervical instability D) None of the above

B) Cannot play some contact sports because of an increased risk of cervical spine injury Children with the diagnosis of Down syndrome are at higher risk for atlantoaxial instability, congenital heart defects, and early onset of Alzheimer's disease. Therefore, due to the risk of cervical spine injury, some contact sports are not advised

Fitz-Hugh-Curtis syndrome is associated with which following infection? A) Syphilis B) Chlamydia trachomatis C) Herpes genitalis D) Lymphogranuloma venereum

B) Chlamydia trachomatis Fitz-Hugh-Curtis is a complication of having pelvic inflammatory disease that was caused by a vaginal infection, such as gonorrhea or Chlamydia trachomatis. This causes inflammation and infection in the pelvic cavity. Left untreated, this infection can cause adhesions that stretch from the peritoneum to the liver.

411. Which of the following is the most common cause of nongonococcal urethritis? A) Escherichia coli B) Chlamydia trachomatis C) Neisseria gonorrhoeae D) Mycoplasma

B) Chlamydia trachomatis The most common cause of nongonococcal urethritis is Chlamydia trachomatis.

Patients who are diagnosed with gonorrhea should also be treated for which of the following infections? A) Chancroid B) Chlamydia trachomatis C) Herpes genitalis D) PID (pelvic inflammatory disease)

B) Chlamydia trachomatis When diagnosed with gonorrhea, the patient should also be treated for Chlamydia trachomatis.

What is the most common cause of left ventricular hypertrophy in the United States? A) Chronic atrial fibrillation B) Chronic hypertension C) Mitral valve prolapse D) Pulmonary hypertension

B) Chronic hypertension The most common cause of LVH is chronic hypertension.

The heart sound S2 is caused by: A) Closure of the atrioventricular valves B) Closure of the semilunar valves C) Opening of the atrioventricular valves D) Opening of the semilunar valves

B) Closure of the semilunar valves The heart sound S2 is caused by closure of the semilunar valves.

A woman who is in the third trimester of pregnancy presents to the nurse practitioner for a physical exam. During the physical exam, the nurse practitioner finds all of the following cardiac changes associated with pregnancy except: A) Systolic ejection murmur B) Diastolic murmur C) Displaced apical impulse D) Louder S1 and S2

B) Diastolic murmur Diastolic murmurs are more likely to be pathologic. The heart is displaced in a more transverse position that is lateral to the midclavicular line. The systolic ejection murmur is due to increased stroke volume caused by increased cardiac output and higher basal heart rate.

A 40-year-old male complains to the nurse practitioner of severe stabbing pains behind his left eye for the past 2 days. It is accompanied by some nasal congestion and rhinorrhea, which is clear in color. The patient denies pharyngitis and fever. Which of the following conditions is most likely? A) Migraine headache with aura B) Cluster headache C) Tic douloureux D) Cranial neuralgia

B) Cluster headache Signs/symptoms of cluster headaches include severe stabbing pain behind the eyes, with nasal congestion and rhinorrhea. Migraine head- aches with aura include visual changes, such as blind spots or flashing lights that appear before the onset of the headache. Trigeminal neuralgia (tic douloureux) is a unilateral headache from compression or inflammation of the trigeminal nerve (cranial nerve 5).

A physician is referring one of his patients to a nurse practitioner. What type of relationship will exist between the physician and nurse practitioner? A) Consultative B) Collaborative C) Professional D) Advocate

B) Collaborative A collaborative relationship exists when a health caregiver refers a patient to others (physicians, specialists, physical therapy, etc.) to help with patient treatment and management. Consult reports and progress reports are sent to the primary caregiver to report the patient's progress. Consultative relationships are informal, such as talking to a colleague about a patient treatment

A nurse practitioner is writing a referral for a middle-aged diabetic who has an A1c of 8.5% despite being on 3 antidiabetic medications. She is referring the patient to an endocrinologist. What type of relationship exists between the nurse practitioner and the endocrinologist? A) Consultative relationship B) Collaborative relationship C) Referral relationship D) Formal relationship

B) Collaborative relationship A collaborative relationship is a formal process of sharing responsibility for treating a patient together (sharing progress reports); for example, referring a patient to a specialist or for rehabilitation.

A 68-year-old woman with hypertension and diabetes is seen by the nurse practitioner for a dry cough that worsens at night when she lies in bed. She has shortness of breath, which worsens when she exerts herself. The patient's pulse rate is 90/min and regular. The patient has gained 6 lbs over the past 2 months. She is on a nitro- glycerine patch and furosemide daily. The best explanation for her symptoms is: A) Kidney failure B) Congestive heart failure C) ACE inhibitor-induced coughing D) Thyroid disease

B) Congestive heart failure Signs and symptoms of congestive heart failure (CHF) include dyspnea on exertion, edema, fatigue, hemoptysis, cough, orthopnea, hypertension, and nocturnal dyspnea.

Which of the following would you recommend to this 55-year-old patient? A) Start an exercise program by starting with walking instead of jogging B) Consult with a cardiologist for further evaluation C) Consult with a gastroenterologist to rule out acute cholecystitis D) Take ibuprofen (Advil) 600 mg for pain every 4 to 6 hours PRN

B) Consult with a cardiologist for further evaluation Physical assessment and examination by a cardiologist are advised to rule out cardiac problems and deter- mine the safety of physical exercise.

An nurse practitioner, who is a recent graduate, is asking an experienced nurse practitioner's opinion about managing a patient who has multiple health problems. What type of relationship exists between the nurse practitioners? A) Collaborative relationship B) Consultative relationship C) Referral relationship D) Formal relationship

B) Consultative relationship A consultative relationship is an informal process between two or more providers who exchange information about a patient occasionally.

Which of the following laboratory tests is used in primary care to evaluate renal function? A) Electrolyte panel B) Creatinine C) Alkaline phosphatase D) Blood urea nitrogen (BUN) to creatinine ratio

B) Creatinine Serum creatinine is measured to evaluate renal function. Creatinine is the end product of creatine metabolism. Creatinine clearance is not affected by fluid or dietary intake of meat.

Which of the following is considered an objective finding in patients who have a case of suppurative otitis media? A) Erythema of the tympanic membrane B) Decreased mobility of the tympanic membrane as measured by tympanogram C) Displacement of the light reflex D) Bulging of the tympanic membrane

B) Decreased mobility of the tympanic membrane as measured by tympanogram Fluid behind the ear drum will decrease the mobility of the TM when measured by a tympanogram. When perforation occurs, discharge will flow through the pars tensa portion of the ear drum.

Heberden's nodes are commonly found in which of the following diseases? A) Rheumatoid arthritis B) Degenerative joint disease C) Psoriatic arthritis D) Septic arthritis

B) Degenerative joint disease Heberden's nodes are bony nodules on the distal interphalangeal joints, commonly seen in degenerative joint disease.

There is a higher risk of balanitis in which of the following conditions? A) Renal insufficiency B) Diabetes mellitus C) Graves' disease D) Asthma

B) Diabetes mellitus Balanitis is an yeast infection of the glans of the penis. Men who are not circumcised and who have diabetes mellitus are at higher risk for developing balanitis.

A 56-year-old mechanic is brought to your office complaining of heavy pressure in the substernal area of his chest that is radiating to his jaw. The pain began while he was lifting up a tire. He now appears pale and is diaphoretic. His blood pressure is 100/60 mm Hg, and his pulse rate 50. What is the most appropriate action? A) Perform a 12-lead EKG B) Dial 9-1-1 C) Administer a morphine injection for pain D) Observe the patient in the office

B) Dial 9-1-1 Heavy chest pressure in the substernal area, radiating to the jaw, diaphoresis, low BP, and bradycardia are signs of cardiac distress and 9-1-1 should be called immediately for transfer to the emergency department.

A newborn's mother is discovered to be HBsAg (hepatitis B surface antigen) positive. Which of the following would you recommend for this infant? A) Give the baby hepatitis B immunoglobulin B) Give the baby both hepatitis B vaccine and hepatitis B immunoglobulin C) Give the baby hepatitis B vaccine only D) Send the baby home because he is not infected

B) Give the baby both hepatitis B vaccine and hepatitis B immunoglobulin For a mother who tests positive for HBsAg, the newborn infant should be given hepatitis B vaccine and hepatitis B immunoglobulin for protection.

When starting an elderly patient on a new prescription of levothyroxine (Synthroid), the nurse practitioner should keep in mind that the rationale for starting an elderly patient on a lower dose is which of the following? A) Due to its central nervous system effects B) Due to its cardiac effects C) Due to its renal effects D) Due to its hepatic effects

B) Due to its cardiac effects Levothyroxine (Synthroid) should be started on the lowest dose in elderly patients due to the severe side effects that can occur. Side effects include palpitations, tachycardia, anxiety, irritability, elevated BP, flushing, and insomnia.

Which of the following findings is most likely in young primigravidas with pregnancy-induced hypertension? A) Abdominal cramping and constipation B) Edema of the face and the upper extremities C) Shortness of breath D) Dysuria and frequency

B) Edema of the face and the upper extremities Common signs and symptoms of pregnancy-induced hypertension include edema of the face and the upper extremities, weight gain, blurred vision, elevated BP, proteinuria, and headaches.

A 53-year-old crossing guard complains of twisting his right knee while working that morning. The knee is swollen and tender to palpation. The nurse practitioner diagnoses a Grade II sprain. The initial treatment plan includes which of the following? A) Application of cold packs the first 24 hours followed by applications of low heat at bedtime B) Elevation of the affected limb and intermittent applications of cold packs for the next 48 hours C) Rechecking the knee in 24 hours and isometric exercises D) The application of an Ace bandage to the affected kne

B) Elevation of the affected limb and intermittent applications of cold packs for the next 48 hours Elevation of the injured knee above the heart will reduce the amout of swelling that can occur. Use of ice packs immediately after the injury is most effective and will reduce swelling in the tissue. Ice the affected area for 15 minutes at a time intermittently to prevent frostbite and further damage to tissue. Allowing 30-45 minutes between icings of the limb is recommended.

A 21-year-old female who is complaining of random palpitations is diagnosed with mitral valve prolapse (MVP). Her echocardiogram (EKG) reveals redundant and thickened leaflets. You note a Grade III/VI systolic murmur with an ejection click during physical examination. You would recommend: A) Endocarditis prophylaxis for most dental and urologic procedures B) Endocarditis prophylaxis is not necessary C) Lifetime anticoagulation therapy with warfarin sodium D) Endocarditis prophylaxis for dental procedures only

B) Endocarditis prophylaxis is not necessary Prophylaxis treatment for endocarditis is no longer recommended for MVP.

124. Which of the following viral infections is associated with occasional abnormal forms of lymphocytes during an acute infection? A) Cytomegalovirus (CMV) B) Epstein-Barr virus (EBV) C) Human papilloma virus (HPV) D) Coxsackie virus

B) Epstein-Barr virus (EBV) A test for an acute infection of the Epstein-Barr virus will show abnormal forms of lymphocytes.

Duvall and Miller used developmental theory to describe the family. Which of the following statements is true? A) Each family is developmentally unique in comparison to other families B) Families demonstrate common forms of membership across developmental stages C) Families complete each developmental task separately D) Families change over time because of the influence of environmental factors

B) Families demonstrate common forms of membership across developmental stages Families progress through similar developmental stages and experience similar changes through this process.

A newborn infant who is small for gestational age is noted to have shortened palpebral fissures and microcephaly with a small jaw. This infant is most likely to be diagnosed with: A) Down syndrome B) Fetal alcohol syndrome C) Growth retardation D) Hydrocephalus

B) Fetal alcohol syndrome Classic symptoms of fetal alcohol syndrome include small palpebral fissures and microcephaly with a small jaw.

What is the best procedure for evaluating a corneal abrasion? A) Tonometry B) Fluorescein stain C) Visual field test D) Funduscopy

B) Fluorescein stain Fluorescein stain is an eye stain used to detect abrasions or foreign objects in the cornea of the eye. Orange dye (fluorescein) is used to stain the eye and a blue light is used to detect/visualize any foreign bodies or abrasions in the eye. Visual field test assesses vision. Tonometry measures the pressure inside the eye. The funduscopic exam is performed with an ophthalmoscope to visualize the inside of the eye.

You would advise a patient who is on a MAOI (monoamine oxidase inhibitor) prescription to avoid taking one of the following drugs because of increased potential for a serious reaction. Which of the following is this drug? A) Alprazolam (Xanax) B) Fluoxetine (Prozac) C) Erythromycin (E-mycin) D) Amoxicillin (Amoxil)

B) Fluoxetine (Prozac) Patients taking MAOIs should not also take SSRIs (Prozac), due to the possibility of developing serotonin syndrome. This causes excessive serotonin release in the brain, which will cause symptoms of severe anx- iety, restlessness, confusion, and muscle twitching. If not stopped, the patient may develop muscle contractions, renal failure, respiratory failure, coma, and even death.

A high school teacher complains of a dry cough for the past 6 weeks. It worsens when he is supine. He has episodes of heartburn, which he self-treats with an over- the-counter (OTC) antacid. He chews mints for his "bad breath." Which of the fol- lowing is a possible cause for this patient's cough? A) Asthma B) Gastroesophageal reflux C) Pneumonia D) Chronic postnasal drip

B) Gastroesophageal reflux GERD is a condition in which food comes up from the stomach/esophagus due to a weak sphincter. It usually worsens with lying down and can cause a cough and esophageal irritation if not treated.

Podagra is associated with which of the following? A) Rheumatoid arthritis B) Gout C) Osteoarthritis D) Septic arthritis

B) Gout Podagra is pain in the joint of the great toe due to the accumulation of uric acid and salts in the joint.

An 8-year-old is seen as a walk-in appointment by the nurse practitioner. The mother reports that her child has been febrile for 2 days and is not eating well due to painful sores inside the child's mouth. The child's temperature is 101 degrees Fahrenheit, the pulse is 88 beats/minute, and the respirations are at 14 breaths per minute. During the physical examination, the nurse practitioner notices several small blisters and shallow ulcers on the child's pharynx and the oral mucosa. The child has small round red rashes on both palms and soles. Which of the following conditions is most likely? A) Herpes simplex infection B) Hand, foot, and mouth disease (HFMD) C) Varicella infection D) Secondary syphilis infection

B) Hand, foot, and mouth disease (HFMD) Hand, foot, and mouth disease (HFMD) is caused by the coxsackie virus A16. The virus is found in the saliva, sputum, nasal mucus, feces, and blister fluid. It is transmitted through direct contact of the secretion or in fomites (e.g., preschool toys). Treatment is symptomatic.

During a well child visit, a new father wants to know if he can give fresh whole milk to his 6-month-old son. The nurse practitioner would recommend that: A) He can start giving whole milk but not skim milk by 6 months of age B) He should not give whole milk to his son until the boy is at least 12 months of age C) He can give whole milk to his son at anytime D) He should not give whole milk to his son without diluting it with water

B) He should not give whole milk to his son until the boy is at least 12 months of age The American Academy of Pediatrics does not recommend offering whole milk to children younger than 12 months of age.

A 63-year-old patient with a 10-year history of poorly controlled hypertension presents with a cluster of physical exam findings. Which of the following indicate target organ damage commonly seen in hypertensive patients? A) Pedal edema, hepatomegaly, and enlarged kidneys B) Hepatomegaly, AV nicking, bibasilar crackles C) Renal infection, S3, neuromuscular abnormalities D) Glaucoma, jugular vein atrophy, heart failure

B) Hepatomegaly, AV nicking, bibasilar crackles With long-term, uncontrolled hypertension, organ damage may occur. Organs commonly affected include: brain (stroke), eyes (retinopathy, AV nicking, bleeding, blindness), heart (heart dis- ease, left ventricular hypertrophy, MI, and/or CHF), and kidneys (renal failure, proteinuria).

The mother of a 16-year-old boy is concerned that her son is not developing normally. On physical exam, the patient is noted to have small testes with no pubic or facial hair. What is the most appropriate statement to the mother? A) Her son is developing normally B) Her son's physical development is delayed and should be evaluated by a pediatric endocrinologist C) Her son should be rechecked in 3 months; if he still does not have secondary sexual characteristics, a thorough hormonal workup should be initiated D) Her son's physiological development is slower than normal but is within the lower limit of normal for his age group

B) Her son's physical development is delayed and should be evaluated by a pediatric endocrinologist A 16-year-old male without secondary sexual characteristics should be referred to an endocrinologist. If there is no testicular development by 14 years of age, referral to an endocrinologist is recommended.

A 35-year-old primigravida who is at 28 weeks of gestation is expecting twins. What would you would expect her alpha fetoprotein (AFP) values to be? A) Normal B) Higher than normal C) Lower than normal D) None of the above

B) Higher than normal Alpha fetoprotein is produced in the fetal and maternal liver. Higher levels of alpha fetoprotein are commonly seen in multiple gestations due to the growing fetuses and enlargement of the livers.

Mrs.Brands complains of dizziness when she moves her head. You suspect benign paroxysmal positional vertigo. The diagnosis is supported by the presence of: A) Tinnitus B) Horizontal nystagmus with rapid head movement C) New onset of hearing loss D) Duration of greater than 2 years

B) Horizontal nystagmus with rapid head movement Symptoms of benign paroxysmal positional vertigo include horizontal nystagmus with rapid head movement. By performing the Dix-Hallpike maneuver and/or the roll test, the symptoms of vertigo will appear.

You have diagnosed Tom J., a 30-year-old male, with contact dermatitis on the left side of the face secondary to poison ivy. You would recommend: A) Washing with antibacterial soap BID to reduce risk of secondary bacterial infection until it is healed B) Hydrocortisone cream 1% BID until it is healed C) Clotrimazole (Lotrimin) cream BID for 2 weeks D) Halcinonide (Halog) 1% ointment BID for 2 weeks

B) Hydrocortisone cream 1% BID until it is healed Treatment for contact dermatitis includes using a steroid cream BID until healed.

Which of the following conditions is the most common cause of sudden death among athletes? A) Brain aneurysm B) Hypertrophic cardiomyopathy C) Left ventricular hypertrophy D) Aortic stenosis

B) Hypertrophic cardiomyopathy Hypertrophic cardiomyopathy is the most common cause of sudden death in athletes.

You are examining a patient who has just been diagnosed with Bell's palsy. Bell's palsy is characterized by all of the following except: A) Drooling B) Inability to swallow C) Inability to close the eye on the affected side D) Drooping of the corner of the mouth on the affected side

B) Inability to swallow Bell's palsy affects cranial nerve VII, which affects the ability to smile, close the eye on the affected side; drooping and drooling on the affected side can also occur. Swallowing is not affected by the facial nerve.

About one-third of children in the United States are considered obese. Which of the following methods are appropriate interventions for obese school-aged children? A) Severe restriction of dietary carbohydrates B) Increase physical activity and outdoor play C) Prescribe appetite suppressants D) Over-the-counter herbal weight-loss pills

B) Increase physical activity and outdoor play Lifestyle changes such as increasing physical activity and spending more time outdoors are appropriate recommendations for children. Consider referral to a registered dietician for dietary recommendations and counseling. Participation in an exercise program designed for children (if available) is also appropriate. Severe caloric and/or carbohydrate restriction is not recommended for this age group.

A positive straight-leg raising test is indicative of which of the following? A) Myasthenia gravis B) Inflammation of the sciatic nerve/herniated disc C) Multiple sclerosis D) Parkinson's disease

B) Inflammation of the sciatic nerve/herniated disc To perform the straight- leg test, have the patient lie supine on an exam table. Lift the patient's leg toward the head while the knee is straight. If the patient experiences sciatic pain when the straight leg is at an angle of between 30 and 70 degrees, then the test is positive and a herniated disc is likely to be the cause of the pain. The straight-leg test should be done on the pain-free side first to find out which range of movement is normal and to enable the patient to distinguish between "normal" stretching of muscles and a different sort of pain.

Mr. R. J. is a 40-year-old asthmatic male with hypertension. For the past 6 months, he has been following a low-fat, low-sodium diet and walking 3 times a week. His BP readings from the past 2 visits were 160/95 and 170/ 100. On this visit, it is 160/90. What is the most appropriate action for the nurse practitioner to follow at this visit? A) Continue the lifestyle modifications and recheck his blood pressure again in 4 weeks B) Initiate a prescription of hydrochlorothiazide 12.5 mg PO daily C) Initiate a prescription of atenolol (Tenormin) 25 mg PO daily D) Refer the patient to a cardiologist for a stress EKG

B) Initiate a prescription of hydrochlorathiazide 12.5 mg PO daily The patient would be started on HCTZ 12.5 mg PO daily for his high BP. He has been trying lifestyle modifications for 6 months with diet changes and exercise.

Mrs. Nottam, who has a BMI of 29, has a 20-year history of primary hypertension. She has been on hydrochlorothiazide 25 mg PO daily with excellent results. On this visit, she is complaining of feeling thirsty all the time even though she drinks more than 10 glasses of water per day. She reports to the nurse practitioner that she has been having this problem for about 6 months. Upon reading the chart, the nurse practitioner notes that the last two fasting blood glucose levels have been 140 mg/dL and 168 mg/dL. A random blood glucose is at 210 mg/dL. Which of the following is the best treatment plan to follow at this visit? A) Order another random blood sugar test in 2 weeks B) Initiate a prescription of metformin (Glucophage) 500 mg PO BID C) Order a 3-hour glucose tolerance test D) Order a HgbA1c leve

B) Initiate a prescription of metformin (Glucophage) 500 mg PO BID Initiating metformin is recommended because she has met the criteria for diagnosis of diabetes. Criteria for diagnosing type 2 diabetes: fasting blood glucose 126 mg/dL or higher on 2 separate occasions, 2-hour blood glucose 200 mg/dL or higher during OCT with 75 g glucose load, symptoms of diabetes (polyuria, polydipsia, polyphagia) plus random blood glucose greater than 200 mg/dL.

What does a positive posterior drawer sign in a 10-year-old soccer player signify? A) An abnormal knee B) Instability of the knee C) A large amount of swelling on the knee D) An injury of the meniscus

B) Instability of the knee The drawer sign is performed on the knee or ankle to assess for knee instability. The affected knee will have more laxity when compared to the unaffected knee.

A 75-year-old woman presents complaining of a soft lump on her abdomen that is located on the periumbilical area. She tells the nurse practitioner that she does not know how long she has had the lump or whether it has changed in size or shape. She denies abdominal pain, problems with defecation, loss of appetite, weight loss, or trauma. When performing an abdominal exam, what is the best method to differentiate between an abdominal wall mass and an intra-abdominal mass? A) Palpate the abdominal wall while the patient is relaxed B) Instruct the patient to lift her head off the table while tensing her abdominal muscles to visualize any masses and then palpate the abdominal wall C) Instruct the patient to lie still for few seconds while you palpate the abdominal wall D) Palpate the abdomen deeply, then release the palpating hand quickly

B) Instruct the patient to lift her head off the table while tensing her abdominal muscles to visualize any masses and then palpate the abdominal wall An abdominal wall mass will become more prominent when the abdominal wall muscles are tense. If it is an intra-abdominal mass, it will be pressed down by the muscles and will become less obvious or disappear. Some of the most common abdominal wall masses are hernias (epigastric, umbilical, incisional). This patient had a periumbilical hernia (soft lump on her abdomen that is located on the periumbilical area that is painless).

Which of the following is contraindicated in the care of pregnant women with placenta previa? A) Echocardiogram B) Intravaginal ultrasound C) Abdominal ultrasound D) Pelvic ultrasound

B) Intravaginal ultrasound No type of vaginal exam should be performed in patients diagnosed with placenta previa. Intravaginal ultrasound and pelvic exams are contraindicated

Which of the following would be appropriate initial management of a second- degree burn? A) Irrigate with hydrogen peroxide and apply Silvadene cream BID B) Irrigate with normal saline and apply Silvadene cream BID C) Irrigate with tap water and apply Neosporin ointment BID D) Unroof all intact blisters and apply antibiotic ointment BID

B) Irrigate with normal saline and apply Silvadene cream BID Burns should be cleansed with saline solution and Silvadene cream applied BID to the site. Hydrogen peroxide is no longer recommended. Intact blisters should not be unroofed.

Which of the following is used to screen for color blindness in a 7-year-old boy? A) Snellen chart B) Ishihara chart C) Cover/uncover test D) Red reflex

B) Ishihara chart The Ishihara chart is used for screening for color blindness.

All of the following are false statements about atopic dermatitis except: A) Contact with cold objects may exacerbate the condition B) It does not have a linear distribution C) It is associated with bullae D) The lesions have vesicles that are full of serous exudate

B) It does not have a linear distribution Atopic dermatitis is a skin condition in which the lesions occur in a linear fashion. They may have many different stages, including erythematous papules and vesicles, with weeping, drainage, and/or crusting. Lesions are commonly found on the scalp, face, forearms, wrists, elbows, and backs of the knees. Commonly pruritic. Also known as eczema.

A 72-year-old woman has been on hydrochlorothiazide 12.5 mg for many years to control her Stage II hypertension. Her blood pressure (BP) at this visit is 168/96. She is currently complaining of pain on her right hip and on both knees. She has increased her dose of ibuprofen (Motrin) from 400 mg 3 times day (TID) to 800 mg TID. She is still in pain and would like something stronger. Which of the following statements is the best explanation of the effects of ibuprofen (Motrin) on her disease? A) It increases the chances of adverse effects to her health B) It inhibits the effect of renal prostaglandins and blunts the effectiveness of the diuretic C) It prolongs the therapeutic effects of hydrochlorothiazide and other diuretics D) None of the statements are true

B) It inhibits the effect of renal prostaglandins and blunts the effectiveness of the diuretic. NSAIDs and ASA inhibit the vasodilatory effects of prostaglandins, which predisposes the kidney to ischemia. NSAIDs and diuretics can cause acute prerenal failure by decreasing renal blood flow.

Your patient of 10 years, Mrs. Leman, is concerned about her most recent diagnosis. She was told by her dermatologist that she has an advanced case of actinic keratosis. Which of the following is the best explanation for this patient? A) It is a benign condition B) It is a precancerous lesion and should be followed up with her dermatologist C) Apply hydrocortisone cream 1% BID for 2 weeks and most of it will go away D) It is important for her to follow up with an oncologist

B) It is a precancerous lesion and should be followed up with her dermatologist Actinic keratosis is a small, raised skin lesion from having been in the sun for a long period of time. Some actinic keratoses may develop into skin cancer; therefore, further evaluation is needed to determine if removal is required.

All of the following statements about phototherapy are correct except: A) Light from the blue to white spectrum is used B) It is not always necessary to use a shield for the infant's eyes C) Unconjugated bilirubin in the skin is converted to a water-soluble nontoxic substance that is excreted in the bile D) The infant's eyes should be shielded

B) It is not always necessary to use a shield for the infant's eyes When using phototherapy, the eyes should always be protected by using a shield or goggles to prevent damage to the eyes.

What is the median? A) It is the number that occurs the most frequently B) It is the middle number in a group of numbers C) It is the average number in a group of numbers D) It a measure of central tendency

B) It is the middle number in a group of numbers The median is the middle score of a group of numbers. For example, for this group of numbers (2, 3, 6, 7, 7, 8, 10), the number "7" is the median value.

The Somogyi effect is characterized by which of the following? A) It is a complication of high levels of growth hormone B) It is the physiologic spike of serum blood glucose in the early morning C) It is characterized by high fasting blood glucose in the morning D) It is a rare phenomenon that only occurs in type 1 diabetic patient

B) It is the physiologic spike of serum blood glucose in the early morning The Somogyi phenomenon is when nocturnal hypoglycemia (2-3 a.m.) stimulates the liver to produce glucagon to raise the blood sugar. The fasting blood glucose levels will be elevated from this glucagon production.

A pelvic exam on a woman who is 12 weeks pregnant would reveal that her uterus is located at which of the following areas? A) Between the umbilicus and the suprapubic bone B) Just rising above the suprapubic bone C) Between the suprapubic bone and the xiphoid process D) Between the umbilicus and the xiphoid proces

B) Just rising above the suprapubic bone At 12 weeks gestation, the uterus measures approximately the size of a grapefruit, which would be felt just above the suprapubic bone on bimanual exam.

What is the primary carbohydrate found in breast milk and commercial infant formulas? A) Fructose B) Lactose C) Glucose D) Sucrose

B) Lactose Lactose is the primary carbohydrate found in breast milk and formula

The sentinel nodes (Virchow's nodes) are found at the: A) Right axillary area B) Left supraclavicular area C) Posterior cervical chain D) Submandibular chain

B) Left supraclavicular area The sentinel nodes are found at the supraclavicular area of the chest. They are the first lymph nodes that a cancer lesion will drain into. Therefore, when cancer is diagnosed, these nodes are biopsied to see if the cancer has spread into the lymph system.

A 67-year-old retired clerk presents with complaints of shortness of breath and weight gain over a 2-week period. A nonproductive cough accompanies her symptoms. The lung exam is positive for fine crackles in the lower lobes with no wheezing. The exam is positive for egophony. Which of the following conditions is most likely? A) Acute exacerbation of asthma B) Left-heart failure C) Right-heart failure D) Chronic obstructive pulmonary disease

B) Left-heart failure Signs and symptoms of left-heart failure include tachypnea, labored breathing, and rales or crackles in the lower bases of the lungs, which can develop into pulmonary edema.

A kindergarten teacher is diagnosed with acute pharyngitis. On exam, the throat is a bright red color with no tonsillar exudate, and clear mucus is seen on the lower nasal turbinates. The result of her urinalysis shows a large amount of white blood cells and is positive for nitrites. The patient has a sulfa allergy and thinks she is also allergic to penicillins. Which of the following is the best treatment choice? A) Amoxicillin/clavulanic acid (Augmentin) 500 mg PO BID B) Levoquinolone (Levaquin) 250 mg PO daily C) Trimethoprim sulfamethoxazole (Bactrim DS) 1 tablet PO BID D) Clarithromycin (Biaxin) 500 mg PO BID

B) Levoquinolone (Levaquin) Pharyngitis and urinary tract infection are both covered by using Levaquin, which is a quinolone. Augmentin and Bactrim could not be used due to her allergies.

While performing a Pap smear on a postmenopausal patient, several areas of flat white skin lesions that are irregularly shaped are found on the patient's labia. The patient reports that the lesions are extremely itchy and have been present for several years without much change. Which condition is best described? A) Chronic scabies infection B) Lichen sclerosus C) Chronic candidal vaginitis D) A physiologic variant found in some older women

B) Lichen sclerosus Lichen sclerosus is a disease of the skin, in which white spots appear on the skin and change over time. It is most commonly seen in the genital and rectal areas, but can appear in other areas. The spots are usually shiny and smooth and can eventually spread into patches. The skin appears thin and crinkled. Then the skin tears easily, and bright red or purple bruises are common. Sometimes, the skin becomes scarred. If the disease is a mild case, there may be no symptoms

An 80-year-old woman complains about her "thin" and dry skin. Which of the following is the best explanation for her complaint? A) Genetic predisposition B) Loss of subcutaneous fat and lower collagen content C) Loss of sebaceous glands D) Damage from severe sun exposure

B) Loss of subcutaneous fat and lower collagen content Thinning of the skin in older females is due to the loss of subcutaneous fat and lower collagen content

Which of the following is the most common cause of cancer deaths for women in the United States? A) Breast cancer B) Lung cancer C) Colon cancer D) Heart disease

B) Lung cancer Lung cancer is the most common cause of cancer deaths in women as well as men.

The gold-standard test for visualizing a torn meniscus or joint abnormalities is the: A) Computed tomography (CT) scan B) Magnetic resonance imaging (MRI) scan C) X-ray with special views of the affected knee D) Lachman's maneuver

B) Magnetic resonance imaging (MRI) scan MRI provides good visualization of soft tissues of the body (most cancers, brain, cartilage, muscles, inflammation, etc.). It is best used in tissues with high water content. Patients with metal implants such as cochlear implants and cardiac pacemakers should be carefully screened. The MRI does not use radiation, but uses strong magnetic and radio waves to visualize body structures.

A major risk factor for a Down syndrome infant is: A) Maternal age younger than 16 years B) Maternal age older than 35 years C) A positive family history of Down syndrome D) A positive family history of genetic disease

B) Maternal age older than 35 years Maternal age greater than 35 years during pregnancy is a risk factor for infant Down syndrome.

Koplik's spots are associated with: A) Poxvirus infections B) Measles C) Kawasaki's disease D) Reye's syndrome

B) Measles Signs and symptoms of the measles include fever over 101°F, coryza, cough, conjunctivitis, rash, and Koplik's spots on buccal mucosa.

An elderly female patient who is a retired nurse has recently been discharged from the hospital. A few days later, she started having random and recurrent episodes of dizziness, but denies passing out. The patient describes it as the sensation of the room spinning or moving, which is worsened by sudden head movement. During the episodes, she gets very nauseated and sometimes vomits. The patient reports that she was given IV antibiotics and one of them was tobramycin. Which of the following medications is helpful in treating her symptom of dizziness? A) Scopolamine patch (Transderm Scop) B) Meclizine (Antivert) C) Dimenhydrinate (Dramamine) D) Duloxetine (Cymbalta)

B) Meclizine (Antivert) The case is describing vertigo. Meclizine (Antivert) 12.5 mg to 50 mg TID to QID is used to treat vertigo. Do not forget to also treat nausea, which can be severe. Anti-nausea medicine like dimenhydrinate (Dramamine) or prochlorperazine (Compazine) are effective. Advise the patient that these drugs can cause drowsiness.

A 22-year-old male is brought to an urgent care center by his anxious mother. She reports that her son returned from a camping trip 2 days ago with a high fever and bad headache. Apparently, the patient had complained to her of a painful and stiff neck along with nausea shortly after he arrived. According to the mother, her son started breaking out in a rash the day before, some of which is turning a dark red to purple color. During the physical exam, the nurse practitioner evaluates the patient for Kernig's sign, which is positive. Which of the following conditions is most likely? A) Stevens-Johnson syndrome B) Meningococcemia C) Rocky Mountain spotted fever D) Erythema multiforme

B) Meningococcemia Kernig's maneuver is performed by flexing both hips and legs and having the patient straighten the legs against resistance, testing for men- ingitis. A positive test indicates meningitis. Other characteristics of meningitis include high fever, headache, stiff neck, and nausea/vomiting.

A 55-year-old woman who is on a prescription of clindamycin for a dental infection presents to the nurse practitioner with complaints of watery diarrhea for the past 4 days. She complains of abdominal cramping and bloating with diarrheal stools up to 10 times a day. She denies seeing blood or pus in her stool. There is no history of recent travel. The patient has been taking over-the-counter medicine with no relief. The nurse practitioner suspects that the patient has a mild case of Clostridium difficile colitis. Which of the following antibiotics is indicated for this infection? A) Ciprofloxacin (Cipro) 400 mg PO BID x 7 days B) Metronidazole (Flagyl) 500 mg PO TID x 10 days C) Levofloxacin (Levaquin) 750 mg PO daily x 7 days D) Trimethoprim-sulfamethoxazole (Bactrim DS) 1 tablet PO BID x 10 days

B) Metronidazole (Flagyl) 500 mg PO TID x 10 days First-line treatment for a mild case of Clostridium difficile colitis is metronidazole (Flagyl) 500 mg PO TID x 10 days. Discontinuation of the offending antibiotic (if possible) or switching to another antibiotic class is recommended. The role of probiotic supplementation is controversial. Complications are pseudomembranous colitis, toxic megacolon, and fulminant colitis.

You are following up a 65-year-old male who has been on a new prescription of fluvastatin (Lescol) for 6 weeks. During a follow-up visit, he reports feeling extremely fatigued and having dark-colored urine. He denies any generalized muscle soreness. Which of the following is the most appropriate treatment plan? A) Order a CBC with differential B) Order a liver function profile C) Recommend an increase in fluid intake and rest D) Order a urine for C&S test

B) Order a liver function profile Statin medications, such as Lescol, can affect liver function and increase liver enzymes, as well as cause the patient to feel weak, fatigued, and have muscle aches. Therefore, checking the liver function profile is recommended.

A 35-year-old male presents complaining of the acute onset of episodes of dizziness with nausea that started a few days after he got over a cold. The patient describes it as the sensation of the room moving or of the room spinning. It is worsened by sudden head movement. During the episodes, he gets very nauseated. He also has tinnitus with hearing loss in his right ear. The patient is a type 2 diabetic and is on a prescription of metformin 500 mg PO BID and an ACE inhibitor. The blood glucose during the visit is 80 mg/dL. Which of the following conditions is most likely? A) Vasovagal presyncopal episode B) Ménière's disease C) Atypical migraine D) Hypoglycemia

B) Ménière's disease The classic triad of symptoms of Ménière's disease are vertigo, tinnitus, and hearing loss. The condition can resolve spontaneously or may be chronic. BPV has similar symptoms except that it does not cause hearing loss. Vertigo is due to the dysfunction of the labyrinthine system. Differential diagnoses are many (Ménière's, acute labyrinthitis, acoustic neuroma, etc.). Vasovagal syncope does not cause hearing loss or tinnitus, nor is it episodic.

Cluster headaches are most often seen in: A) Adolescent females B) Middle-aged men C) Elderly men D) Postmenopausal women

B) Middle-aged men Cluster headaches are most commonly seen in middle- aged men.

Jean, a 68-year-old female, is suspected of having Alzheimer's disease. Which of the following is the best initial method for assessing the condition? A) Computed tomography scan of the brain B) Mini Mental Status Exam C) Obtain the history from the patient, friends, and family members D) EEG (electroencephalography)

B) Mini Mental Status Exam Begin with administering the MMSE for a baseline assessment, followed by taking a history from the patient and others.

Which of the following is indicated for initial treatment of an uncomplicated case of Helicobacter pylori negative peptic ulcer disease? A) Omeprazole (Prilosec) B) Misoprostol (Cytotec) C) Ranitidine (Zantac) D) Pepto-Bismol tablets

B) Misoprostol (Cytotec) Misoprostol is recommended for short-term, uncomplicated PUD; it acts by decreasing gastric acid production and enhancing mucosal resistance to injury.

During a routine physical exam of a 90-year-old woman, a low-pitched diastolic murmur Grade II/VI is auscultated. It is located on the fifth ICS on the left side of the midclavicular line. Which of the following is the correct diagnosis? A) Aortic regurgitation B) Mitral stenosis C) Mitral regurgitation D) Tricuspid regurgitation

B) Mitral stenosis Mitral stenosis is best heard at the fifth ICS to the left of the midclavicular line. It is ausculated as a low-pitched diastolic murmur, grade II/ VI. Aortic regurgitation is a high-pitched diastolic murmur, heard at the second ICS to the right of the sternum. Mitral regurgitation is a pansystolic murmur that radiates to the axilla, loud and high pitched when ausculated.

A 28-year-old student is seen in the school health clinic with complaints of a hacking cough that is productive of small amounts of sputum and a runny nose. He does not take any medications, denies any allergies, and has no significant medical history. Physical examination reveals a low-grade temperature of 99.9 degrees Fahrenheit, respirations of 16/min, a pulse of 90 beats per minute, and diffuse fine crackles in the base of the lungs. A chest radiograph (x-ray) shows diffuse infiltrates on the lower lobe of the right lung. The total white blood cell count is 10,500/uL. What is the most likely diagnosis? A) Streptococcal pneumonia B) Mycoplasma pneumonia C) Acute bronchitis D) Legionnaires disease

B) Mycoplasma pneumonia Mycoplasma pneumonia is the organism most com- monly seen in children and young adults. It is easily spread from droplets, from sneezing and coughing, in close proximity. Diagnosis is based on symptoms and x-ray results of infiltrates in lower lobes.

A lab technician has a 10.5-mm area of redness and induration in his left forearm after getting a Mantoux test 72 hours ago. The last test, which was done 12 months ago, was negative. He denies cough, night sweats, and weight loss. What is the next best intervention? A) Obtain a sputum culture and a chest x-ray B) Obtain a chest x-ray C) Obtain a sputum for C&S and an acid fast stain D) Obtain a CBC and chest x-ray

B) Obtain a chest x-ray Mantoux results are negative if results show less than 10 mm induration on the forearm for low-risk clients. If results are greater or equal to 10 mm, then a chest x-ray should be ordered for further screening.

A 68-year-old woman has recently been diagnosed with polymyalgia rheumatica. The nurse practitioner is discussing the treatment options with the patient. Which of the following medications is the first-line treatment for this condition? A) Etanercept (Enbrel) B) Oral prednisone C) Indomethacin D) Methotrexate

B) Oral prednisone Patients with PMR are treated with oral steroids (glucocorticoids). One of the hallmarks of the disorder is the dramatic improvement of symptoms after starting treatment with oral prednisone. Usually, the symptoms can be controlled with long-term (2-3 years) low-dose oral prednisone, which can be tapered when symptoms are under control. For most patients, PMR is a self-limiting illness (from a few months to 3 years).

A 65-year-old Hispanic woman has a history of type 2 diabetes. A routine urinalysis reveals a few epithelial cells and is negative for leukocytes, nitrites, and protein. Which of the following would you recommend next? A) Order a urine for C&S B) Order a 24-hour urine for microalbumin C) Because it is negative, no further tests are necessary D) Recommend a screening IVP (intravenous pyelogram)

B) Order a 24-hour urine for microalbumin Epithelial cells are cells that are present on the inside lining of the organs. Few epithelial cells can be normal; however, with her history of type 2 diabetes, a 24-hour urine for microalbumin should be ordered to assess kidney function.

A patient is positive for anti-HCV (hepatitis C virus antibody). What is the next step to further evaluate this patient? A) Refer the patient to a gastroenterologist B) Order a hepatitis C PCR (polymerase chain reaction) test C) Order a hepatitis B comprehensive panel D) The patient is immune to hepatitis C and no further testing is indicated

B) Order a hepatitis C PCR (polymerase chain reaction) test When a patient tests positive for anti-HCV (hepatitis C virus antibody), labs ordered should include hepatitis C PCR to further evaluate the patient.

You note the following result on a routine urinalysis of a 37-year-old primigravida who is at 30 weeks of gestation. Leukocyte = trace, nitrite = negative, protein = 2 +, blood = negative. Her weight has increased by 5 lbs during the past week. Which of the following is most likely? A) HELLP syndrome B) Pregnancy-induced hypertension (preeclampsia) C) Eclampsia of pregnancy D) Primary hypertension

B) Pregnancy-induced hypertension (preeclampsia) The classic triad of symptoms of preeclampsia includes hypertension, edema (weight gain), and proteinuria.

You are checking a 75-year-old woman's breast during an annual gynecologi- cal exam. The left nipple and areola are scaly and reddened. The patient denies pain and pruritis. She has noticed this scaliness on her left nipple for the past 8 months. Her dermatologist gave her a potent topical steroid, which she used twice a day for 1 month. The patient never went back for the follow-up. She still has the rash and wants an evaluation. Which of the following is the best intervention for this patient? A) Prescribe another potent topical steroid and tell the patient to use it twice a day for 4 weeks B) Order a mammogram and refer the patient to a breast surgeon C) Advise her to stop using soap on both breasts when she bathes to avoid drying up the skin on her areolae and nipples D) Order a sonogram and fine-needle biopsy of the breast

B) Order a mammogram and refer the patient to a breast surgeon A scaly, reddened rash on the breast that does not resolve after a few weeks of medical treatment may indicate breast cancer. She should have a mammogram performed and see a breast surgeon for evaluation and treatment.

An elderly woman has been on digoxin (Lanoxin) for 10 years. Her electrocardio- graph (EKG) is showing a new onset of atrial fibrillation. Her pulse is 64/min. She denies syncope and dizziness. Which of the following interventions is most appropriate? A) Order an electrolyte panel and a digoxin level B) Order a serum thyroid-stimulating hormone (TSH), digoxin level, and an electrolyte panel C) Order a serum digoxin level and decrease her digoxin dose by half while waiting for results D) Discontinue the digoxin and order another 12-lead EKG

B) Order a serum thyroid-stimulating hormone (TSH), digoxin level, and an electrolyte panel Obtaining baseline blood work to evaluate for causes of new- onset atrial fibrillation is recommended prior to decreasing or stopping medications. Thyroid disease is a common cause of new-onset atrial fibrillation.

Glucosamine sulfate is a natural supplement that is used for which of the following conditions? A) Rheumatoid arthritis B) Osteoarthritis C) Osteoporosis D) Metabolic syndrome

B) Osteoarthritis Glucosamine sulfate has been found to have a beneficial effect on cartilage growth and repair. It may also have an anti-inflammatory effect. Many patients with OA who take it claim that it helps to relieve joint pain. It can take from 1 to 3 months of taking the medicine to feel its effects. Glucosamine is a compound made up of glucose and an amino acid.

What is the most common cause of infertility among women in the United States? A) Scarring of the fallopian tubes due to a history of pelvic inflammatory disease (PID) B) Ovulation disorders C) Age older than 35 years D) Endometriosis

B) Ovulation disorders Ovulation disorders are the top cause of female infertility (25%). There is no ovulation (anovulation) or infrequent ovulation that results in oligoamenorrhea (i.e., PCOS). The second cause is endometriosis (15%). About 10% of women in the United States (ages 15 to 44 years) have difficulty getting pregnant (CDC, 2009). PCOS is one of the most common causes of female infertility. Infertility in males is often caused by a varicocele (heats up the testes) and abnormal and/or low sperm count.

A 40-year-old cashier complains of periods of dizziness and palpitations that have a sudden onset. The EKG shows P waves before each QRS complex and a heart rate of 170 beats/minute. A carotid massage decreases the heart rate to 80 beats/min. These best describe: A) Ventricular tachycardia B) Paroxysmal atrial tachycardia C) Atrial fibrillation D) Ventricular fibrillation

B) Paroxysmal atrial tachycardia Signs/symptoms of paroxysmal atrial tachycardia include rapid, regular heart rate that begins and ends very quickly. The atria are beating at a very fast rate, but it is not life-threatening. Ventricular tachycardia is usually associated with heart disease, occurs when the ventricles are beating rapidly and inefficiently, and can lead to death if not treated. Atrial fibrillation is an irregular heartbeat that can be life threatening if not treated. Ventricular fibrillation occurs when the heartbeat is rapid and chaotic, and death will occur if the condition is not treated.

Which of the following is not a characteristic of delirium? A) Sudden onset B) Patient is coherent C) Worse in the evenings D) It has a brief duration

B) Patient is coherent Characteristics of delirium include an acute and dramatic onset of symptoms that is temporary, and usually will worsen in the evening. May last hours to days. Patient is incoherent and disoriented. Usually brought on by fever, shock, drugs, alcohol, or dehydration.

Hypovolemic shock would most likely occur with fractures of the: A) Spine B) Pelvis C) Femur D) Humerus

B) Pelvis Hypovolemic shock can occur from a fractured pelvis secondary to internal bleeding from a fractured bone fragment that lacerates an artery or vein. The pelvis can accommodate a large amount of fluid.

Which of the following conditions is a possible complication of severe eclampsia? A) Placenta previa B) Placenta abruptio C) Erythroblastosis fetalis D) Uterine rupture

B) Placenta abruptio Abruptio placenta is a possible complication of severe eclampsia. With elevated blood pressure, the placenta can pull away from the uterine lining and cause painful, bright red bleeding.

A 38-year-old multigravida who is at 32 weeks of gestation calls the family nurse practitioner complaining of bright red vaginal bleeding. There is no watery dis- charge. She complains that her uterus feels hard and is very painful. Which of the following conditions is most likely? A) Placenta previa B) Placenta abruptio C) A molar pregnancy D) An ectopic pregnancy

B) Placenta abruptio Abruptio placenta symptoms are bright red vaginal bleeding, board-like uterus on palpation, and pain. However, there can be concealed hemorrhage and the patient may not have vaginal bleeding. Placenta previa is painless bleeding. Ectopic and molar pregnancy would not progress to 32 weeks gestation

A 28-year-old multipara who is at 32 weeks of gestation presents to your office complaining of a sudden onset of small amounts of bright red vaginal bleeding. She has had several episodes and appears anxious. On exam, her uterus is soft to palpation. Which of the following is most likely? A) Placenta abruptio B) Placenta previa C) Acute cervicitis D) Molar pregnancy (hydatidiform mole)

B) Placenta previa Placenta previa occurs when abnormal implantation of the placenta occurs. A common symptom of placenta previa is painless, bright red bleeding.

Rocky Mountain spotted fever is caused by the bite of the: A) Mosquito B) Tick C) Insect D) Flea

B) Tick Rocky Mountain spotted fever is caused by a tick bite from a tick that was infected with the parasite Rickettsia rickettsii. The mortality rate is 1%-7% if left untreated.

Medicare Part D will reimburse for which of the following services? A) Preventive health care such as routine Pap smears and physical exams B) Prescription drugs C) Alcohol misuse/abuse counseling D) Over-the-counter drugs and vitamins

B) Prescription drugs Medicare Part D is a voluntary program that charges a premium. Like all Medicare services, patients need to enroll during the "open enrollment" periods during the year (there is a penalty for late enrollment). There is a drug formulary and not all drugs are available or reimbursed. Use of generic drugs is preferred, as there is a spending limit. Medicare Part B will reimburse for alcohol misuse/abuse treatment.

Folic acid supplementation is recommended for women who are planning pregnancy in order to: A) Prevent renal agenesis B) Prevent anencephaly C) Prevent kidney defects D) Prevent heart defects

B) Prevent anencephaly Folic acid supplementation during pregnancy has been shown to decrease the risk of neural tube defects of the fetus

Jane, a young primigravida, reports to you that she is starting to feel the baby's movements in her uterus. This is considered to be which of the following? A) Presumptive sign B) Probable sign C) Positive sign D) Possible sign

B) Probable sign Probable signs of pregnancy are symptoms of pregnancy that are felt by the woman.

Hegar's sign is considered a: A) Positive sign of pregnancy B) Probable sign of pregnancy C) Presumptive sign of pregnancy D) Problem in pregnancy

B) Probable sign of pregnancy Hegar's sign is softening of the lower portion of the uterus and is considered a probable sign of pregnancy.

The most important job of an institutional review board (IRB) is: A) Protecting the interests of the hospital or the research institution B) Protecting the rights of the human subjects who participate in research done at the institution C) Protecting the researcher and research team from lawsuits D) Evaluating research protocols and methodology for appropriateness and safety

B) Protecting the rights of the human subjects who participate in research done at the institution Every research institution has an institutional review board (IRB) whose job is to review all the research that is conducted in that institution. The IRB's most important role is to protect the rights of the human subjects who participate in research done at the institution of which the IRB is a part (e.g., research hospitals, universities).

You note, during a physical exam on a 6-year-old child, some pitting on the finger nails. This finding is correlated with: A) Iron-deficiency anemia B) Psoriasis C) Onychomycosis D) Vitamin C deficiency

B) Psoriasis Fingernail pitting is correlated with psoriasis. Psoriasis can cause pitting on all finger and toenails, along with thickening and irregular shape of the nail.

During a routine physical exam of an elderly woman, a triangular thickening of the bulbar conjunctiva on the temporal side is noted to be encroaching on the cornea. She denies any eye pain or visual changes. Which of the following is most likely? A) Corneal arcus B) Pterygium C) Pinguecula D) Chalazion

B) Pterygium Pterygium is a triangular growth on the white part of the eye that also extends onto the cornea. Corneal arcus is a ring around the edge of the cornea. Chalazion is a stye in the eye that may cause pain and swelling. Pinguecula is a benign growth on the conjuctiva caused by the degeneration of its collagen fibers. Thick, yellow fibers may be seen.

What type of follow-up should this patient receive? A) Refer him within 48 hours to a urologist B) Refer him to the emergency department as soon as possible C) Prescribe ibuprofen (Advil) 600 mg QID for pain D) Order a testicular ultrasound for further evaluation

B) Refer him to the emergency department as soon as possible Immediate referral to the emergency department is required. Success of treatment is usually 100% if treated within the first 6 hours and 0% if treated after 24 hours.

Which type of exercise would you recommend to a 65-year-old arthritic patient who complains of a new onset of a painful, swollen left knee caused by gardening for 2 days? A) Quadriceps-strengthening exercises of the left knee followed by the application of cold packs for 20 minutes 4 times a day B) Rest the joint and apply cold packs intermittently for the next 48 hours C) Passive range of motion and cold packs D) A cool tub bath with warm packs on the knee to avoid stiffening of the joint

B) Rest the joint and apply cold packs intermittently for the next 48 hours New onset of a painful, swollen left knee caused by unusual activity should be treated with rest and alternating ice packs to rule out other causes of pain first

The best test for diagnosing congenital glaucoma is which of the following? A) Fluorescein staining B) Tonometry C) Snellen vision exam D) The refractive index

B) Tonometry A tonometer is used to measure the intraocular pressure (IOP) of the eye to screen for glaucoma. Normal range IOP is 10-22 mm Hg.

Mrs. Green, age 45, is complaining of generalized morning stiffness, especially in both her wrist and hands. It is much worse in the morning and lasts for a few hours. She also complains of fatigue and generalized body aches that have been present for the past few months. Which of the following is most likely? A) Osteoporosis B) Rheumatoid arthritis C) Osteoarthritis D) Gout

B) Rheumatoid arthritis Signs and symptoms of rheumatoid arthritis include symmetrical joint stiffness, swelling, and pain in the hands, wrists, elbows, ankles, and shoulders. Other common complaints include fatigue, malaise, fever, and generalized body aches.

A 15-month-old child who is eating and behaving normally is found to have a high fever. After a few days, the fever resolves and the infant breaks out in a maculopapular rash. This is a description of which of the following conditions? A) Erythema infectiosum B) Roseola infantum C) Fifth disease D) Scarlet fever

B) Roseola infantum The signs/symptoms of roseola infantum include high fever for a few days with a maculopapular rash occuring after the fever breaks. Fifth disease is a lacy-appearing rash that usually begins on the face and moves to the extremities and trunk. It is known by the "slapped cheek " appearance at onset. Erythema infectiosum is the same as Fifth disease. Scarlet fever is a red rash that feels like sandpaper that usually begins on the neck and trunk and spreads to the extremities. Patients will have other symptoms such as fever, sore throat, and beefy red tongue. This is caused by the Group A Streptococcus bacterium.

The mother of a 12-month-old infant reports to the nurse practitioner that her child had a high fever for several days, which spontaneously resolved. After the fever resolved, the child developed a maculopapular rash. Which of the following is the most likely diagnosis? A) Fifth disease (erythema infectiosum) B) Roseola infantum (exanthema subitum) C) Varicella D) Infantile maculopapular rashes

B) Roseola infantum (exanthema subitum) Roseola infantum is a common viral rash that is caused by the human herpes virus; the most common ages of onset are between 6 months and 2 years. The rashes are maculopapular (small round pinkcolored) rashes that first appear on the trunk and then spread to the extremities.

A 40-year-old White female with a BMI (body mass index) of 32 complains of col- icky pain in the right upper quadrant of her abdomen that gets worse if she eats fried food. During the physical exam, the nurse practitioner presses deeply on the left lower quadrant of the abdomen. After releasing her hand, the patient complains of pain on the right side of the lower abdomen. What is the name of this finding? A) Rebound tenderness B) Rovsing's sign C) Murphy's sign D) Psoas test

B) Rovsing's sign Rovsing's sign is referred pain to the opposite side of the abdomen after release of palpation.

A toddler with congenital heart disease is seen for a 1-week history of facial and lower-extremity edema accompanied by shortness of breath. The child's mother reports that the child's appetite has been poor. The chest x-ray reveals that the child has congestive heart failure (CHF). Which of the following heart sounds are found in these patients? A) S1 and S2 B) S1, S2, and S3 C) S1, S2, and S4 D) Still's murmur and S4

B) S1, S2, and S3 Congestive heart failure is the inability of the heart to pump a sufficient amount of blood to the organs to meet the body's requirements. It is common to hear S1, S2, and S3 heart sounds on exam. Common signs and symptoms include fatigue, shortness of breath with activity, and edema of lower extremities.

A 16-year-old complains of a severe sore throat for 3 days along with a generalized rash and fever. The skin has the texture of fine sandpaper. This best describes: A) Kawasaki's disease B) Scarlet fever C) German measles D) Rubeola

B) Scarlet fever Scarlet fever is a bacterial infection caused by the group A Streptococcus infection. The patient will exhibit sore throat, high fever, and a fine sandpaper rash, which is characteristic of scarlet fever.

A 15-year-old female who attends a public school is referred to the nurse practitioner by one of her teachers. The teen has been missing school and is falling behind in her schoolwork. After closing the exam room door, the nurse practitioner starts to interview the teen, asking about her moods, her appetite, her sleep, whether she any plan of hurting herself or others, and other questions. What is the level of health prevention that the nurse practitioner is performing? A) Primary prevention B) Secondary prevention C) Tertiary prevention D) Dropout prevention program

B) Secondary prevention The nurse practitioner is evaluating the teenager for major depression. Because the teenager already has the disease (depression), it is a screening "test." All screening tests/labs (mammography, Pap smears, etc.) are secondary prevention activities.

A hypertensive middle-aged man who is Native American has recently been diagnosed with mild renal insufficiency. He has been on lisinopril (Accupril) for many years. Which of the following laboratory values should be carefully monitored? A) Hemoglobin, hematocrit, and the MCV (mean corpuscular volume) B) Serum creatinine and potassium levels C) Aspartate aminotransferase and alanine aminotransferase D) Serum sodium, potassium, and magnesium

B) Serum creatinine and potassium levels Native Americans have a much higher rate of kidney disease and renal failure when compared to other races. Native Americans have a 1 in 3 incidence of hypertension. Hypertension is the second leading cause of kidney failure. BUN, creatinine, estimated GFR, and urinalysis are performed to assess the function of the kidneys.

A 25-year-old female presents with an onset of severe right-sided pelvic pain for the past 48 hours. She reports small amounts of vaginal bleeding. The pain is aggravated by jumping or any movement that jars her pelvis. The best initial intervention is which of the following? A) Follicle-stimulating hormone (FSH) B) Serum quantitative pregnancy test C) Pelvic ultrasound D) CBC with white cell differentials

B) Serum quantitative pregnancy test Severe right-sided pain with vaginal bleeding in a 25-year-old patient requires an evaluation for pregnancy, by performing a serum quantitative pregnancy test. Ectopic pregnancy and/or miscarriage should be ruled out.

You would advise an 18-year-old female student who has been given a booster dose of MMR at the college health clinic that: A) She might have a low-grade fever during the first 24 to 48 hours B) She should not get pregnant within the next 4 weeks C) Her arm will be very sore on the injection site for 24 to 48 hours D) Her arm will have some induration on the injection site in 24to48hours

B) She should not get pregnant within the next 4 weeks MMR should not be administered to women known to be pregnant. In addition, women should be counseled to avoid becoming pregnant for 28 days following vaccination.

The nurse practitioner is evaluating a middle-aged female who is complaining of gradual weight gain, lack of energy, dry hair, and an irregular period over an 8-month period. The routine annual laboratory testing showed a TSH result of 10 mU/L. The nurse practitioner decides to order a thyroid profile. The TSH is 8.50 mU/L and the serum free T4 is decreased. During the physical exam, the patient's BMI is 28. The heart and lung exams are both normal. Which of the fol- lowing is best treatment plan? A) Advise the patient that because the TSH level has decreased, she does not have a thyroid problem anymore B) Start the patient on levothyroxine (Synthroid) 0.25 mcg PO daily C) Start the patient on Armour thyroid D) Refer the patient to an endocrinologist

B) Start the patient on levothyroxine (Synthroid) 0.25mcg PO daily The patient is symptomatic (weight gain, lack of energy, and irregular periods) with low free T4. Even though the TSH went down slightly, the free T4 remains low. An elevated TSH and low free T4 are indicative of hypothyroidism. The next step is to start the patient on levothyroxine (Synthroid) 0.25 mcg daily and recheck the TSH in 6 weeks. The goal is to normalize the TSH (between 1.0 and 3.5) and to ameliorate the patient's symptoms (increased energy, feels better, etc.). Armour thyroid (desic- cated thyroid) is a natural supplement composed of dried (desiccated) pork thyroid glands. It is used in alternative medicine as an alternative to synthetic levothyrox- ine/T4 (Synthroid).

The cover/uncover test is a screening tool for: A) Color blindness B) Strabismus C) Visual acuity D) Cataracts

B) Strabismus The cover/uncover test screens for strabismus. Color blindness is evaluated by using the Ishihara tool. Visual acuity is evaluated using the Snellen chart. Cataracts are screened by using the ophthalmoscope and shining a light into the eyes at an angle, approximately 15 inches away. If the red reflex is not elicited, then cataracts are present

A newly diagnosed middle-aged type 2 diabetic wants to start an exercise program. All of the following statements are true except: A) If the patient is unable to eat due to illness, anti-diabetic agents can be continued with frequent glucose monitoring B) Strenuous exercise is contraindicated for type 2 diabetics because of a higher risk of hypoglycemic episodes C) Exercise increases the body's ability to metabolize glucose D) Patients who exercise vigorously in the afternoon may have hypoglycemic episodes in the evening or at night if they do not eat

B) Strenuous exercise is contraindicated for type 2 diabetics because of a higher risk of hypoglycemic episodes Exercise is recommended because exercise helps to use the glucose stores and reduce blood sugar. Blood sugar should be monitored closely, especially if on insulin, when exercising to avoid hypoglycemia.

A post menopausal woman's dual-energy x-ray absorption (DXA or DEXA) result shows osteopenia. Which of the following t-scores is indicative of osteopenia? A) T-score of -1.0 or higher B) T-score between -1. 0 to -2.5 C) T-score of less than -2.5 D) T-score of -2.50 to -3.50 D) Normal results

B) T-score between -1. 0 and -2.5 Osteopenia is defined as a t-score between -1. 0 and -2.5. Osteoporosis is defined as a t-score of less than -2.5.

A college freshman who is on oral contraceptives calls the nurse practitioner's office asking for advice. She forgot to take her pills 2 days in a row during the second week of the pill cycle and wants to know what to do. What is the best advice? A) Start a new pack of pills and dispose of the old one B) Take 2 pills today and 2 pills the next day; use condoms for the rest of the cycle C) Stop taking the pills right away and start a new pill cycle in 2 weeks D) Take 1 pill now and 2 pills the next day and use condoms

B) Take 2 pills today and 2 pills the next day; use condoms for the rest of the cycle When forgetting to take the birth control pill on 2 consecutive days, it is recommended that she take 2 pills today and 2 pills tomorrow, then continue the rest of her pack. Stress the importance of the use of condoms for protection against pregnancy and STIs.

At what Tanner stage does puberty start? A) Tanner Stage I B) Tanner Stage II C) Tanner Stage III D) Tanner Stage IV

B) Tanner Stage II Puberty is defined as the period in life when secondary sexual characteristics begin to develop, identified as Tanner Stage II for boys and girls.

When an adolescent male's penis grows more in length than width, at which of the following Tanner stages is he classified? A) Tanner Stage II B) Tanner Stage III C) Tanner Stage IV D) Tanner Stage V

B) Tanner Stage III Tanner Stage III in males consists of penis lengthening and darker, coarse pubic hair, which begins to curl

A sexually active 16-year-old female is brought in by her mother for a physical exam. During the exam, the nurse practitioner notices some bruises on both breasts. All of the following are important areas to evaluate in this patient during this visit except: A) Depression B) Tanner stage C) Sexual history D) STDs

B) Tanner stage The stem of the question is asking for the important areas to evaluate in this patient "during this visit." This is a priority-type question. The priorities to evaluate in this patient are depression, STD testing, and sexual history. The Tanner staging does not have to be done "during this visit."

A 13-year-old adolescent female is brought in by her mother for a sports physical. The mother reports that the teen's last vaccines were given at the age of 6 years. Which of the following vaccines is recommended by the CDC for this patient? A) Td and HPV vaccines B) Tdap, MCV4, and the HPV vaccines C) DTap and the flu vaccine D) DT and MCV4 vaccines

B) Tdap, MCV4, and the HPV vaccines Vaccine questions usually are not this complicated, but there are several lessons that can be learned with this question. The 2012 CDC recommendations for the ages 13 to 18 years are the Tdap catch-up (if did not receive at age 11-12 years), HPV or Gardasil catch-up (if did not receive at age 11-12 years), and the MCV4 or meningococcal conjugate vaccine (Menactra). Only one dose of Tdap is recommended (lifetime). Thereafter, the Td form of the vaccine is indicated every 10 years.

Which of the following is useful in the primary care area when evaluating a patient for possible acute sinusitis or hydrocele? A) Checking for the cremasteric reflex B) Transillumination C) Ultrasound D) CT scan

B) Transillumination Transilliumination is a technique used with a light source, such as the otoscope, to visualize fluid below the skin surface, which will appear as a "glow." Ultrasound and CT scan are not readily available in the primary care setting; the patient must be sent to radiology for these tests to be performed

Laura, who is 18 years old, is being followed up for acne by the nurse practitioner. The facial exam reveals that papules and pustules are present mostly on the forehead and the chin areas. The patient has been using prescription topicals and OTC medicated soap daily for 6 months without much improvement. Which of the following would you recommend next? A) Isotretinoin (Accutane) B) Tetracycline (Sumycin) C) Clindamycin topical solution (Cleocin T) D) Minoxidil (Rogaine)

B) Tetracycline (Sumycin) First-line treatment for acne vulgaris includes OTC medicated soap and water with topical antibiotic gels. The next step of treatment would be the initiation of oral tetracycline.

All of the following are true statements about the human papilloma virus vaccine (Gardasil) except: A) The Centers for Disease Control and Prevention (CDC) recommends the first dose at age 11 to 12 years B) The CDC does not recommend the HPV vaccine for males C) The vaccine is not complete until a total of 3 doses have been administered D) The minimum age the vaccine can be given is 9 years

B) The CDC does not recommend the HPV vaccine for males The HPV vaccine is now recommended for both males and females. It can be given until the age of 26 years, especially if the individual is at high risk. Do not use the vaccine if age is less than 9 years.

A 14-year-old female with amenorrhea is tested for pregnancy and has a positive result. The patient tells the NP that she is seriously considering terminating the pregnancy. She tells the NP that she wants to be referred to a Planned Parenthood clinic. The NP's personal beliefs and religious beliefs are pro-life. Which of the fol- lowing is the best action for the NP? A) The nurse practitioner should tell the patient about her personal beliefs and advise her against getting an abortion B) The NP advises the patient that an NP peer who is working with the NP can help answer her questions more thoroughly C) The NP should excuse herself from the case D) The nurse practitioner should refer the patient to an obstetrician

B) The NP advises the patient that an NP peer who is working with the NP can help answer her questions more thoroughly In general, discussing personal beliefs is considered unprofessional behavior. Respecting the patient's right to choose is an example of supporting patient autonomy.

Which of the following is a true statement regarding pes planus in an infant? A) Pes planus should be evaluated by a pediatric orthopedist if it does not spontaneously correct itself by the age of 12 months B) The fat pads on an infant's feet can mimic pes planus C) It is always corrected by wearing special orthotic shoes D) It is also called talipes equinovarus

B) The fat pads on an infant's feet can mimic pes planus The fat pads on an infant's feet can resemble pes planus or flat feet.

The mother of a 4-month-old calls your office and reports that the infant has a fever of 101.4 degrees Fahrenheit. The infant received her immunizations yesterday. Which of the following is correct? A) The fever is most likely due to the combination of the MMR and polio vaccines B) The fever is most likely due to the pertussis component of the DTP vaccine C) The infant is probably starting a viral upper respiratory infection D) The infant had an allergic reaction to one of the vaccines given and should be brought to the emergency room

B) The fever is most likely due to the pertussis component of the DPT vaccine The pertussis component of the vaccine is most likely the cause of the fever.

The Patient Protection and Affordable Care Act is President Obama's plan for national insurance coverage. All of the following are true statements about this law except: A) Preexisting health conditions will be covered immediately B) The health plan will start to be effective in the year 2013 C) Young adults up to the age of 26 years who live with their parents will be covered under their parents' health insurance plan D) The health insurance plan mandates will fine employers who choose not to participate in the national health insurance plan

B) The health plan will start to be effective in the year 2013 President Obama's national health insurance plan is known the "Patient Protection and Affordable Care Act." It covers preexisting health conditions immediately and prohibits an insurance company from rejecting people with preexisting health conditions. There is a penalty for employers (and individuals) who choose not to participate in the national health plan.

Which of the following statements is false regarding jaundice in breastfed infants? A) Breastfed infants have a higher incidence of hyperbilirubinemia compared with formula-fed infants B) The mechanism of breast milk jaundice is still not known C) Phytotherapy is usually not indicated for these infants D) It usually starts in the first week of life

B) The mechanism of breast milk jaundice is still not known Jaundice occurs after the first week of life for breastfed infants. Breastfed infants usually have a higher incidence of jaundice, and the condition may last for 2-3 weeks. Bilirubin is broken down and excreted in the urine and stool. The mechanism of breast milk jaundice is still unknown.

All of the following are true statements regarding Munchausen syndrome except: A) It is considered a mental illness B) The patient has a medical illness that causes an anxiety reaction and denial C) The patient fakes an illness in order to gain attention from health care providers D) The patient has an inconsistent medical history along with a past history of frequent hospitalizations

B) The patient has a medical illness that causes an anxiety reaction and denial Munchausen syndrome is a psychiatric disorder in which the patient fakes a medical illness or disorder to gain attention from health care provid- ers. These patients commonly use the emergency department frequently to gain attention.

A 70-year-old woman complains of left lower quadrant abdominal pain and fever for 2 days. Her blood pressure of 130/80, pulse 90/minute, respirations 14/minute and a temperature of 100.5°F. During the abdominal exam, the left lower quadrant of the abdomen is tender to palpation. The NP does not palpate a mass, guarding, or rigidity. Rovsing's sign is negative. Bowel sounds are present in all quadrants. The nurse practitioner is familiar with the patient, who is alert and is asking appropriate questions about her condition. The nurse practitioner suspects that the patient has acute diverticulitis. Which of the following treatment plans is appropriate for this patient? A) The patient should be referred to the physician as soon as possible B) The patient has a mild case of acute diverticulitis and can be treated with antibiotics in the outpatient setting with close follow-up C) This patient has a moderate to severe case of acute diverticulitis and needs to be admitted to the hospital for IV antibiotics D) Refer the patient to the emergency department as soon as possible

B) The patient has a mild case of acute diverticulitis and can be treated with antibiotics in the outpatient setting with close follow-up The patient has a mild case of acute diverticulitis and can be treated as an outpatient with antibiotics and a clear fluid diet. If outpatient treatment is selected, close follow-up (within 24 to 48 hours) is very important. Instruct patients to go to the hospital if symptoms get worse, if fever increases, if unable to tolerate PO treatment, and if pain worsens. Order CBC (leukocytosis, neutrophils, and possible shift to the left), chemistry profile, and urinalysis (to rule out renal causes)

What is the clinical significance of the above finding on a 35-year-old? A) The patient has a higher risk of blindness B) The patient should be evaluated for hyperlipidemia C) The patient should be evaluated by an ophthalmologist D) The patient should be evaluated for acute glaucoma

B) The patient should be evaluated for hyperlipidemia Arcus senilis is caused by lipid deposits deep in the edge of the cornea.

A split S2 is best heard at which of the following areas? A) The aortic area B) The pulmonic area C) The tricuspid area D) The mitral area

B) The pulmonic area The split S2 is best heard at the pulmonic area of the heart.

The major difference between the quasi-experimental design is which of the following? A) The quasi-experimental design is a type of observational study B) The quasi-experimental design uses convenience sampling instead of random sampling to recruit subjects C) The quasi-experimental design is also known as a survey D) The quasi-experimental design does not have an intervention group

B) The quasi-experimental design uses convenience sampling instead of random sampling to recruit subjects The quasi-experimental design uses an intervention (it is not an observational study or a survey). It has many similarities to an experimental study except that the human subjects are recruited by convenience and not at random (as in an experimental study).

The research term/symbol that is used to indicate the "total population" in a research study is: A) The symbol called "n = " B) The symbol called "N =" C) The symbol called "p =" D) The symbol called "P ="

B) The symbol called "N = " The correct symbol to indicate total number of subjects in a study (total population) is "N." For example, a research study has a total number of subjects or total population of 100 (N = 100). The small letter n is used to indicate a subpopulation. For example, a study uses a total population of N = 100 that is divided into 2 groups of 50 subjects (n = 50).

The following statements are all true regarding herpes zoster except: A) It is due to reactivation of latent varicella virus B) The typical lesions are bullae C) It is usually more severe in immunocompromised individuals D) Infection of the trigeminal nerve ophthalmic branch can cause corneal blindness

B) The typical lesions are bullae Herpes zoster occurs secondary to reactivation of the varicella virus. This infection can be more severe in the immunocompromised patient due to the inability to fight infection. This rash appears as a vesicular, painful rash, commonly in clusters, following one dermatome on one side of the body. When the trigeminal nerve is involved, there is an increased risk of corneal blindness.

During a sports physical exam, a1 6-year-old patient is noted to have a few beats of horizontal nystagmus on extreme lateral gaze that disappear when the eyes move back toward midline. Which statement best describes this clinical finding? A) It is caused by occult bleeding of the retinal artery B) This is a normal finding C) It is a sign of a possible brain mass D) This is a borderline result and requires further evaluation

B) This is a normal finding Horizontal nystagmus is a normal variation on physical exam

While reviewing some lab reports, the nurse practitioner notes that one of her teenage male patient's lab results is abnormal. The liver function tests are all normal except for a slight elevation in the alkaline phosphatase level. The patient is a member of a soccer team and denies any recent injury. What is the next step in this patient's evaluation? A) Order a liver ultrasound to rule out fatty liver B) This is a normal finding for his age group C) The patient needs to be evaluated further for pancreatic disease D) Refer the patient to a pediatric rheumatologist

B) This is a normal finding for his age group Elevated levels of alkaline phosphatase are commonly seen in growing children. Growth of bone tissue elevates the alkaline phosphatase levels.

Which of the following is not a relative contraindication for oral contraceptive pills? A) Active hepatitis A infection B) Thrombosis related to an IV needle C) Undiagnosed vaginal bleeding D) Migraine headache without focal aura

B) Thrombosis related to an IV needle Thrombosis related to either a known trauma or IV needle does not represent a contraindication for use of oral contraceptives.

The best screening test for detecting and monitoring both hyperthyroidism and hypothyroidism is: A) The total T3-4 B) Thyroid-stimulating hormone (TSH) C) Thyroid profile D) Palpation of the thyroid gland

B) Thyroid-stimulating hormone (TSH) The best "screening" test of hypothyroidism and hyperthyroidism is the TSH. If this result is abnormal, then further diagnostic tests should be performed.

The bacille Calmette-Guerin (BCG) vaccine is used to immunize a person against which of the following? A) Enterobiasis B) Tuberculosis C) Anthrax D) Smallpox

B) Tuberculosis The BCG vaccine is given routinely in some countries where tuberculosis is endemic (or epidemic). One of the few exceptions for the BCG vaccine in the United States is for health care workers who see a high percentage of patients who are infected with M. tuberculosis strains resistant to both isoniazid and rifampin. BCG is considered a biohazardous material (U.S. Black Box Warning) and proper handling and disposal must be followed.

Which of the following is a good example of how the "utilitarian" principle is applied? A) Helping a patient decide the type of treatment that he/she wants B) Using limited societal financial resources on programs that will positively affect the largest number of people possible and have the lowest possible negative outcomes C) Minimize the bad outcome when choosing treatment choices for a patient D) Health caregivers should be more careful when using health care financial resources

B) Using limited societal financial resources on programs that will positively affect the largest number of people possible and have the lowest possible negative outcomes Generally, the utilitarian principle refers to societal programs that will affect or benefit the largest number of people in a positive manner. It is not used to refer to an individual or to one person.

Women who are pregnant during the winter months are recommended to have which of the following? A) Increased intake of Vitamin C and folate B) Vaccination against the influenza virus C) Increased caloric intake of fruits and vegetables D) Heavier winter clothes to avoid chilling the fetus

B) Vaccination against the influenza virus Influenza vaccine is recommended for all pregnant patients for prevention due to the decrease in immune status during pregnancy. The vaccine is safe to use during pregnancy.

What type of breath sounds are best heard over the base of the lungs? A) Fine breath sounds B) Vesicular breath sounds C) Bronchial sounds D) Tracheal breath sounds

B) Vesicular breath sounds Vesicular breath sounds are heard best over the base of the lungs. Vesicular sounds are soft and/or blowing, heard throughout inspiration, and fade away with expiration. Bronchial sounds are heard over the bronchi, the largest tubes in the anterior chest. Sounds are loud and high pitched. Tracheal breath sounds are heard over the trachea. These sound harsh and similar to air being blown through a pipe.

Which of the following is the confirmatory test for the HIV screening test? A) ELISA test for HIV B) Western blot C) HIV polymerase chain reaction test D) HIV antibody

B) Western blot A positive Western blot test confirms an HIV screening test.

The nurse practitioner calls a patient to discuss the results of routine laboratory tests, which are all normal. She calls the patient twice and each time, the answering machine is on. Which of the following is the most appropriate action for the nurse practitioner? A) Because they are all normal, leave the laboratory results on the answering machine B) When the nurse practitioner is unable to speak with the patient directly, she should leave a message with her name and telephone number and instruct the patient to call back C) Some clinics have a policy of not calling patients back if their laboratory results are normal D) It is up to the physician to determine whether a laboratory result can be discussed with a patient on the telephone

B) When the nurse practitioner is unable to speak with the patient directly, she should leave a message with her name and telephone number and instruct the patient to call back It is against HIPAA regulations to leave laboratory results on an answering machine even if they are normal, routine laboratory tests. HIPAA is also known as the "Privacy Rule."

What does a KOH (potassium hydroxide) prep help the nurse practitioner diagnose? A) Herpes zoster infections B) Yeast infections C) Herpes simplex infections D) Viral infections

B) Yeast infections The KOH prep test is performed by placing a sample of dis- charge on a glass slide, with one drop of potassium hydroxide and a coverslip on top to evaluate for yeast infections. Budding spores and pseudohyphae will be seen with Candida, with a pH of 3.5-4.5.

Which of the following pharmacologic agents is the best choice for an elderly patient with insomnia? A) Diazepam (Valium) B) Zolpidem (Ambien) C) Temazepam (Restoril) D) Diphenhydramine (Benadryl)

B) Zolpidem (Ambien) Ambien has a quick onset of action (15 minutes) and a short half-life of 2 hours. Avoid diphenhydramine in the elderly, as there is a higher incidence of adverse effects (confusion, prolonged sedation) in this population. Avoid long-acting benzodiazepines such as diazepam (half-life 12 hours) and temazepam (half-life of 10 hours). Hypnotics can be used PRN up to 2 weeks; otherwise, benzodiazepines can cause addiction and withdrawal symptoms.

The ELISA and Western Blot tests are both used to test for HIV. Which of the following statements is correct? A) It is a test to detect viral RNA B) A positive ELISA screening does not mean the person has HIV infection C) It is a test to detect viruses D) It is a diagnostic test for the AIDS virus

B)A positive ELISA screening does not mean the person has HIV infection The ELISA test is always followed by a Western blot test to confirm diagnosis.

All of the following drugs can interact with theophylline (Theo-24) except: A) Erythromycin B) Montelukast (Singulair) C) Phenytoin sodium (Dilantin) D) Cimetidine (Tagamet)

B)Montelukast(Singulair) Medications that are contraindicated with theophylline include erythromycin, phenytoin sodium, and cimetidine.

During a breast exam of a 30-year-old nulliparous female, the nurse practitioner palpates several rubbery mobile areas of breast tissue. They are slightly tender to palpation. Both breasts have symmetrical findings. There are no skin changes or any nipple discharge. The patient is expecting her menstrual period in 5 days. Which of the following would you recommend? A) Referral to a gynecologist for further evaluation B) Tell her to return 1 week after her period so her breasts can be rechecked C) Advise the patient to return in 6 months to have her breasts rechecked D) Schedule the patient for a mammogram

B)Tell her to return 1 week after her periods so her breasts can be rechecked Prior to having menses, women will experience breast changes of tenderness with palpation, with symmetrical soft, mobile areas of breast tissue that are benign. Her symptoms should improve after her menses and repeating the exam in 1 week would be recommended.

What is the pedigree symbol for a diseased male? A) An empty square B) An empty circle C) A filled-in square D) A filled-in circle

C) A filled-in square A filled-in square is a diseased or affected male and a filled-in circle is a diseased or affected female. An empty square is a healthy male and an empty circle is a healthy female.

A 39-year-old migrant worker presents to the clinic 2.5 days after a purified protein derivative (PPD) test. What minimum size of induration would be considered posi- tive for this patient? A) 3 mm B) 5 mm C) 10 mm D) 15 mm

C) 10 mm The PPD is administered on the volar aspect of the lower arm and read 48 hours after the test is given. The PPD result must have induration and measure 10 mm or greater to be positive in a low-risk patient. Induration (firmness with palpation) must be present. If the site has erythema but no induration, result would be negative. Color is not important.

When does an infant triple its birth weight? A) 3 months B) 6 months C) 12 months D) 15 months

C) 12 months By the age of 12 months, an infant is expected to have tripled its birth weight.

What is the caloric content of infant formula and breast milk? A) 10 kcal/30 mL B) 15 kcal/30 mL C) 20 kcal/30 mL D) 25 kcal/30 mL

C) 20 kcal/30 mL The caloric content of infant formula and breast milk is 20k cal/30 mL.

Mrs. J. L. is a 55-year-old female with a body mass index (BMI) of 24 and a history of asthma. She has hypertension that has been under control with hydrochlorothiazide 12.5 mg PO daily. Her total cholesterol is 230 g/dl. How many risk factors for coronary heart disease (CAD) does she have? A) 1 risk factor B) 2 risk factors C) 3 risk factors D) 4 risk factors

C) 3 risk factors Risk factors for coronary heart disease include hypertension, hypercholesterolemia, and women 55 years of age and older

At what age can a child copy a circle and ride a tricycle? A) 1 year B) 2 years C) 3 years D) 4 years

C) 3 years Developmental stages in children include the following: 1 year: walk; 2 years: walks up steps with the same foot; 3 years: pedals a tricycle and copies a circle; 4 years: rides a bicycle and copies a cross and draws a person with 2 parts.

The following are patients who are at high risk for complications due to urinary tract infections. Who does not belong in this category? A) A 38-year-old diabetic patient with a HbA1C of 7.5% B) A woman with a history of rheumatoid arthritis who is currently being treated with a regimen of methotrexate and low-dose steroids C) A 21-year-old woman who is under treatment for 2 sexually transmitted infections D) Pregnant women

C) A 21-year-old woman who is under treatment for 2 sexually transmitted infections Risk factors for complications due to urinary tract infection include pregnancy, diabetes, and steroid therapy.

What is the best description of Cullen's sign? A) The onset of hyperactive bowel sound before the onset of ileus B) A reddish-purple discoloration that is located on the flank area C) A bluish discoloration or bruising that is located on the umbilical area D) The acute-onset subcutaneous bleeding seen during acute pancreatitis

C) A bluish discoloration or bruising that is located on the umbilical area Cullen's sign is the acute onset of bluish discoloration that is located on the umbilical/periumbilical area; it is caused by bruising underneath the skin. The bluish discoloration that is located on the flank area is called the Grey-Turner sign. It is a sign of a severe case of pancreatitis.

All of the following are associated with emphysema except: A) A barrel-shaped chest B) Pursed-lip breathing C) A chest radiograph result with infiltrates and flattening of the costovertebral angle D) Dyspnea when at rest

C) A chest radiograph result with infiltrates and flattening of the costovertebral angle Emphysema is characterized by having a barrel-shaped chest, pursed- lip breathing, and dyspnea when at rest. Infiltrates on an x-ray indicate bacterial infection, such as pneumonia

The following children are considered at higher risk for tuberculosis (TB) except: A) A child who has recently been diagnosed with leukemia B) An infant whose family is homeless C) A child who was born in Japan D) A Hispanic child with asthma who is using a steroid inhaler

C) A child who was born in Japan Risk factors for tuberculosis include compromised immune system, high-risk populations (minorities, foreign-born people, prisoners, nursing home residents, indigents, migrant workers and health care providers). Steroid therapy and blood disorders also increase the risk of TB.

Pulsus paradoxus is best described as: A) An increase in systolic blood pressure on inspiration B) A decrease in diastolic blood pressure on exhalation C) A decrease in systolic blood pressure on inspiration D) An increase in diastolic blood pressure on expiration

C) A decrease in systolic blood pressure on inspiration Pulsus paradoxus: With inspiration, systolic pressure drops due to the increased pressure (positive pres- sure). Some pulmonary risks of having increased pressure include asthma and emphysema

The following conditions are absolute contraindications for the use of oral contraceptives except: A) Hepatomas B) History of emboli that resolved with heparin therapy 15 years ago C) A family history of migraines with aura D) A history of gallbladder disease during pregnancy

C) A family history of migraines with aura All of the items are contraindications, but a family history does not substantiate a need to avoid oral contraceptives.

Which of the following should you expect to find on a wet-mount slide of a patient diagnosed with bacterial vaginosis? A) Tzanck cells B) A large amount of leukocytes and epithelial cells C) A large amount of bacteria coating the squamous cells and very few leukocytes D) Epithelial cells and a small amount of blood 108

C) A large amount of bacteria coating the squamous cells and very few leukocytes Results of the wet mount for bacterial vaginosis include bacteria coating the squamous cells (clue cells) and very few leukocytes or RBCs

All of the following pharmacologic agents are used to treat inflammation in the lungs of asthmatics except: A) Nedocromil sodium (Tilade) two sprays QID B) Cromolyn sodium inhaler (Intal) two puffs QID C) A long-acting oral theophylline (Theo-Dur) 200 mg every 12 hours D) Fluticasone inhaler (Flovent) two puffs BID

C) A long-acting oral theophylline (Theo-Dur) 200 mg every 12 hours Theophylline is used to control inflammation of the lungs. Tilade, Intal, and Flovent help to treat inflammation.

Which of the following patients is least likely to become an alcoholic? A) A patient whose father has a history of alcoholism B) A patient whose wife complains that he drinks too much C) A patient who drinks one cup of wine nightly with dinner D) A patient who feels he drinks all the time

C) A patient who drinks one cup of wine nightly with dinner Excessive use or exposure to alcohol puts that patient at risk for becoming an alcoholic. A patient who drinks one glass of wine at dinner has a lower risk of becoming an alcoholic than someone who has been exposed to alcohol while growing up, one who drinks all the time, or one whose family believes he is drinking excessively.

During a routine physical exam on an 82-year-old, the nurse practitioner palpates an irregular mass on the midabdomen that is not tender and is about 2 cm in size. Which of the following is the best initial imaging test to further evaluate the abdominal mass? A) CT scan of the abdomen B) KUB study C) Abdominal ultrasound D) MRI of the abdomen

C) Abdominal ultrasound The ultrasound or sonogram is used as an initial imaging test for abdominal tumors. A CT scan can be ordered at a later time, but it is not considered an initial imaging test in the primary care area.

The following skin findings are considered macules except: A) A freckle B) Petechiae C) Acne D) A flat, 0.5-cm brown birthmark

C) Acne A macule is a flat, non-raised lesion on the skin. Acne lesions are pap- ules because they consist of raised, erythemic lesions on the skin. A freckle, petechiae, and a flat birthmark are all considered macules.

The following findings are considered benign lesions of the skin except: A) Lentigo B) Seborrheic keratosis C) Actinic keratosis D) Rosacea

C) Actinic keratosis Actinic keratosis is a small, raised skin lesion on skin that has been in the sun for a long period of time. Actinic keratoses are usually benign, but can develop into skin cancer; therefore, further evaluation is needed to deter- mine if removal is required.

A 12-year-old girl is complaining of a 2-week history of facial pressure that worsens when she bends over. She complains of tooth pain in her upper molars on the right side of her face. On physical exam, her lung and heart sounds are normal. Which of the following is the most likely diagnosis? A) An acute dental abscess B) Chronic sinusitis C) Acute sinusitis D) Severe allergic rhinitis

C) Acute sinusitis Signs/symptoms of acute sinusitis includes headache, facial pain that worsens with bending over, eye/ear pressure and pain, aching in upper jaw/teeth, reduced smell and taste, cough (especially at night due to the nasal drainage), sore throat, bad breath, and fatigue.

All of the following are implicated in causing chronic cough except: A) Chronic bronchitis B) Allergic rhinitis C) Acute viral upper respiratory infection D) Gastroesophageal reflux disease

C) Acute viral upper respiratory infection Chronic cough can be caused by chronic bronchitis, allergic rhinitis, and GERD.

While performing a sports physical on a 16-year-old girl, the nurse practitioner notes a split S2 during inspiration that disappears during expiration. The girl is active and her growth and development have been uneventful. What is the best recommendation for the child's mother? A) Her daughter needs to be referred to a pediatric cardiologist B) Her daughter needs to be referred for a stress EKG C) Advise the mother that this is a normal finding D) The child should avoid strenuous physical exertion until further evaluation

C) Advise the mother that this is a normal finding Explain to the mother that in healthy adolescents, a split S2 during inspiration that disappears during expira- tion is a normal variation.

An 13-year-old girl is brought in by the mother because her daughter is complaining of vaginal discharge and pain. The mother tells the nurse practitioner that her daughter is not sexually active yet. The mother is divorced and lives with her boy- friend and works full time. During the exam, the nurse practitioner notes that the vaginal introitus is red, with tears and a torn hymen. The cervix is covered with green discharge. The nurse practitioner suspects that the child has been sexually abused by the mother's boyfriend. What is the best action for the nurse practitioner to take? A) Ask the mother questions about her boyfriend's behaviors B) Advise the mother to watch how her boyfriend interacts with her daughter and to call within 1 week to discuss his behavior with her C) Advise the mother that you suspect that her daughter has been sexually abused and that she is legally required to report the case to the child protection program D) Report the child abuse to the local police department

C) Advise the mother that you suspect that her daughter has been sexually abused and that she is legally required to report the case to the child protection program The NP is legally required to report the case to the child protection program. If the child is in danger, child protective services may ask for a court order to take the child away for protection until the investigation is completed. Talking about the boyfriend's behavior will not be effective and may put the child and/or mother in danger if the boyfriend suspects that he is being watched.

Which of the following is appropriate follow-up for this 70-year-old patient? A) Referral to an optometrist B) Referral to an ophthalmologist C) Advise the patient that it is a benign condition and will resolve spontaneously D) Prescribe an ophthalmic antibiotic solution

C) Advise the patient that it is a benign condition and will resolve spontaneously A subconjunctival hemorrhage is a benign disorder that resolves without any treatment

A 44-year-old patient with Down syndrome starts to develop impaired memory and difficulty with his usual daily life routines. He is having problems functioning at the job that he has done for the past 10 years. The physical exam and routine labs are all negative. The vital signs are normal. His appetite is normal. The most likely diagnosis is: A) Tic douloureux B) A stroke C) Alzheimer's disease D) Delirium

C) Alzheimer's disease Delirium is an acute decline in mental status and is temporary. Common causes are fever, shock, drugs, alcohol, and dehydration. Alzheimer's disease is a permanent change to the brain that causes short-term memory loss, agnosia, apraxia, and aphasia. In this case, the patient's physical exam is normal; however, he is having memory loss and difficulty working and carrying out his normal tasks

A new mother reports to you that her 6-month-old infant has a cold and has a fever of 99.8 degrees Fahrenheit. The infant is not irritable and is feeding well without problems. The mother wants to know whether it is okay for him to be immunized at this time. Which of the following statements is true? A) The infant should not be immunized until he is afebrile B) An infant with a cold can be immunized at any time C) An infant with a cold can be immunized as long as the infant's temperature is no higher than 100.4 degrees Fahrenheit D) Because immunization is so important, it should be given to the infant as scheduled

C) An infant with a cold can be immunized as long as the infant's temperature is no higher than 100.4 degrees Fahrenheit Immunization is acceptable as long as the temperature is not higher than 100.4°F.

The NP suspects that a middle-aged woman may have systemic lupus erythematosus (SLE). Which of the following laboratory tests is strongly associated with this disease? A) Sedimentation rate B) C-reactive protein C) Antinuclear antibody (ANA) D) Systemic lupus erythematosus (SLE)

C) Antinuclear antibody (ANA) The ANA is usually positive in lupus patients. Other types of autoantibody testing recommended for these patients, in addition to antinuclear antibody (ANA) tests, are antiphospholipid antibodies, antibodies to double-stranded DNA, and anti-Smith (Sm) antibodies. Patient with suspected lupus should be referred to a rheumatologist. The sedimentation rate and the C-reactive protein are nonspecific findings of inflammation and are present in autoimmune diseases, infections, and others.

When palpating a woman who is at 20 weeks of gestation, the nurse practitioner should be able to feel the uterine fundus at what level? A) Just rising above the level of the pubic symphysis B) Between the pubic symphysis and the umbilicus C) At the level of the umbilicus D) Above the level of the umbilicus

C) At the level of the umbilicus Uterine growth at 12 weeks gestation, palpable just above the symphysis pubis; 20 weeks gestation, at the level of the umbilicus; after this, measurements in cm should approximately equal weeks of gestation.

Which of the following is a CDC-recommended treatment for a case of uncomplicated gonorrheal and chlamydial infection? A) Metronidazole (Flagyl) 250 mg PO TID x 7 days B) Valacyclovir (Valtrex) 500 mg PO BID x 10 days C) Azithromycin 1 g orally OR doxycycline 100 mg orally twice a day for 7 days D) 1 dose of oral fluconazole (Diflucan) 150 mg

C) Azithromycin 1 g orally OR doxycycline 100 mg orally twice a day for 7 days Drug therapy is based on the 2010 CDC STD guidelines for treatment.

An older male is diagnosed with conductive hearing loss in the left ear by the nurse practitioner. Which of the following is the expected result on this patient when performing a Rinne test? A) AC (air conduction) > BC (bone conduction) B) Lateralization to the bad ear C) BC > AC D) Lateralization to the good ear

C) BC > AC A normal result in the Rinne test is AC greater than BC. When there is a conduction hearing loss (i.e., cerumenosis, otitis media), the result will be BC greater than AC. The reason is that the sound waves are blocked (i.e., cerumen, fluid in middle ear). Therefore, the patient cannot hear them as well as through bone conduction.

A middle-aged hypertensive male presents to a public health clinic with complaints of an acute onset of fever, chills, and cough that is productive of rusty-colored sputum. The patient reports episodes of sharp pains on the left side of his back and chest whenever he is coughing. His temperature is 102.2 degrees Fahrenheit, the pulse is 100/min, and the BP is 130/80. The urinalysis does not show leukocytes, nitrites, or blood. This finding is most consistent with: A) Atypical pneumonia B) An upper urinary tract infection C) Bacterial pneumonia D) Acute pyelonephritis

C) Bacterial pneumonia Symptoms of bacterial pneumonia include high fever, chills, productive cough, and pain in back/chest area with deep inspiration and cough.

Which of the following classes of drugs is implicated with blunting the signs and symptoms of hypoglycemia in diabetics? A) Calcium channel blockers B) Diuretics C) Beta-blockers D) ARBs (angiotensin receptor blockers)

C) Beta-blockers Beta-blockers are known to blunt the signs/symptoms of hypoglycemia in patient with diabetes.

Which of the following antihypertensive medications should the nurse practitioner avoid when treating patients with emphysema? A) Calcium channel blockers B) Angiotensin-converting enzyme (ACE) inhibitors C) Beta-blockers D) Diuretics

C) Beta-blockers Beta-blockers should be avoided in patients with a history of emphysema. Studies have shown evidence of a reduction in forced expiratory volume in 1 second (FEV1), increased airway hyperresponsiveness, and inhibition of bronchodilator response to beta agonists in patients receiving non-selective beta- blockers and high doses of cardioselective beta-blockers.

A 55-year-old nurse brings her mother, who is 82 years of age, to the emergency department of a local hospital. She reports she found her mother on the floor when she checked on her that morning. Her mother was awake and oriented, but needed help getting up. Her mother states that she thinks she passed out. She is being evaluated by a physician who orders an EKG and x-rays of both hips. Regarding laboratory testing, which of the following is important to perform initially? A) Urinalysis B) Serum electrolytes C) Blood glucose D) Hemoglobin and hematocrit

C) Blood glucose Checking the blood glucose is indicated for patients with syncopal and near syncopal episodes. The NP has performed a thorough history of the incident (diabetic, rapid onset or slow, position, provocation), medical history, health history, and the medications.

All of the following groups are classified as "vulnerable populations" and have additional protections as human subjects except: A) Prisoners B) Pregnant women, fetuses, and children C) Frail elders D) Mentally incompetent individuals

C) Frail elders Frail elders who are mentally competent are not considered a vulnerable population group.

A new mother is breast feeding her full-term 4-week-old infant. She wants to know whether she should give the infant vitamin supplements. The best advice is: A) Since she is breast feeding, the infant does not need any vitamin supplements until he is at least 6 months of age B) Breast milk gives the infant all the vitamins he needs until 12 months of age C) Breastfed infants require iron and Vitamin D supplementation right away D) BreastfedinfantsrequireironandVitaminEsupplementationat3monthsof age

C) Breastfed infants require iron and Vitamin D supplementation right away While infants do receive Vitamin D through human milk, the AAP recom- mends a daily supplement of 200 IU Vitamin D for all breastfed infants.

A first-grader presents to a school nurse practitioner with a few blisters on one arm and on the face. The child keeps scratching the affected areas. Some of the lesions have ruptured with yellow serous fluid that crusts easily. This best describes: A) Acute cellulitis B) Herpes zoster C) Bullous impetigo D) Erysipelas

C) Bullous impetigo Bullous impetigo is a skin infection, commonly seen on the face and hands, with yellow, honey-colored fluid blisters with drainage that turn into scabbed lesions that continue to spread until treated.

Which cranial nerve (CN) is being evaluated when patients are instructed to shrug their shoulders? A) CN IX B) CN X C) CN XI D) CN XII

C) CN XI Cranial nerves IX, X, XI, and XII are: glossopharyngeal, vagal, spinal accessory, and hypoglossal, respectively.

Balanitis is caused by: A) Staphyloccocus aureus B) Streptococcus pyogenes C) Candida albicans D) Trichomonads

C) Candida albicans Balanitis is an yeast infection of the glans of the penis. Candida albicans is the causative source. Staphyloccocus aureus and Streptococcus pyogenes are bacterial infections. Trichomonads are protozoans that cause infection.

A 62-year-old female complains of chronic severe low back pain. She also reports mild episodes of fecal incontinence and numbness to her lower legs over the past week. You would suspect which of the following? A) Fracture of the lower spine B) A herniated disc C) Cauda equina syndrome D) Ankylosing spondylitis

C) Cauda equina syndrome Cauda equina syndrome is a surgical emergency. Signs and symptoms include a change in bowel and bladder control (incontinence). Acute pressure on the sacral nerve root causes inflammatory and ischemic changes to the nerve and can lead to permanent nerve damage.

Human papilloma virus infection in women has been associated with the development of: A) Ectopic pregnancy B) Infertility C) Cervical cancer D) Pelvic inflammatory disease

C) Cervical cancer HPV is a virus associated with cervical cancer. HPV vaccine is now available for girls and boys between the ages of 15-21 years of age to help prevent 4 strains of this virus that is linked to cervical cancer.

When the nurse practitioner is evaluating a patient for intermittent claudication, he/she would first: A) Order a venogram B) Order T.E.D. anti-embolism stockings C) Check the ankle and brachial blood pressure before and after exercise D) Check the pedal and posterior tibial pulses

C) Check the ankle and brachial blood pressure before and after excercise Initial evaluation for intermittent claudication would include checking the ankle and brachial blood pressure before and after exercise.

All of the following may help relieve the symptom(s) of GERD except: A) Losing weight B) Stopping caffeine intake C) Chewing breath mints D) Stopping alcohol intake

C) Chewing breath mints GERD is a condition in which food comes up from the stomach/esophagus through a weak sphincter. It usually worsens with lying down, from the reflux, and can cause a cough and esophageal irritation if not treated. Effective treatment may include weight loss, decreased caffeine intake, and avoidance of alcohol.

A cauliflower-like growth with foul-smelling discharge is seen during an otoscopic exam of the left ear of an 8-year-old boy with a history of chronic otitis media. No tympanic membrane or ossicles are visible and the patient seems to have difficulty hearing the nurse practitioner's instructions. Which of the following conditions is best described? A) Chronic perforation of the tympanic membrane with secondary bacterial infection B) Chronic mastoiditis C) Cholesteatoma D) Cancer of the middle ea

C) Cholesteatoma Cholesteatoma is an abnormal growth that occurs in the middle ear that can cause hearing loss. Difficulty hearing is common.

Asthmatics may have all of the following symptoms during an exacerbation except: A) Rapid pulse B) Wheezing C) Chronic coughing D) Tachypnea

C) Chronic coughing Signs and symptoms of an acute exacerbation of asthma include breathlessness, coughing, wheezing, and chest tightness. Agitation, increased respiratory and pulse rate, and decreased lung function are also noted. A "chronic" cough is not a symptom.

Patients who are considered mentally competent have a right to consent or refuse medical treatment. What is the legal term for this right? A) Informed consent B) Durable power of attorney C) Competence D) Advance directives

C) Competence Competence is having the ability to make one's own decisions regarding one's own course of health care, as long as one is documented as being mentally competent. Informed consent is giving permission to perform a particular procedure, understanding the purpose of the procedure/treatment, success/failure rate, other alternatives to the procedure, risks and benefits of the procedure, prognosis, and success rate. To give informed consent, the patient must be 18 years old or older. Durable power of attorney is the legal document giving one person the authority to make decisions regarding another person's health care and personal affairs on that person's behalf. Advance directives are written instructions to be per- formed in the event the patient is not mentally competent and not able to make decisions for himself/herself. The Living Will is an example of an advance directive.

Which of the following structures of the eyes is responsible for color vision? A) Rods B) Macula C) Cones D) Pupils

C) Cones Rods and cones are photoreceptor cells of the retina. The cones of the eyes are responsible for color vision. Cones are very sensitive to colors (red, blue, or green) and work better in brighter light. Rods are good for night vision and for vision in low light conditions because they are sensitive to light and dark. To remember them, note that both cone and color start with the letter "C."

A 62-year-old male with chronic obstructive pulmonary disease (COPD) complains to the nurse practitioner that his prescription of ipratropium bromide (Atrovent) is not working. He reports that he still feels short of breath even after using it 4 times a day for 3 months. Which of the following actions is the next step for the nurse practitioner? A) Increase the patient's dose of ipratropium bromide (Atrovent) to 3 inhalations QID B) Continue the ipratropium bromide and start the patient on oxygen by nasal cannula C) Continue ipratropium bromide (Atrovent) and add albuterol (Ventolin) inhaler 2 inhalations QID D) Start the patient on oxygen by nasal cannula at bedtime and PRN during the daytime

C) Continue ipratropium bromide (Atrovent) and add albuterol (Ventolin) inhaler 2 inhalations QID Treatment of COPD starts with an anticholinergic (ipratropium bromide). The next step is to add a short-acting beta-2 agonist (albuterol)

The Rinne and the Weber tests are used to assess which of the following cranial nerve(s)? A) Cranial nerves 3, 4, and 6 B) Cranial nerve 7 C) Cranial nerve 8 D) Cranial nerves 9 and 10

C) Cranial nerve 8 The Rinne and Weber tests are used to assess cranial nerve 8 or the acoustic nerve. The patient's hearing is tested by air conduction (Rinne and Weber) and bone conduction (Rinne only).

Mr. Brown is a 65-year-old carpenter complaining of morning stiffness and pain in both his hands and right knee upon awakening. He feels some relief after warming up. On exam, the nurse notices the presence of Heberden's nodes. Which of the fol- lowing is most likely? A) Osteoporosis B) Rheumatoid arthritis C) Degenerative joint disease D) Reiter's syndrome

C) Degenerative joint disease Heberden's nodes are commonly seen in degenerative joint disease. Heberden's nodes are bony nodules located on the distal interphalangeal joints (DIPs).

At what time of the day would you recommend a scotch tape test be done to evaluate for a suspected case of enterobiasis? A) In the evening after dinner B) At night before bed C) Early in the morning D) It does not matter what time of the day the test is done

C) Early in the morning The scotch tape test is best done in the morning for several days in a row. The females do not lay eggs every day and therefore testing for several days will be more accurate.

A possible side effect from the use of nifedipine (Procardia XL) is: A) Hyperuricemia and hypoglycemia B) Hyperkalemia and angioedema C) Edema of the ankles and headache D) Dry hacking cough

C) Edema of the ankles and headache Common side effects of calcium channel blockers, such as Procardia, include edema of the ankles, dizziness, headaches, flushing, and weakness. ACE inhibitors tend to have the side effects of angioedema and a dry hacking cough. Diuretics can cause hyperkalemia and hyperuricemia.

A new mother who is on her fourth day of breastfeeding complains to the nurse practitioner of very sore breasts. The nurse practitioner would: A) Recommend a decrease in the number of times she breastfeeds her infant per day B) Recommend that she stop breastfeeding and use infant formula for the next 48 hours C) Educate the mother that this is normal during the first week or 2 of breastfeeding and the soreness will eventually go away D) Recommend that she purchase plastic nipple pads for her nursing bra and use them daily

C) Educate the mother that this is normal during the first week or 2 of breastfeeding and the soreness will eventually go away Nursing during the first 2 weeks after delivery may cause tenderness and soreness of the nipples and usu- ally resolves after this. The mother should continue to breastfeed as she has been advised, and she should make sure the infant is latching on appropriately.

When confirming a case of temporal arteritis, the erythrocyte sedimentation rate (ESR) is expected to be: A) Normal B) Lower than normal C) Elevated D) Indeterminate result

C) Elevated Temporal arteritis can be diagnosed by having an elevated erythrocyte sedimentation rate.

The nurse practitioner who suspects that one of her hypertensive patients has Cushing's syndrome would expect to find which of the following laboratory results? A) Hyponatremia B) Hypoglycemia C) Elevated serum cortisol levels D) Decreased urine 17-ketosteroids

C) Elevated serum cortisol levels Elevated serum cortisol levels are seen with Cushing's syndrome.

All of the following factors are associated with a higher risk of osteopenia and osteoporosis except: A) Excessive alcohol intake and cigarette smoking B) Asian or Caucasian ancestry C) Estrogen and progesterone deficiency D) Older age

C) Estrogen and progesterone deficiency Bone loss is not associated with progesterone deficiency (only estrogen deficiency). Risk factors include excessive alcohol intake, cigarette smoking, ancestry (Asian or Caucasian), older age, anorexia nervosa, small bone frame, and others.

Which of the following drugs is classified as a 5-alpha reductase inhibitor? A) Terazosin (Hytrin) B) Tamsulosin (Flomax) C) Finasteride (Proscar) D) Sildenafil (Viagra)

C) Finasteride (Proscar) Finasteride (Proscar) belongs to the drug class called 5-alpha reductase inhibitors. .It helps to lower serum testosterone, which helps to decrease the size of the prostate. It is also used for male-pattern baldness. A smaller prostate results in less obstructive voiding symptoms such as weak stream, frequency, and nocturia. Both terazosin (Hytrin) and tamsulosin (Flomax) are alpha-blockers and may start to control symptoms in as little as 3 days. They work by relaxing the smooth muscle tissue of the prostate gland, which enlarges the diameter of the urethra.

Which of the following drugs is classified as a 5-alpha reductase inhibitor? A) Terazosin (Hytrin) B) Tamsulosin (Flomax) C) Finasteride (Proscar) D) Sildenafil (Viagra)

C) Finasteride (Proscar) Finasteride (Proscar) belongs to the drug class called 5-alpha reductase inhibitors. It helps to lower serum testosterone, which helps to decrease the size of the prostate. It is also used for male-pattern baldness. A smaller prostate results in less obstructive voiding symptoms such as weak stream, frequency, and nocturia. Both terazosin (Hytrin) and tamsulosin (Flomax) are alpha-blockers and may start to control symptoms in as little as 3 days. They work by relaxing the smooth muscle tissue of the prostate gland, which enlarges the diameter of the urethra.

All of the following are true statements about sexuality in the older adult except: A) Erectile dysfunction is very common B) It may take longer to become aroused C) Frail elderly are not interested in sexuality anymore D) Dyspareunia is a common symptom of atrophic vaginitis

C) Frail elderly are not interested in sexuality anymore Some studies have shown that older adults in their 80s can remain sexually active. By the age of 70 years, about 80% of males have ED.

A multigravida who is at 34 weeks of gestation wants to know at what level her uterine fundus should be. The best answer is to advise the mother that her fun- dus is: A) Midway between the umbilicus and the lower ribs B) At the level of the umbilicus C) From 33 to 35 cm D) From 32 to 34 cm

C) From 33 to 35 cm After 20 weeks gestation, fundal height in centimeters should measure approximately the same as the number of weeks of gestation

Which of the following drugs is most likely to relieve the patient's symptoms? A) Atenolol (Tenormin) B) Trimethoprim/sulfamethoxazole (Bactrim DS) C) Furosemide (Lasix) D) Hydrocodone/guaifenesin syrup (Hycotuss)

C) Furosemide (Lasix) Lasix would be used to help remove the extra fluid load.

While assessing for a cardiac murmur, the first time that a thrill can be palpated is at: A) Grade II B) Grade III C) Grade IV D) Grade V

C) Grade IV A thrill is palpated at grade IV.

All of the following are considered instrumental activities of daily living (IADLs) except: A) Grocery shopping B) Managing one's finances C) Grooming and hygiene D) Using a telephone and a computer

C) Grooming and hygiene Grooming and hygiene are classified as basic activities of daily living (ADLs). Grocery shopping, paying bills, using telephones, and driving a car are all IADLs.

A middle-aged male who is homeless reports to the local Public Health Clinic com- plaining of a painless and shallow ulcer on the penile shaft for the past 2 weeks. He is sexually active and had unprotected intercourse with two male partners over the past few months. The patient is tested for HIV, syphilis, gonorrhea, hepatitis B, and herpes types 1 and 2. The syphilis and HIV tests are both positive. The gonorrhea, hepatitis B, and herpes tests are negative. The nurse practitioner is aware of Nationally Notifiable Infectious Conditions. Which of the following is true regarding reporting of any of these sexually transmitted infections? A) Obtain the patient's permission before reporting the positive HIV and syphilis test results to the local public health department B) Obtain the patient's and sexual partner's permission before reporting the posi- tive test results to the local health department C) Health care providers are mandated by law to report certain types of diseases to the local health department even if the patient does not give permission D) The nurse practitioner should consult with the supervising physician about this issue

C) Health care providers are mandated by law to report certain types of diseases to the local health department even if the patient does not give permission Physicians and laboratories are legally mandated to report certain types of diseases. STDs, HIV infection/AIDS, gonorrhea, and syphilis must be reported to the local health department even if the patient does not give permission. Partner tracing and notification are done by the local health department. The CDC web- site contains a list of nationally reportable diseases. Other diseases that are on the CDC 2012 list of reportable diseases (i.e., diseases that must be reported) are TB, diphtheria, hepatitis A, B, and C, measles, mumps, pertussis, Lyme disease, Rocky Mountain spotted fever, and many others.

All of the following are true about strawberry hemangiomas found in infants except: A) Most will involute spontaneously by the age of 18 to 24 months B) Watchful waiting is the most useful strategy C) Hemangiomas should be treated with laser therapy if they have not resolved by the age of 12 months D) Strawberry hemangiomas are benign

C) Hemangiomas should be treated with laser therapy if they have not resolved by the age of 12 months True strawberry hemangiomas will eventually resolve by the time the child goes to kindergarten. Most will reduce or disappear in the first 2 years. Laser treatment is rarely needed.

A 19-year-old female has recently been diagnosed with acute hepatitis B. She is sexually active and is monogamous. She reports using condoms inconsistently. What would you recommend for her male sexual partner who was also tested for hepatitis with the following results: HBsAg (−), anti-HBs (−), anti-HCV (−), anti-HAV (+)? A) A hepatitis B vaccination B) Hepatitis B immunoglobulin C) Hepatitis B vaccination and hepatitis B immunoglobulin D) No vaccination is needed at this time

C) Hepatitis B vaccination and hepatitis B immunoglobulin Hepatitis B vaccination is given for long-term prophylaxis treatment to prevent hepatitis B infection. Hepatitis B immunoglobulin is given for prevention of hepatitis B infection when the person has been directly exposed to the hepatitis B infection. The immnoglobulin is not a vaccine and does not protect against long-term prophylaxis.

A patient diagnosed with bacterial vaginosis should be advised that her sexual partner be treated with: A) Ceftriaxone(Rocephin)250mgIMwithdoxycycline100mgBIDfor14days B) Metronidazole (Flagyl) 500 mg PO BID for 7 days and 1 dose of azithromycin (Zithromax) C) Her partner does not need treatment D) Clotrimazole cream (Lotrimin) on his penis BID for 1 to 2 weeks

C) Her partner does not need treatment Bacterial vaginosis is a bacterial infection, but is not considered an STD for which the partner needs treatment. Studies show that men rarely carry this infection.

All of the following signs and symptoms are present with an anticholinergic drug overdose except: A) Dilated pupils B) Flushing and tachycardia C) Hypertension D) Confusion

C) Hypertension Drugs with strong anticholinergic properties include diphenhydramine, scopolamine, TCAs, antipsychotics, and others. The mnemonic is "dry as a bone, red as a beet, mad as a hatter, blind as a bat." Look for a low-grade temperature.

According to Erik Erikson, adolescents are at what psychosocial developmental stage? A) Autonomy vs. shame B) Industry vs. inferiority C) Identity vs. role confusion D) Intimacy vs. isolation

C) Identity vs. role confusion Adolescents (age 12 to 18 years) are in the stage known as "identity vs. role confusion." At this time, the teen is transitioning into adulthood and reexamining his/her identity and beliefs. The teen wonders about himself/herself (e.g., "who am I"). Peers are highly valued.

The first teeth to erupt during infancy are which of the following? A) First molars B) Second molars C) Incisors D) Canines

C) Incisors The incisors are the first teeth to erupt during infancy.

All of the following physiologic changes are present in the lungs of the elderly except: A) Decrease in the forced expiratory volume (FEV1) B) Slight increase in the residual volume (RV) C) Increase in lung compliance D) Airways tend to collapse earlier (than young patients) with shallow breathing

C) Increase in lung compliance There is a decrease in lung compliance as we get older; therefore, the FEV1 also decreases. There is minimal change in the total lung volume. Airways tend to collapse earlier (than young patients) with shallow breathing, which increases the risk of pneumonia.

Which of the following drugs is effective therapy for treating pain in patients who are having an acute exacerbation of gout? A) Acetaminophen (Tylenol) B) Systemic steroids C) Indomethacin (Indocin) D) Allopurinol (Zyloprim)

C) Indomethacin (Indocin) Common medications used for acute flare-ups of gout include two NSAIDs, such as indomethacin and naproxen sodium, BID prn. Colchicine may be added to the NSAIDs if relief is not obtained. Maintenance therapy consists of allopurinol and/or probenecid.

A coworker calls the nurse practitioner and wants to know about a patient's progress. She tells the NP that they are neighbors and she is worried about the patient's health status. The coworker works in the same facility, but is not directly involved with this patient's care. Which of the following actions is the most appropriate? A) Share with the coworker the patient's health status B) Advise the coworker to call the patient right away and ask her for verbal permission C) Inform the coworker that you cannot release any information about this patient because the coworker is not directly involved in the patient's care D) Reassure the coworker that the patient is doing fine and is getting better quickly

C) Inform the coworker that you cannot release any information about this patient because the coworker is not directly involved in the patient's care Releasing any information about this patient to a coworker who is not directly involved in the patient's care would violate the HIPAA privacy rules

A research participant tells the nurse practitioner that he wants to withdraw from the study. Regarding this case, the nurse practitioner is aware that all research study consent forms should contain which of the following information? A) The patient's demographic information B) The possible risks of the study C) Information that a research subject can voluntarily withdraw from the study at any time without any penalties or adverse consequences D) The benefits from the study

C) Information that a research subject can voluntarily withdraw from the study at any time without any penalties or adverse consequences Information that should be included in the written consent forms is that the research subject can voluntarily withdraw from the study at any time without any penalties or adverse consequences. In addition, the human subject should be verbally informed of this right.

Which of the following anatomic areas are involved with sensorineural hearing loss? A) Outer ear B) Middle ear C) Inner ear D) Cranial nerve VIIII

C) Inner ear Sensorineural hearing loss (i.e., presbycusis) involves damage to both the hair cells in the cochlea (sensory portion) and cranial nerve VIII (neural).

AllofthefollowingarecorrectstatementsregardingtheS3componentoftheheart sound except: A) It occurs very early in diastole and is sometimes called an opening snap B) It is a normal finding in some children, healthy young adults, and athletes C) It can be a normal variant if heard in a person age 40 or older D) It signifies CHF (congestive heart failure)

C) It can be a normal variant if heard in a person age 40 or older S3 heart sounds occur early in diastole and are sometimes referred to as an "opening snap." It is a normal variant in children, healthy young adults, and athletes. Bibasilar crackles in lung bases and the presence of S3 heart sounds are classic findings of CHF.

A 17-year-old high school student is considering her birth control options. She wants to know more about Seasonale. Which of the following statements is false? A) Taking Seasonale results in only 4 periods per year B) Her period will occur within the 7 days when she is on the inert pills C) It is a progesterone-only method of birth control and does not contain estrogen D) Take 1 tablet daily for 84 consecutive days followed by 7 days of inert pills

C) It is a progesterone-only method of birth control and does not contain estrogen Seasonale is an extended-cycle form of birth control. It contains both levonorgestrel and ethinyl estradiol. There are 84 pink tablets (active) and 7 white pills (inert). In general, more spotting (breakthrough bleeding) is experienced with extended-cycle pills during the first few months of use (compared with the monthly birth control pills).

Which of the following is correct regarding the best site to listen for mitral regurgitation? A) The apical area during S2 B) It is best heard at the base at S1 C) It is best heard at the apex at S1 D) It is best heard at the base at S2

C) It is best heard at the apex at S1 Mitral regurgitation is best heard at the apical area; it is a high-pitched, blowing pansystolic murmur.

A new patient who is a 40-year-old female postal worker is being evaluated for complaints of a new onset of an erythematous rash on both her cheeks and on the bridge of the nose that is accompanied by fatigue. She reports a history of Hashimoto's thyroiditis and is currently being treated with Synthroid 1.25 mg daily. Which of the following conditions is most likely? A) Atopic dermatitis B) Thyroid disease C) Lupus erythematosus D) Rosacea

C) Lupus erythematosus Classic symptoms of lupus erythematosus are butterfly rash across both cheeks and bridge of nose and fatigue. Risk factors also include being female and 40 years old.

The red blood cells in pernicious anemia will show: A) Microcytic and hypochromic cells B) Microcytic and normochromic cells C) Macrocytic and normochromic cells D) Macrocytic and hypochromic cells

C) Macrocytic and normochromic cells The blood cells in pernicious anemia will display macrocytic, normochromic cells. RBCs in iron-deficiency anemia are microcytic and hypochromic.

What is the first-line class of antibiotics recommended by the American Thoracic Society (ATS) for patients younger than 60 years of age who are diagnosed with community-acquired pneumonia with no comorbidity? A) First-generation cephalosporins B) Second-generation cephalosporins C) Macrolides D) Beta-lactam antibiotics

C) Macrolides The American Thoracic Society recommends macrolides as the first line of therapy for community-acquired pneumonia without comorbidity.

A 65-year-old woman's bone density result shows severe demineralization of cortical bone. All of the following pharmacologic agents are useful in treating this condition except: A) Raloxifene (Evista) B) Calcitonin (Miacalcin) C) Medroxyprogesterone (Depo-Provera) D) Calcium with Vitamin D

C) Medroxyprogesterone (Depo-Provera) Common treatments for osteoporosis include calcium and Vitamin D, Evista, Miacalcin, and bisphosphonates. Depo- Provera is a type of contraception.

Beta thalassemia minor is considered a: A) Macrocytic anemia B) Normocytic anemia C) Microcytic anemia D) Hemolytic anemia

C) Microcytic anemia Beta thalassemia minor is a genetic disorder in which the bone marrow produces small, pale RBCs in which mild hypochromic, microcytic anemia occurs.

Lead poisoning can cause which type of anemia? A) A mild macrocytic anemia B) Normocytic anemia C) Microcytic anemia D) A mild hemolytic anemia

C) Microcytic anemia Lead poisoning can cause microcytic anemia. Signs and symptoms of lead poisoning are abdominal pain, constipation, vomiting, and blue-black line on the gums. Lead causes anemia by mimicking healthful minerals such as calcium, iron, and zinc. It is absorbed by the bones, where it interferes with the production of RBCs. This absorption can also interfere with calcium absorption that is needed to keep the bones healthy.

You note a high-pitched and blowing pansystolic murmur in a 70-year-old male. It is Grade II/VI and is best heard at the apical area. Which of the following is most likely? A) Ventricular septal defect B) Tricuspid regurgitation C) Mitral regurgitation D) Mitral stenosis

C) Mitral regurgitation Mitral regurgitation is best heard at the apical area, and manifests as a high-pitched, blowing pansystolic murmur.

A 12-year-old male's peak expiratory flow results indicate 60 to 80% of the predicted range. How would you classify his asthma? A) Mild intermittent asthma B) Mild persistent asthma C) Moderate persistent asthma D) Severe asthma

C) Moderate persistent asthma Asthma classifications: Intermittent: normal FEV between exacerbations, FEV > 80%; mild persistent, FEV > 80%; moderate persistent, FEV 60-80%; severe persistent, FEV < 60%.

Jane, a middle-aged nurse, complains of localized pain on the sole of her left foot that is located between the third and fourth toes. The pain is aggravated by weight bearing and feels like "a pebble in my shoe." During the physical examination, the nurse practitioner palpates a tender nodule in the metatarsal interspace on the left foot. No redness or swelling is noted. Which of the following conditions is being described? A) Plantar wart B) Foreign body C) Morton's neuroma D) Metatarsalgia

C) Morton's neuroma Morton's neuroma is characterized by experiencing pain, burning, or numbness with weight bearing between two toes. Some experience a sense or feeling like they are walking on a pebble between two adjacent toes.

All of the following are true statements about diverticula except: A) Diverticula are located in the colon B) A low-fiber diet is associated with the condition C) Most diverticula in the colon are infected with gram-negative bacteria D) Supplementingwithfibersuchaspsyllium(Metamucil)isrecommended

C) Most diverticula in the colon are infected with gram-negative bacte- ria Diverticuli in the colon can be infected with both gram-negative and gram- positive bacteria.

An elderly Hispanic male has been taking finasteride (Proscar) for several months. The nurse practitioner decides to check the prostate specific antigen (PSA). The PSA result is 10 ng/mL. The patient's baseline PSA was 30 ng/mL. Which of the following is the next step? A) Add the baseline PSA value to the treatment PSA value B) The treatment PSA value is the correct value C) Multiply the treatment PSA value by 2 D) Divide the baseline PSA value by the treatment PSA value

C) Multiply the treatment PSA value by 2 Before starting a prescription of Proscar, obtain the baseline PSA. Recheck the PSA again within 2 to 3 months during treatment to assess the patient's response (treatment PSA). For this example, the corrected treatment PSA is 20 (multiply 10 x 2 = 20). When com- paring the corrected treatment PSA (20) with the baseline PSA (30), the value is lower (means the prostate has shrunk in size). The patient's symptoms will also improve, including less nocturia, less dribbling, and stronger urinary stream.

All of the following infections are reportable diseases except: A) Lyme disease B) Gonorrhea C) Nongonococcal urethritis D) Syphilis

C) Nongonococcal urethritis The public health department requires all agencies to report Lyme disease, gonorrhea, and syphilis. Statistics of these reportable diseases are kept in each state.

All of the following services are covered under Medicare Part A except: A) Inpatienthospitalizations B) Medicines administered to a patient while hospitalized C) Nursing home care D) Surgeons

C) Nursing home care Medicare A coverage includes inpatient hospitalization and skilled care given in a certified skilled nursing facility. Most nursing home care is custodial care (help with bathing, dressing, using a bathroom, and eating). This care is not covered by Medicare A.

All of the following patients have an increased risk of developing adverse effects from Metformin (Glucophage) except: A) Patients with renal disease B) Patients with hypoxia C) Obese patients D) Patients who are alcoholics

C) Obese patients Metformin is used as a hyperglycemic agent for NIDDM. However, increased mortality has been associated with drug-induced lactic aci- dosis. Contraindications for the use of metformin include patients with significant renal and hepatic disease, alcoholism, and conditions associated with hypoxia (cardiac/pulmonary problems), sepsis, dehydration, high doses, and advanced age. Metformin is used in obese patients and education regarding the lactic acidosis risk should be discussed with the patient and family.

A neighbor's 14-year-old son, who is active in basketball, complains of pain and swelling on both knees. On physical exam, there is tenderness over the tibial tuberosity of both knees. Which of the following is most likely? A) Chondromalacia patella B) Left knee sprain C) Osgood-Schlatter disease D) Tear of the medial ligament

C) Osgood-Schlatter disease Osgood-Schlatter disease is characterized by bilateral pain over the tibial tuberosity upon palpation, along with knee pain and edema with exercise.

A 15-year-old basketball player who is 6 ft tall is seen for complaints of painful lumps on his knees. Upon inspection, the nurse practitioner notes a bonelike growth on the upper tibia midline below the kneecap on both knees. The patient has full range of motion with no joint tenderness, redness, or swelling. Which of the following conditions is best described? A) Osteosarcoma of the tibia B) Juvenile rheumatoid arthritis C) Osgood- Schlatter disease D) Paget's disease of the bone

C) Osgood-Schlatter disease Osgood-Schlatter disease is characterized by having pain over the tibial tuberosity with palpation of a bony mass over the anterior tubercle of one or both knees. Exercise worsens the pain

Kyphosis is a late sign of: A) Old age B) Osteopenia C) Osteoporosis D) Osteoarthritis

C) Osteoporosis Kyphosis is a curvature of the spine that causes a rounding of the back, which leads to a slouching posture. Severe thinning of the bones (osteoporosis) contributes to this curvature in the spine. Symptoms that may occur with severe cases of kyphosis include difficulty breathing, fatigue, and back pain.

During the eye exam of a 50-year-old hypertensive patient who is complaining of an onset of a severe headache, you find that the borders of the disc margins on both eyes are blurred. What is the name of this clinical finding? A) Normal optic disc B) Optic neuropathy C) Papilledema D) Hypertensive retinopathy

C) Papilledema Papilledema is defined as having optic disc swelling that is caused by increased intracranial pressure. It is commonly seen in both eyes and can develop over a time period of hours or weeks. Headache is a common complaint.

Acute bronchitis is best characterized by: A) Fever and wheezing B) Purulent sputum and fever C) Paroxysms of coughing that is dry or productive of mucoid sputum D) A gradual onset and fatigue

C) Paroxysms of coughing that is dry or productive of mucoid sputum Acute bronchitis is characterized by spastic cough that is dry or productive with mucus.

The Health Insurance Portability and Accountability Act (HIPAA) was passed by Congress in 2003. All of the following statements about HIPAA are correct except: A) It provides federal protections for personal health information B) It is applicable to all health care providers and payers who bill electronically and transmit health information over the Internet C) Patients have the right to view their mental health and psychotherapy-related health information D) It gives patients the right to view and correct errors in their medical records

C) Patients have the right to view their mental health and psychotherapy-related health information Mental health and psychotherapy/psychiatric records do not have to be released to patients even if they request those records. Otherwise, any type of medical records can be released if it is requested by the health insurance or health plan for billing purposes and reimbursement. HIPAA applies to all health care providers, health plans, health insurance companies, medical clearinghouses, and others who bill electronically and transmit health information over the Internet.

A 35-year-old male has a history of an upper respiratory viral infection 4 weeks ago. He reports that he started feeling short of breath and now complains of sharp pain in the middle of his chest that seems to worsen when he lies down. The patient's physical exam is within normal limits with the exception of a precordial rub on auscultation. The most likely diagnosis would be: A) Pulmonary embolism B) Dissecting aneurysm C) Pericarditis D) Esophageal reflux

C) Pericarditis Pericarditis is inflammation of the sac around the heart. Common signs/symptoms include chest pain over the center/left side of the chest; shortness of breath, especially with lying down; low-grade fever; weakness; fatigue; dry cough; and abdominal or leg swelling. Pericardial rub may be auscultated.

The nurse practitioner orders an ankle-brachial index (ABI) test for a patient. Which of the following disorders is the ABI test used for? A) Venous insufficiency B) Osteoarthritis of the arm or the ankle C) Peripheral arterial disease (PAD) D) Rheumatoid arthritis

C) Peripheral arterial disease (PAD) The ankle-brachial index (ABI) is a test that is used to stratify the severity of arterial blockage in the lower extremities for patients with peripheral arterial disease (PAD). An ABI score of 1.0 to 1.4 is normal. Any value less than 1.0 is abnormal. A score of 0.5 or less is indicative of severe PAD.

All of the following conditions are contraindications for bupropion (Wellbutrin, Zyban) except: A) Anorexia nervosa and bulimia B) Seizure disorders C) Peripheral neuropathy D) Within 14 days after discontinuation of a monoamine oxidase inhibitor (MAOI)

C) Peripheral neuropathy Bupropion increases the risk of seizures. Contraindications are seizures, anorexia nervosa, and bulimia. Avoid with any condition that increases seizures, such as after abrupt withdrawal of alcohol or sedatives and certain head injuries. For peripheral neuropathy, atypical antide- pressants such as duloxetine (Cymbalta) and TCAs such as amitriptyline (Elavil) are used.

A 25-year-old male who was involved in a car accident is brought in to the local emergency department. He reports wearing a seat belt and was the driver of the vehicle. The patient is complaining of pain on his right leg. The skin is intact, but the right foot is abducted slightly. The right knee is tender and swollen. Which of the following tests is/are the best choice for evaluating for possible fractures and joint damage? A) Plain radiographs of the right hip and knee B) Ultrasound with a CT scan of the right leg C) Plain radiograph of the right hip and leg with an MRI of the knee joint D) Radiograph of the right hip and knee with special view of the hip

C) Plain radiograph of the right hip and leg with an MRI of the knee joint The stem is asking for two things " . . . evaluating for possible fractures joint damage." Plain x-rays are the first exam ordered for suspected or obvious bony fractures. An MRI scan is the best test for any type of joint pathology (swollen right knee).

Multiple myeloma is a malignancy of the: A) White blood cells (WBC) B) Red blood cells (RBC) C) Plasma cells D) Platelets

C) Plasma cells Myeloma is a cancer of the plasma cells (or mature B-cells/ lymphocytes), which affects the bone marrow. Plasma cells produce antibodies and reside mainly in the bone marrow. Signs/symptoms are bone pain, fractures, hypercalcemia, depressed immunity, and anemia. The bone marrow produces WBCs (neutrophils, lymphocytes, eosinophils, basophils), RBCs, and platelets. The typical patient is an older adult who is age 60 years or older.

Mary, who is 65 years of age, comes into the clinic during the first week of November for her annual wellness visit. Her last Td booster was 9 years ago. Which immunization(s) would you recommend for this visit? A) Influenza vaccine B) Tetanus and influenza vaccine C) Pneumococcal (Pneumovax) and influenza vaccines D) She does not need any vaccinations to be administered in this visit

C) Pneumococcal (Pneumovax) and influenza vaccines At 65 years of age, Pneumovax is recommended. Influenza is recommended annually. Td is recommended every 10 years.

Which of the following diseases is associated with a high risk of giant cell arteritis? A) History of transient ischemia attacks (TIA) B) Frequent migraine headaches with focal neurologic findings C) Polymyalgia rheumatica D) Systemic lupus erythematosus (SLE)

C) Polymyalgia rheumatica Giant cell arteritis (also known as temporal arteritis) is more common among patients with PMR. It can cause permanent blindness if it is not treated. PMR patients are taught how to recognize it. The quick onset of vision loss in one eye, accompanied by a tender indurated artery and scalp tenderness on the same side, are classic symptoms. The screening test is the sedimentation rate and C-reactive protein test. Both will be markedly elevated.

A 76-year-old woman reports that for the previous 4 months, she has noticed severe stiffness and aching in her neck and both shoulders and hips that is worsened by movement. She reports having a difficult time getting out of bed because of the severe stiffness and pain. It is difficult for her to put on a jacket or blouse or to fasten her bra. Along with these symptoms, she also has a low-grade fever, fatigue, loss of appetite, and weight loss. Starting yesterday, the vision in her right eye has progressively worsened. She has annual eye exams and denies that she has glaucoma. Which of the following conditions is most likely? A) Rheumatoid arthritis (RA) B) Degenerative joint disease C) Polymyalgia rheumatica (PMR) D) Fibromyalgia

C) Polymyalgia rheumatica(PMR) Polymyalgia rheumatica (PMR )is associated with a high risk of giant cell arteritis (GCA) or temporal arteritis (15%-30%). The new onset of vision loss and the location of the pain (neck, both shoulders/hips) are the most important clues. PMR is a rheumatic condition that involves joints and the arteries. Temporal arteritis can cause permanent blindness. The sedimentation rate is very high (40 mm/hr to 100 mm/hr). Almost all will have elevated C-reactive protein levels (up to 99%). These patients are managed by rheumatologists via long-term steroids.

A 25-year-old woman's last menstrual period was 6 weeks ago. She is complaining of nausea with vomiting in the morning and fatigue. Her breasts feel bloated. The nurse practitioner suspects that she is pregnant. Her symptoms would be considered: A) Positive signs of pregnancy B) Probable signs of pregnancy C) Presumptive signs of pregnancy D) Possible signs of pregnancy

C) Presumptive signs of pregnancy Presumptive signs of pregnancy are symptoms experienced by the woman, such as amenorrhea, breast tenderness, nausea/ vomiting, fatigue, and increased urinary frequency.

Which of the following is responsible for the symptoms of dysmenorrhea? A) Estrogen B) Human chorionic gonadotropin C) Prostaglandins D) Progesterone

C) Prostaglandins Prostaglandins are hormones the body produces prior to menses; they eventually cause the uterus to contract to shed the endometrial lining. Contractions cause pain. The greater the amount of prostaglandins that are released, the more pain one will experience. Contractions of the uterus cause vasoconstriction of blood supply to the uterus, which in turn will cause pain.

Which of the following pathogenic bacteria are commonly found in the lungs of older children and adults with cystic fibrosis? A) Streptoccocus pneumoniae B) Chlamydia pneumoniae C) Pseudomonas aeruginosa D) Staphylococcus aureus

C) Pseudomonas aeruginosa Pseudomonas aeruginosa is a common chronic lung infection seen in older children and adults with cystic fibrosis. This infection is difficult to get rid of and sometimes is the cause of death.

What is the least common pathogen found in community-acquired atypical pneumonia? A) Moxarella catarrhalis B) Streptococcus pneumoniae C) Pseudomonas aeruginosa D) Mycoplasma pneumonia

C) Pseudomonas aeruginosa Pseudomonas aeruginosa is an uncommon cause of CAP but is a particularly difficult bacterial infection to treat.

Which of the following conditions are possible causes of secondary hypertension? A) Leukemia and thalassemia major B) Hashimoto's thyroiditis and polycystic ovaries C) Renal stenosis and adrenal tumors D) Myocardial infarction and coronary artery disease

C) Renal stenosis and adrenal tumors Secondary hypertension is most likely seen following renal stenosis and adrenal tumors. Renal stenosis causes secondary hypertension by plaque formation in the arteries, causing damage to coronary arteries (atherosclerosis). Adrenal tumors will initiate secondary hypertension by releasing a large amount of aldosterone, which will cause water and salt retention and loss of too much potassium.

A 13-year-old boy wants to be treated for his acne. He has a large number of closed and open comedones on his face. The patient has been treating himself with OTC benzoyl peroxide and salicylic acid topical products. Which of the following would be recommended next? A) Retinoic acid (Accutane) B) Tetracycline C) Retin A 0.25% gel D) Wash face carefully with medicated soap at bedtime

C) Retin A 0.25% gel Topical agents are the first-line treatment for acne vulgaris. Retin A 0.25% gel would be the next step. Oral preparations would then be offered for the next step (tetracycline), and Accutane would be the final step of therapy.

While checking for the red reflex on a 3-year-old boy during a well child visit, a white reflection is seen on the child's left pupil. Which of the following conditions should be ruled out? A) Unilateral strabismus B) Unilateral cataracts C) Retinoblastoma of the left eye D) Color blindness of the left eye

C) Retinoblastoma of the left eye Retinoblastoma is a rare type of cancer in which a cancerous tumor of the retina is present. This is diagnosed by noting a pupil that appears white or has white spots on it. One or both eyes may be affected. It is often seen in photographs, where there will be a white glow in the eye instead of the usual "red eye" noted in pictures from the flash.

A new patient who recently visited a relative in North Carolina complains of an onset of fever and red rashes that started 2 days ago. The rash first appeared on the wrist and the ankles and included the palms of the hands. The patient reports that it is spreading toward his trunk. The patient's eyes are not injected and no enlarged nodes are palpated on his neck. There is no desquamation of the skin. Which of the following is most likely? A) Kawasaki's disease B) Meningococcemia C) Rocky Mountain spotted fever D) Measles

C) Rocky Mountain spotted fever Given the location in the East, Rocky Mountain spotted fever is most likely the infection causing symptoms of fever, rash on ankles/wrists moving to the palms of the hands and the trunk. Rocky Mountain spotted fever is a tickborne disease caused by the bacterium Rickettsia rickettsii, which is transmitted by a bit from an infected tick.

The following statements about benign prostatic hypertrophy are correct except: A) It is seen in up to 50% of males older than 50 B) Dribbling and nocturia are common patient complaints C) Saw palmetto is always effective in reducing symptoms D) The PSA value is usually slightly elevated

C) Saw palmetto is always effective in reducing symptoms Research does not support the theory that saw palmetto reduces the symptoms of benign prostatic hyperplasia in all cases.

In most states, patients younger than age 18 years may consent to healthcare without parental or legal guardian consent except for which of the following? A) Contraception B) Pregnancy C) School physicals D) STD evaluation and treatment

C) School physicals Any student younger than 18 years of age must have paren- tal permission to have a school physical exam done

Which of the following tests would you order for an older diabetic male with the following CBC results? Hb 11 g/dL, Hct 38%, and an MCV 105 fl. His reticulocyte count is normal. A) Serum ferritin and a peripheral smear B) Hemoglobin electrophoresis C) Serum folate acid and B12 level D) Schilling test

C) Serum folate acid and B12 level Serum folate acid and B12 levels would be ordered to evaluate him for folic acid deficiency anemia. Many patients who are deficient in folic acid are also deficient in B12.

A 48-year-old woman is told by a physician that she is starting menopause. All of the following are possible findings except: A) Hot flashes B) Irregular menstrual periods C) Severe vaginal atrophic changes D) Cyclic mood swings

C) Severe vaginal atrophic changes As women reach menopause, changes that may occur include hot flashes, irregular menstrual periods, and cyclic mood swings. Vaginal changes, such as dryness and thinning, may also begin to occur

Mary Walker is a 16-year-old who presents in the clinic for a physical exam. She tells the triage nurse that she would like contraception as well. How should the nurse practitioner proceed? A) Refuse to see the patient until consent can be obtained from her parent or legal guardian B) Perform a physical exam and discuss contraceptive options C) Speak with the patient about contraception and have her obtain parental consent for the physical exam D) Have the nurse discuss contraception with the patient

C) Speak with the patient about contraception and have her obtain parental consent for the physical exam In the United States, state laws and policies may vary regarding contraceptive counseling and treatment for anyone under the age of 18 years. It is required that you be familiar with the state laws. In all states, anyone under the age of 18 years must have parental consent for the physical exam.

You are reviewing a Pap smear report on a 25-year-old female. Which of the following cells should be on a Pap smear to be classified as a satisfactory specimen? A) Clue cells and endometrial cells B) Vaginal cells and cervical cells C) Squamous epithelial cells and endocervical cells D) Leukocytes and RBCs

C) Squamous epithelial cells and endocervical cells Squamous epithelial cells and endocervical cells must be obtained when performing a Pap smear to be considered satisfactory to evaluate the cells from the endocervix.

Pulsus paradoxus is more likely to be associated with: A) Sarcoidosis B) Acute bronchitis C) Status asthmaticus D) Bacterial pneumonia

C) Status asthmaticus Pulsus paradoxus is most likely to be seen with status asthmaticus. With inspiration, systolic pressure drops due to the increased pres- sure (positive pressure). Some pulmonary risks of having increased pressure include asthma and emphysema. Cardiac causes for pulsus paradoxus include tamponade, pericarditis, and cardiac effusion.

Which of the following T-scores is indicative of osteoporosis? A) T-score of 0 to -1.0 B) T-score of -1.0 to -2.0 C) T-score of -2.5 or less D) The diagnosis of osteoporosis is based on an x-ray series of the spine

C) T-score of -2.5 or less Osteoporosis is defined as having a T-score of -2.5 or less. Risk factors should also be considered when interpreting T-scores for diagnosis.

Carol, a 73-year-old patient, complains of episodic vertigo, slight confusion, and weakness that last nearly an hour each time. Movement does not worsen the vertigo. She "rests" and her symptoms subside, but she is puzzled because the weakness "jumps from side to side," sometimes on the right and sometimes on the left of her body. Her symptoms suggest: A) Benign paroxysmal positional vertigo B) Ménière's disease C) TIA (transient ischemic attack) D) CVA (cerebrovascular accident)

C) TIA (transient ischemic attack) Transient ischemic attack (TIA) is caused by vascular occlusion. Symptoms of a TIA usually last less than 1 hour; however, the symptoms may also be permanent. Common signs/symptoms include neurologic deficits, vertigo, confusion, weakness, hemiparesis, temporary monocular blindness, ataxia, and diplopia.

During a sports physical of a 14-year-old female, you note her breast development. The areola and the breast tissue are all in one mound. In which Tanner stage is this patient? A) Tanner Stage I B) Tanner Stage II C) Tanner Stage III D) Tanner Stage IV

C) Tanner Stage III Tanner stages for breast development:I , prepuberty; II, breast bud; III, breast and areola one mound; IV, breast and areola secondary mound; V, adult pattern.

What Tanner stage is a girl at when her breasts form a secondary mound? A) Tanner Stage II B) Tanner Stage III C) Tanner Stage IV D) Tanner Stage V

C) Tanner Stage IV Tanner stages for breast development: I, prepuberty; II, breast bud; III, breast and areola one mound; IV, breast and areola secondary mound; V, adult pattern.

Swim therapy for a 13-year-old with cerebral palsy is an example of: A) Primary prevention B) Secondary prevention C) Tertiary prevention D) Health prevention

C) Tertiary prevention Tertiary prevention is any type of rehabilitation for a particular condition. Examples include physical rehab (swimming), cardiac rehab, and/or speech therapy.

Spermatogenesis occurs at the: A) Vas deferens B) Seminal vesicles C) Testes D) Epididymis

C) Testes Spermatogenesis occurs in the testes, taking approximately 3 months to develop.

A 10-year-old boy complains of a sudden onset of scrotal pain when he woke up that morning. He is also complaining of severe nausea and vomiting. During the physical examination, the nurse practitioner finds a tender, warm, and swollen left scrotum. The cremasteric reflex is negative and the urine dipstick is negative for leukocytes, nitrites, and blood. The most likely diagnosis is: A) Acute epididymitis B) A severe salmonella infection C) Testicular torsion D) Acute orchitis

C) Testicular torsion Testicular torsion signs and symptoms include sudden onset of unilateral scrotal pain, nausea, vomiting, and abdominal pain. Acute epididymitis causes fever, chills, nausea, and unilateral pain and is most com- monly seen in sexually active men. Unilateral scrotal pain does not occur with Salmonella infection. Acute orchitis is often based on having a recent mumps infec- tion or parotitis with testicular edema.

Which of the following methods is used to diagnose gonorrheal pharyngitis or proctitis? A) Serum chlamydia titer B) Gen-Probe C) Thayer-Martin culture D) Culture and sensitivity of the purulent discharge

C) Thayer-Martin culture Thayer-Martin culture is recommended for screening to diagnose gonorrheal pharyngitis or proctitis.

Candidal intertrigo is the name for an infection that is caused by the yeast Candida albicans. What is the location of this type of candida infection? A) The scalp B) The flexor areas of the elbows and the knees C) The areas of the body where skin is rubbing together, such as under the breast or in the groin area D) The hands

C) The areas of the body where skin is rubbing together, such as under the breast or in the groin area Candidal intertrigo infections are more common in the obese and in women with pendulous breasts. It is found in areas where skin rubs against skin (under breasts, in the groin area, and on stomach folds in the obese). It is more common in warm and humid weather (summer).

All of the following are correct statements regarding the Healthy People 2020 objectives except: A) The document's objectives are not only applicable nationally, but also internationally B) One of the objectives of the document is to help people achieve physical health C) The document's objectives are applicable only to people of the United States of America D) The document is formulated by a special committee formed by an alliance of all the state health departments

C) The document's objectives are applicable only to people of the United States of America Healthy People 2020 was written for all people

Some nurse practitioners bill directly for their services. Regarding reimbursement, who is considered a third-party payer? A) The patient B) The health care provider C) The health insurance companies, health plans, Medicare and Medicaid D) The federal government

C) The health insurance companies, health plans, Medicare and Medicaid Third- party payers are health insurance companies, health plans (HMO or PPO), Medicare and Medicaid. The "first party" is the patient. The "second party" is the health care provider

All of the following are correct statements regarding oral contraceptives except: A) The actual failure rate of oral contraceptives is 3% B) Desogestrel belongs to the progesterone family of drugs C) The newer low-dose birth control pills do not require back-up during the first 2 weeks of use D) Oral contraceptives are contraindicated for women 35 years of age or older who smoke

C) The newer low-dose birth control pills do not require backup during the first 2 weeks of use Low-dose OCPs require a second backup method of contraception during the first 2 weeks of use.

A 19-year-old student who is on a prescription of Triphasil is being seen for an annual gynecologic exam in the college health center. The nurse practitioner has obtained the Pap smear and is about to perform the bimanual exam. She gently removes the plastic speculum from the vagina. While the NP is performing the bimanual vaginal exam, the patient complains of slight discomfort during deep palpation of the ovaries. Which of the following is a true statement? A) The uterus and the ovaries are both very sensitive to any type of palpation B) The fallopian tubes and ovaries are not sensitive to light or deep palpation C) The ovaries are sensitive to deep palpation but they should not be painful D) The uterus and the ovaries are not important organs of reproduction

C) The ovaries are sensitive to deep palpation but they should not be painful The ovaries are usually slightly sensitive to deep palpation, but they should not be painful. Unilateral adnexal pain accompanied by cervical motion tenderness and purulent endocervical discharge is suggestive of PID.

A sexually active 22-year-old man is asking to be screened for hepatitis B because his new girlfriend has recently been diagnosed with hepatitis B infection. His lab results are the following: anti-HBV is negative, HBsAg is positive, and HBeAg is negative. Which of the following is indicated? A) The patient is immune to the hepatitis B virus B) The patient is not infected with hepatitis B virus C) The patient needs hepatitis B vaccine and hepatitis B immunoglobulin D) The patient needs only hepatitis B immunoglobulin

C) The patient needs hepatitis B vaccine and hepatitis B immunoglobu- lin Because he is HBsAg positive, and anti-HBV negative and HBeAg negative, he needs hepatitis B immunoglobulin and hepatitis B vaccine.

You suspect an enterobiasis infection in a 6-year-old girl. Which of the following tests would you recommend? A) Stool culture and sensitivity B) Stool for ova and parasites C) The scotch tape test D) A Hemoccult test

C) The scotch tape test Enterobiasis infection (pinworms) is caused by small worms that infect the intestines. Symptoms include itching around the anus, which is usually worse at night. The scotch tape test is done by applying the scotch tape on the anal area in the morning; the worms commonly come out at night and will stick to the tape, which is used for diagnosis.

All of the following are false statements regarding acute gastritis except: A) Chronic intake of nonsteroidal anti-inflammatory drugs (NSAIDs) can cause the disorder B) Chronic lack of dietary fiber is the main cause of the disorder C) The screening test for the disorder is the barium swallow test D) The gold standard to evaluate the disorder is a colonoscopy

C) The screening test for the disorder is the barium swallow test The gold standard for diagnosing gastric disease is biopsy of gastric and/or duodenal tissue by upper endoscopy. Chronic use of NSAIDs disrupts the production of prostaglandins, which decreases blood supply and then breaks down the protective layer of the mucosa, which leads to gastric disease.

A 13-year-old male is brought in by his mother for a physical exam. During the genital exam, the nurse practitioner notices that the patient is at Tanner Stage II. Which of the following is the best description of this Tanner stage? A) The penis is growing more in length than in width and the testicles become larger with darker scrotal skin and the pubic hair is starting to curl B) The penis is growing more in width than in length with darker scrotal skin and more numerous pubic hairs that are darker, curly, and coarser C) The testicles become larger and the skin of the scrotum starts to become darker with straight, fine, countable hairs on the genitals and the axilla D) The testicles, penile width and length are all developing quickly

C) The testicles become larger and the skin of the scrotum starts to become darker with straight, fine, countable hairs on the genitals and the axilla Tanner Stage II is when the testicles start to grow. The scrotal skin becomes thicker and starts to get darker (hyperpigmentation). The pubic hair is of a fine texture and straight and there are few countable hairs on the genitals and the axilla.

All of the following are physiologic changes that occur in the body as we age except: A) The half-life of some drugs is prolonged B) There is an increase in cholesterol production by the liver C) There is a mild increase in renal function D) There is a slight decrease in the activity of the immune system

C) There is a mild increase in renal function Physiologic changes that occur in the elderly include decrease in renal function, increased half-life of some medica- tions, increase in cholesterol production by the liver, and slight decrease in the immune system.

All of the following are correct statements regarding physiologic changes found in the elderly with the exception of: A) There is an increase in the fat-to-lean body ratio B) There is a decrease in the ability of the liver to metabolize drugs C) There is an increase in renal function D) Loss of hearing for sounds in the high-frequency range (presbycusis)

C) There is an increase in renal function Physiological changes in the elderly include increase in the fat-to-lean body ratio, decrease in the ability of the liver to metabolize drugs, loss of hearing for sounds in the high-frequency range, and a decrease in renal function.

All of the following drug classes are approved for treating hypertension. Which of the following antihypertensive drug classes is associated with the largest number of research studies? A) ACE inhibitors B) Angiotensive receptor blockers (ARBs) C) Thiazide diuretics D) Calcium channel blockers (CCBs)

C) Thiazide diuretics Thiazide diuretics have been used to treat hypertension for many decades and numerous placebo-controlled studies have documented their effectiveness as an antihypertensive drug class.

During a sports physical exam of a 14-year-old high school athlete, the nurse practitioner notices a split of the S2 component of the heart sound during deep inspiration. She notes that it disappears upon expiration. The heart rate is regular and no murmurs are auscultated. Which of the following is correct? A) This is an abnormal finding and should be evaluated further by a cardiologist B) A stress test should be ordered C) This is a normal finding in some young athletes D) An echocardiogram should be ordered

C) This is a normal finding in some young athletes It is common to hear a split of the S2 over the pulmonic area of the heart with inspiration as long as it disappears with expiration, with no other abnormal symptoms. This is caused by split- ting of the aortic and pulmonic components.

An elderly patient with a productive cough and fever is diagnosed with pneumonia. All of the following organisms are capable of causing community-acquired pneumonia except: A) Haemophilus influenzae B) Mycoplasma pneumoniae C) Treponema pallidum D) Streptococcus pneumoniae

C) Treponema pallidum Treponema pallidum is a gram-negative spirochete bacterium that causes syphilis

Which of the following benzodiazepines has the shortest half-life? A) Lorazepam (Ativan) B) Alprazolam (Xanax) C) Triazolam (Halcion) D) Clonazepam (Klonopin)

C) Triazolam (Halcion) Triazolam (Halcion) has an average half-life of about 2 hours. Xanax has a half-life of 12 hours. Ativan has a half-life of 15 hours. Klonopin has a half-life of 34 hours

Julia, a 16-year-old patient, is being treated for her first urinary tract infection. Julia had an allergic reaction with hives after taking sulfa as a child. Which of the following antibiotics would be contraindicated? A) Cephalexin (Keflex) B) Ampicillin (Amoxil) C) Trimethoprim-sulfamethoxazole (Bactrim) D) Nitrofurantoin crystals (Macrobid)

C) Trimethoprim-sulfamethoxazole (Bactrim) With the allergic history to sulfa drugs, it would be safest to avoid Bactrim.

A sexually active woman is complaining of amenorrhea and vaginal spotting. On exam, her left adnexa is tender and cervical motion tenderness is positive. Which test should the nurse practitioner initially order? A) Flat plate of the abdomen B) Complete blood count (CBC) with white cell differentials C) Urine pregnancy test D) Pelvic ultrasound

C) Urine pregnancy test A urine pregnancy test should be performed to rule out miscarriage and/or tubal pregnancy.

In small children with acquired immunodeficiency syndrome (AIDS), which of the following vaccines is contraindicated? A) Diphtheria and tetanus (Td) B) Hepatitis B and mumps C) Varicella D) Td and oral polio

C) Varicella The data regarding efficacy of the varicella vaccine are insufficient; therefore, varicella vaccine is contraindicated in HIV-infected individuals.

A mother brings in her 4-year-old daughter who just started attending preschool. She tells the nurse practitioner that her child is complaining of burning and itching in both eyes along with a runny nose. During the physical exam, the child's eyes appear injected bilaterally. The throat is red and her inferior nasal turbinates are swollen. Which of the following is most likely? A) Herpes keratitis B) Corneal ulcer C) Viral conjunctivitis D) Bacterial conjunctivitis

C) Viral conjunctivitis Viral conjunctivitis is inflammation of the conjunctiva with common complaints of red, itchy eyes, without signs of infection.

Symptoms suggestive of ulcerative colitis include all of the following except: A) Bloody diarrhea mixed with mucus B) Nausea and vomiting C) Weight gain D) Abdominal pain

C) Weight gain Symptoms of ulcerative colitis includes bloody diarrhea mixed with mucus, nausea/vomiting, abdominal pain, and possible weight loss with long-term diarrhea

Physiologic anemia of pregnancy is due to: A) An increase in the cardiac output at the end of the second trimester B) A physiologic decrease in the production of RBCs in pregnant women C) An increase of up to 50% of the plasma volume in pregnant women D) An increase in the need for dietary iron in pregnancy

C)An increase of up to 50% of the plasma volume in pregnant women Physiologic anemia of pregnancy is caused by the increased volume of plasma during pregnancy when compared to the production of RBCs.

A college student has recently been informed that he has an HPV (human papilloma virus) infection on the shaft of his penis. Which of the following may reveal subclinical lesions on the penile skin? A) Perform a KOH (potassium hydroxide) exam B) Scrape off some of the affected skin and send it for a culture and sensitivity C) Apply acetic acid to the penile shaft and look for acetowhite changes D) Order a serum herpes virus titer

C)Apply acetic acid to the penile shaft and look for acetowhite changes Lesions of HPV infection will turn white with application of acetic acid.

Which of the following is indicated for the prophylactic treatment of migraine headache? A) Ibuprofen (Motrin) B) Naproxen sodium (Anaprox) C) Propranolol (Inderal) D) Sumatriptan (Imitrex)

C)Propranolol (Inderal) Medications used to prophylactically prevent migraine headaches include beta-blockers (propranolol) and tricyclic antidepressants (ami- triptyline). Motrin, Anaprox, and Imitrex are all medications used to treat migraine headache.

When a domestic dog is suspected to be infected with the rabies virus, it can either be killed for a brain biopsy or it can be quarantined. What is the minimum number of days that a dog suspected of rabies must be quarantined? A) 4 weeks B) 21 days C) 14 days D) 10 days

D) 10 days The minimum number of days to quarantine an animal suspected of rabies is 10 days. If the animal is healthy and has no symptoms of rabies at 10 days, it is not infected with the rabies virus and can be returned to the owner.

The earliest age that an MMR can be administered is at: A) 4 months B) 6 months C) 8 months D) 12 months

D) 12 months The earliest age that MMR is recommended is 12 months. This age is recommended because giving it any earlier may be less effective because the infant still has antibodies from the mother. Antibodies still present from the mother may interfere with the production of the antibodies stimulated by the MMR vaccine.

All of the following are considered emancipated minors except: A) 15-year-old male who is married B) 14-year-old female who is a single parent C) 17-year-old male who is enlisted in the U.S. Army D) 13-year-old being treated for a sexually transmitted disease

D) 13-year-old being treated for a sexually transmitted disease There are three primary ways for a minor to become emancipated: marriage, court order, and military service.

At what age can a child copy a cross and ride a bicycle? A) 1 year B) 2 years C) 3 years D) 4 years

D) 4 years Developmental stages in children include the following: 1 year: walk; 2 years: walks up steps with the same foot; 3 years: pedals a tricycle and copies a circle; 4 years: rides a bicycle and copies a cross and draws a person with 2 parts.

At what age can a child ride a bicycle? A) 2 to 3 years of age B) 3 to 4 years of age C) 4 to 5 years of age D) 5 to 6 years of age

D) 5 to 6 years of age At about the age of 5 to 6 years, most children can ride a bicycle with training wheels. Helmets should always be worn (primary prevention).

All of the following do not require parental consent to be obtained by the nurse practitioner except: A) A 17-year-old who wants to be treated for a sexually transmitted infection B) A 12-year-old who wants a serum pregnancy test C) A 15-year-old who wants birth control pills D) A 14-year-old who wants to be treated for dysmenorrhea

D) A 14-year-old who wants to be treated for dysmenorrhea Treatment for teenagers may be done without parental consent for sexually transmitted infections, pregnancy testing, contraception counseling and treatment. Parental consent is required for any type of physical exam for other problems

All of the following patients are at higher risk for suicide except: A) A 66-year-old White male whose wife of 40 years recently died B) A high school student with a history of bipolar disorder C) A depressed 45-year-old female with family history of suicide D) A 17-year-old teen who has only 1 close friend in school

D) A 17-year-old teen who has only one close friend in school Risk factors for suicide include: 1) elderly white males (especially after the death of a spouse); 2) past history of suicide; 3) family history of suicide; 4) plans for use of a lethal weapon such as a gun or knife; 5) female gender has a higher attempt rate, but males have a higher success rate; 6) personal history of bipolar disorder or depression.

All of the following patients should be screened for diabetes mellitus except: A) An obese man of Hispanic descent B) An overweight middle-aged Black woman whose mother has type 2 diabetes C) A woman who delivered an infant weighing 9.5 lb D) A 30-year-old White man with hypertension

D) A 30-year-old White man with hypertension The 30-year-old White man with hypertension would be the last patient to be screened for diabetes. Not having any information about him also puts him lower on the list. Obesity, ethnicity (Hispanic/Latino Americans, African Americans, Native Americans, Asian Americans, Pacific Islanders, and Alaska natives), family history of diabetes, and gestational diabetes (mother of the infant weighing 9.5 lbs) are all risk factors. These were present in all of the other selections. Other risk factors for diabetes include impaired glucose tolerance test, sedentary lifestyle, PCOS, and hypertension.

Which of the following situations is considered emergent? A) A laceration on the lower leg of a patient on aspirin (Bayer) 81mg every other day B) Rapid breathing and tachycardia in a patient with a fever C) An elderly man with abdominal pain whose vital signs appear stable D) A 37-year-old male biker with a concussion due to a fall who appears slightly agitated and does not appear to understand instructions given by the medical assistant checking his vital signs

D) A 37-year-old male biker with a concussion due to a fall who appears slightly agitated and does not appear to understand instructions given by the medical assistant checking his vital signs The biker who had the concussion is the emergent situation due to his agitation and his inability to follow directions, which could mean he has some type of brain trauma. The laceration can be treated with pressure to stop the bleeding until help arrives. The man with abdominal pain has normal vital signs. Tachycardia is common with fever.

What is the pedigree symbol for a diseased or affected female? A) An empty square B) An empty circle C) A filled-in square D) A filled-in circle

D) A filled-in circle A filled-in circle is a diseased/affected female and an empty circle is a healthy female. A tip to remember is that females make eggs (or follicles), which resemble a circle. By default, the square symbol is the male.

Treatment for mild preeclampsia includes all of the following except: A) Bed rest except for bathroom privileges B) Close monitoring of weight and blood pressure C) Close follow-up of urinary protein, serum creatinine, and platelet count D) A prescription of methyldopa (Aldomet) to control blood pressure

D) A prescription of methyldopa (Aldomet) to control blood pressure Recommended care for women diagnosed with preeclampsia includes bed rest with bath- room privileges, weight and BP monitoring, and closely following urine protein and serum protein, creatinine, and platelet counts. Oral medications are not used as first-line treatment.

The signs and symptoms of dementia may include all of the following except: A) Personality changes B) Difficulty in verbalizing C) Difficulty in recognizing familiar objects D) Abstract thinking ability is increased

D) Abstract thinking ability is increased Characteristics of dementia include irreversible symptoms with a gradual onset. Short-term memory loss is an early sign of dementia. As symptoms progress, the patient may become incoherent, unable to talk, walk, feed self, or perform self-care.

You would associate a positive iliopsoas muscle test result with: A) Left cerebral vascular accident B) Urinary tract infection C) Heel fractures D) Acute abdomen

D) Acute abdomen A positive iliopsoas muscle test is seen with an acute abdomen. This test is performed by asking the patient to actively flex the thigh at the hip. A "positive psoas sign" is noted when the patient exhibits pain in the right lower quadrant due to the inflamed tissue. The right iliopsoas muscle lies under the appendix, so pain may suggest appendicitis.

A 35-year-old sexually active male presents with a 1-week history of fever and pain over the left scrotum. It is accompanied by frequency and dysuria. The scrotum is edematous and tender to touch. He denies flank pain, nausea, and vomiting. He reports that the pain is lessened when he uses scrotal support briefs. His urinalysis shows 2+ blood and a large number of leukocytes. What is the most likely diagnosis? A) Acute urinary tract infection B) Acute pyelonephritis C) Acute orchitis D) Acute epididymitis

D) Acute epididymitis Acute epididymitis is the infection presented here. Scrotal edema and pain with palpation do not occur in UTI or pyelonephritis. Acute orchitis symptoms include testicular pain and edema, are usually associated with the mumps, but do not have frequency and dysuria.

All of the following factors have been found to increase the risk of atrial fibrillation in predisposed individuals except: A) Hypertension B) Excessive alcohol intake in susceptible individuals C) Theophylline (Theodur) and pseudoephedrine (Sudafed) D) Acute esophagitis

D) Acute esophagitis Factors that influence the risk of atrial fibrillation include hypertension, excessive alcohol consumption, and medications such as Theodur and Sudafed.

Which of the following is most likely to cause delirium? A) Dehydration B) Multiple brain infarcts C) Malnutrition D) Acute infection

D) Acute infection Delirium is an acute decline in mental status and is temporary. Common causes are fever, shock, drugs, alcohol, severe dehydration, and acute infection.

Prophylaxis for Pneumocystis carinii pneumonia includes all of the following drugs except: A) Trimethoprim-sulfamethoxazole B) Dapsone C) Aerosolized pentamidine D) Aerosolized albuterol sulfate (Ventolin)

D) Aerosolized albuterol sulfate (Ventolin) Prophylaxis for Pneumocystis carinii pneumonia includes the use of Bactrim, dapsone, and aerosolized pentamidine. Aerosolized pentamidine is a antimicrobial treatment for prevention, along with Bactrim and dapsone, which are antibacterial medications.

All of the following are clinical signs and symptoms seen early in testicular torsion except: A) Nausea and vomiting B) Absence of the cremasteric reflex C) Affected testicle is elevated compared with the normal testicle D) Affected testicle is swollen and feels cold to touch

D) Affected testicle is swollen and feels cold to the touch The affected testicle will be swollen, but it will feel very warm to the touch. A cold testicle is abnormal and is indicative of gangrene (after 24 hours).

Which of the following is a true statement regarding the first-pass metabolism process? A) Drugs that are administered by intramuscular injection all go through the process of first-pass metabolism B) After being swallowed, oral drugs are absorbed by the GI tract and metabolized by the bacteria in the small intestines before being released into the general circulation C) Drugs administered through the skin (patches) are metabolized by the dermis of the skin D) After a drug is taken by the oral route, it is absorbed in the small intestines and enters the liver through the portal circulation, where it is metabolized before being released into the general circulation

D) After a drug is taken by the oral route, it is absorbed in the small intestines and enters the liver through the portal circulation, where it is metabolized before being released into the general circulation After drugs are taken by the oral route, they are absorbed by the small intestines and enter the liver through the portal circulation, where they are metabolized before being released into the general circulation. First-pass metabolism (first-pass effect) determines how much of the active drug is available to the body (bioavailability). Depending on the drug and other factors, a drug may be poorly metabolized or extensively metabolized by the liver.

Jim Wheeler is obese (BMI of 33), fatigued, and complaining of excessive thirst and hunger. You suspect type 2 diabetes mellitus. Initial testing to confirm diagnosis can include: A) Fasting plasma glucose level B) Glycosylated hemoglobin level (HbA1 c) C) Glucose tolerance testing D) All of the above

D) All of the above Type 2 diabetes mellitus screening tests include: fasting plasma glucose level (> 126 mg/dL), random plasma glucose level (> 200 mg/dL), and glucose tolerance testing (2 hr blood glucose level > 200 mg/dL) with 75 g glucose load. Normal HbA1c levels are < 6%

Carol M. is a 40-year-old bank teller who has recently been diagnosed with obsessive-compulsive disorder by her therapist. Her symptoms would include: A) Ritualistic behaviors that the patient feels compelled to repeat B) Attempts to ignore or suppress the repetitive behaviors, which increase anxiety C) Frequent intrusive and repetitive thoughts and impulses D) All of the above

D) All of the above Signs and symptoms of obsessive-compulsive disorder include ritualistic behaviors that are repeated, increased anxiety with attempting to ignore repetitive behaviors, and frequent intrusive and repetitive thoughts and impulses.

Ted, who is 15 years old, has just moved into the community and is staying in a foster home temporarily. There is no record of his immunizations. His foster mother wants him to be checked before he enters the local high school. Which of the fol- lowing does this patient need? A) Meningococcal B) MMR C) Tdap D) All of the above

D) All of the above The measles, mumps, and rubella (MMR) is recommended as one of the "catch-up" immunizations for this patient's age group. The tetanus immunization that is recommended as a "catch-up" for that age group is the Tdap instead of just the Td.

All of the following are covered under Medicare Part B except: A) Persons age 65 years or older B) Durable medical equipment C) Mammograms annually starting at age 50 D) Anesthesiologist's services

D) Anesthesiologist's services Medicare Part B covers: 1) outpatient physician visits, labs, x-rays; 2) durable medical equipment; 3) mammograms/colonoscopy after age 50 years annually; 4) rehabilitation. Anesthesiologist's services are covered by Medicare Part A.

Which of the following is used to confirm a diagnosis of Hashimoto's thyroiditis? A) Serum TSH B) Free T4 test C) Antimicrosomal antibody test D) Any of the above

D) Any of the above In addition to conducting a physical examination and taking a thorough history and symptoms into account, 1 or more laboratory tests are used to diagnose Hashimoto's thyroiditis. The 3 most common diagnostic tests that detect this common thyroid disorder are: serum thyroid-stimulating hormone test (TSH), anti-thyroid antibodies tests, and the free T4 hormone test.

Signs and symptoms of depression include all of the following except: A) Anhedonia and changes in appetite B) Decreased energy and irritability C) Apathy and low self-esteem D) Apraxia and fatigue

D) Apraxia and fatigue Apraxia is a disorder of the nervous system in which the brain is affected and the patient is unable to move the arms/legs when asked to do so. Common signs of depression include anhedonia (loss of interest in activities that the patient finds pleasurable), unintentional weight loss or gain, fatigue, change in appetite, insomnia or hypersomnia, feelings of guilt and worthlessness, and recurrent thoughts of suicide.

Which of the following drugs that are used to treat attention deficit hyperactivity disorder (ADHD) is not classified as an amphetamine/stimulant? A) Dexmethylphenidate (Focalin XR) B) Mixed salts of amphetamine (Adderall) C) Methylphenidate (Ritalin) D) Atomoxetine (Strattera)

D) Atomoxetine (Strattera) Strattera is classified as a norepinephrine reuptake inhibitor. It is not a stimulant or an amphetamine. Strattera is contraindicated during/within 14 days of taking an MAOI, narrow-angle glaucoma, heart disorder where increases in BP or heart rate will worsen it, or pheochromocytoma. Children and teenagers should be monitored for suicidal thoughts/plans.

All of the following tests require the patient's voice to perform correctly except: A) Egophony B) Tactile fremitus C) Whispered pectoriloquy D) Auscultation

D) Auscultation Egophony, tactile fremitus, and whispered pectoriloquy require the patient to speak. Auscultation is listening to an organ with the use of a stethoscope.

All of the following statements are false regarding the rehabilitation of alcoholics except: A) Al-Anon is not designed for family members of alcoholics B) Disulfiram (Antabuse) is always effective C) Alcoholics Anonymous is not an effective method for treating this condition D) Avoid foods or drinks that contain alcohol, such as cough syrups

D) Avoid foods or drinks that contain alcohol, such as cough syrups Alcoholics Anonymous is designed for families and is an effective treatment for alcoholism. Antabuse is effective for some patients. The intake of foods/medications that contain alcohol must be avoided.

When a patient is suspected of having acute pancreatitis, initial testing should include all of the following except: A) Electrolyte panel B) Serum amylase level C) Serum lipase level D) A barium swallow

D) Barium swallow For an acute abdominal pain, initial labs performed are serum electrolytes, amylase, and lipase. Barium swallow would not be performed initially.

You are performing a pelvic exam on a 25-year-old sexually active woman. You palpate a tender and warm cystic mass on the lower edge of the left labia majora, which is red. The most likely diagnosis is: A) Skene's gland cyst B) Cystocele C) Lymphogranuloma venereum D) Bartholin's gland abscess

D) Bartholin's gland abscess Bartholin's glands are located in the base of the labia minora at about the 4 o'clock and 8 o'clock positions. Their function is to provide moisture for the vestibule. They are small (about pea sized) unless they become clogged or infected. If glands become clogged or infected, an abscess may form and glands will enlarge and become painful.

All of the following are not recommended for the outpatient treatment of mild preeclampsia except: A) Severe sodium restriction B) Restrict fluid intake to less than 1 liter per 24 hours C) Aldomet (methyldopa) 250 mg PO (orally) BID D) Bed rest on the left side with bathroom privileges

D) Bed rest on the left side with bathroom privileges Outpatient treatment for mild pre-eclampsia includes bed rest on the left lateral side with bathroom privileges.

Which type of hepatitis virus infection is more likely to result in chronic hepatitis and increased risk of developing hepatocellular carcinoma? A) Hepatitis Avirus B) Hepatitis B virus C) Hepatitis C virus D) Both hepatitis B and hepatitis C

D) Both hepatitis B and hepatitis C Of the primary hepatitis viruses, only B and C are associated with hepatocellular cancer.

Which of the following cranial nerves is evaluated when a wisp of cotton is lightly brushed against the corner of the eye? A) CN II B) CN III C) CN IV D) CN V

D) CN V Cranial nerves are assessed as follows: II (optic): distance vision, near vision . III, IV, VI (oculomotor, trochlear, abducens): EOMs, visual fields of gaze. V1 (trigeminal 3 branches: V19 ophthalmic), V2 (maxillary), V3 (mandibular): motor portion, clench jaws; sensory portion, corneal reflex/facial sensation

A 55-year-old female with a history of migraine headaches has recently been diagnosed with Stage II hypertension. Her EKG strips reveal second-degree heart block. The chest x-ray is normal. Which of the following drugs should this patient avoid? A) ACE inhibitors B) Angiotensin receptor blockers C) Diuretics D) Calcium channel blockers

D) Calcium channel blockers Common side effects of calcium channel blockers include headaches, edema of the lower extremities, and heart block or bradycardia. Contraindications for calcium channel blockers include second- or third-degree AV block, bradycardia, and congestive heart failure.

A medical assistant is calling out the names of patients who are in the waiting room. The medical assistant is following the HIPAA Privacy Rule if she performs which of the following actions? A) Call patients by their full name to show respect B) Call patients by their last name or surname C) If the patient prefers to be called by a nickname, use that name to call him/her inside D) Call patients by using the first name only

D) Call patients by using the first name only Patients who are in waiting rooms or rooms with other people should be called by their first names only, to protect their privacy.

A 14-year-old is brought in by his mother who reports that her child has been complaining for several months of recurrent bloating, stomach upset, and occasional loose stools. She reports that her son has difficulty gaining weight and is short for his age. She has noticed that his symptoms are worse after eating large amounts of crackers, cookies, and breads. She denies seeing blood in the child's stool. Which of the following conditions is most likely? A) Amebiasis B) Malabsorption C) Crohn's colitis D) Celiac disease

D) Celiac disease Celiac disease is also known as celiac sprue. Patients should avoid foods containing gluten, which causes malabsorption (diarrhea, gas, bloat- ing, abdominal pain, etc.). Foods to avoid are wheat, rye, and barley. Oats do not damage the mucosa in celiac disease. Antigliadin IgA and IgG are elevated in almost all patients (90%).

All of the following are considered benign physiologic variants except: A) Internal tibial torsion B) Supernumerary nipples C) Split uvula D) Cheilosis

D) Cheilosis Cheilosis is a skin disorder in which fissures and maceration occur in the corner of the mouth

2. All of the following are infections that affect mostly the labia and vagina except: A) Bacterialvaginosis B) Candidiasis C) Trichomoniasis D) Chlamydia trachomatis

D) Chlamydia trachomatis Infections that commonly affect the labia and vagina include bacterial vaginosis, candidiasis, and trichomoniasis. Chlamydia trachomatis commonly affects the cervix, endometrial lining , fallopian tubes, and pelvic cavity.

All of the following drugs interfere with the metabolism of oral contraceptives except: A) Tetracycline B) Rifampin C) Phenytoin (Dilantin) D) Ciprofloxacin (Cipro)

D) Ciprofloxacin (Cipro) Cipro is the only medication listed that will not interfere with the metabolism and absorption of OCPs.

Which of the following drugs is recommended by the Centers for Disease Control and Prevention (CDC) as first-line treatment for treating infections by the organ- ism Bacillus anthracis (anthrax)? A) Clindamycin (Cleocin) B) Fluconazole (Diflucan) C) Penicillin G injection D) Ciprofloxacin (Cipro)

D) Ciprofloxacin (Cipro) The CDC's first-line treatment recommendation for anthrax is Cipro.

A 16-year-old male with a recent history of a cat bite is brought in by his father. The bite occurred about 2 hours before the visit. The nurse practitioner evaluates the wound and notes 2 small puncture wounds. There is no redness or purulent discharge. The father reports that the teenager received a tetanus booster when he was 12 years old. Which of the following is the correct action to take? A) Clean the wound with soap and water and apply topical antibiotic and a bandage B) Because the wound is clean and does not appear infected, there is no need for antibiotics C) Give the patient a tetanus booster using the Tdap form of the vaccine D) Clean the wound with soap and water and prescribe Augmentin 500mg PO BID x 10 days for the patient

D) Clean the wound with soap and water and prescribe Augmentin 500 mg PO BID x 10 days for the patient Cat wounds are more likely to become infected compared with dog bites; in addition, this patient's bite is located on an extremity. These facts justify the prescription of Augmentin 500 mg PO BID x 10 days for the patient.

What type of testing is recommended before starting a patient on a prescription of hydroxychloroquine (Plaquenil)? A) CBC B) Serum creatinine and urine for microalbumin C) Liver function tests D) Comprehensive eye exam

D) Comprehensive eye exam A comprehensive eye exam by an ophthalmologist is recommended because hydroxychloroquine can adversely affect the retina (scotomas or visual field defects, loss of central vision, loss of color vision). Higher doses and long-term use increase the risk of retinal toxicity.

A 4-year-old boy is brought in by his mother for a wellness visit. His mother tells the family nurse practitioner that he had a bad case of chickenpox 1 year ago. Which of the following immunizations are indicated? A) IPV, Hib (hemophilus influenzae type B), hepatitis B B) IPV, Hib, MMR C) DTaP, IPV, MMR, hepatitis B D) DTaP, IPV

D) DTaP, IPV DTap and IPV would be due for this 4-year-old child.

A 15-month-old infant is in your office for a well baby check. The mother denies history of chickenpox infection. Which of the following immunizations is indicated at this visit? A) DTaP, Hib, IPV, hepatitis B B) DTaP, Hib, PCV, IPV, MMR C) MMR, hepatitis B, varicella D) DTaP, IPV, MMR, varicella

D) DTap, IPV, MMR, varicella Immunizations for a 15-month-old child include DTap, IPV, MMR, and varicella vaccine.

All of the following are true statements regarding elder abuse except: A) Those aged 80 years or older are at the highest risk for abuse B) A delay in medical care is a common finding C) A new onset of an STD in an elderly patient may signal sexual abuse D) Decreased anxiety and depression are common symptoms of abuse in the elderly

D) Decreased anxiety and depression are common symptoms of abuse in the elderly Elder abuse is commonly seen in elderly patients over the age of 80 years. Common signs/symptoms include anxiety and depression. A new onset of an STD may indicate signs of sexual abuse. These patients will commonly delay treatment for acute/chronic conditions.

According to DSM-5, all of the following are some of the criteria that must be present to diagnose a child with autistic disorder. Which of the following criterion is incorrect? A) Onset of symptoms before age 3 years B) Lack of social interaction or social reciprocity C) Stereotyped and repetitive movements such as hand flapping D) Depressed affect

D) Depressed affect Depression is not included in the criteria for diagnosing autistic spectrum disorders. Signs and symptoms include avoidance of eye contact and social interaction, marked delay or absence of verbal communication, repetitive movements, fixed rituals, and so on. Autism can range from mild to severe. Early diagnosis is important. For evaluation, refer the patient for psychological testing by a psychologist who specializes in autistic spectrum disorders.

Which of the following is considered a relative contraindication for combined oral contraceptive pills? A) Undiagnosed vaginal bleeding B) A hepatoma of the liver C) Suspected history of TIAs D) Depression

D) Depression Depression is a relative contraindication for combined OCPs due to the hormonal effects that can affect mood. Absolute contraindications include hepatoma of the liver, history of embolic episode, history of TIAs, and undiag- nosed vaginal bleeding. OCP should not be considered in these instances due to the high risk factors and health risks.

A 21-year-old woman complains of left-sided pelvic pain accompanied by dyspareunia. During the gynecological exam, the nurse practitioner notices green cervical discharge. The patient mentions a new onset of a painful and swollen left knee and denies a history of trauma. This best describes: A) Septic arthritis B) Reiter's syndrome C) Chondromalacia of the patella D) Disseminated gonorrheal infection

D) Disseminated gonorrheal infection Symptoms of PID with painful, swollen joints of extremities indicate disseminated gonorrheal infection. Untreated disseminated gonorrhea can lead to septic arthritis. Symptoms may be mild from slight joint pain and no fever to severe joint pain with high fever. PID symptoms do not occur with septic arthritis, Reiter's syndrome, or chondromalacia of the patella.

A postmenopausal female complains of random episodes of vaginal bleeding for the past 6 months. Which of the following is recommended management for this condition? A) Cervical biopsy B) Pap smear C) Colposcopy D) Endometrial biopsy

D) Endometrial biopsy Random episodes of vaginal bleeding in a postmenopausal woman are not normal. Endometrial biopsy is needed to biopsy the endometrial lining for abnormal cells, which may indicate cancer.

Medicare Part B will pay for all of the following services except: A) Outpatient physician visits that are medically necessary B) Durable medical equipment C) Outpatient laboratory and radiology tests D) Eyeglasses and routine dental care

D) Eyeglasses and routine dental care Medicare Part B covers: 1) outpatient physician visits, labs, x-rays; 2) durable medical equipment; 3) mammograms/ colonoscopy after age 50 years annually; and 4) rehabilitation.

What structure of the eye is responsible for 20/20 vision (sharpest vision)? A) Rods B) Cones C) Optic disc D) Fovea of the macula

D) Fovea of the macula The fovea is located on the center of the macula and is responsible for the sharpest vision ("20/20 vision") in the eyes. On the fovea, the only receptors are the cones, which allow us to see things in color and in detail. The macula is responsible for central vision.

Erythromycin inhibits the cytochrome P-450 system. The following drugs should be avoided because of a potential for a drug interaction except: A) Theophylline (Theo-Dur) B) Warfarin (Coumadin) C) Diazepam (Valium) D) Furosemide (Lasix)

D) Furosemide (Lasix) There are many medications that are contraindicated with the cytochrome P-450 system. Lasix is one medication that can be used.

You would recommend the pneumococcal vaccine (Pneumovax) to patients with all of the following conditions except: A) Sickle cell anemia B) Splenectomy C) Patients infected with HIV D) G6PD deficiency anemia

D) G6PD deficiency anemia Pneumococcal vaccine should not be given to patients with G6PD deficiency anemia.

A common side effect of metformin (Glucophage) therapy is: A) Weight gain B) Lactic acidosis C) Hypoglycemic episodes D) Gastrointestinal problems

D) Gastrointestinal problems Common side effects of metformin include diarrhea/gastrointestinal problems.

Henry, a 50-year-old male, complains of marked scalp tenderness accompanied by a bad headache on his left temple. He reports a sudden loss of vision in the left eye for the past several hours. The neurologic exam is normal except for the loss of vision in the left eye. Which of the following is most likely? A) Cluster headache B) Migraine headache with aura C) Migraine headache without aura D) Giant cell arteritis

D) Giant cell arteritis Loss of vision does not occur with headaches. Loss of vision, along with marked scalp tenderness with a bad headache on the left temple, are symptoms of giant cell arteritis. Giant cell arteritis is inflammation of the arteries in the head at the temple area, which causes these symptoms.

All of the following are classified as activities of daily living (ADLs) except: A) Ability to feed self (self-feeding) B) Ability to manage bladder and bowel elimination C) Personal hygiene and grooming D) Grocery shopping

D) Grocery shopping Grocery shopping, housework, and managing one's finances are considered instrumental ADLs (IADLs).

All of the following factors increase the risk of mortality for patients diagnosed with bacterial pneumonia except: A) Alcoholism B) Very young age or the elderly C) Multiple lobar involvement D) Hypertension

D) Hypertension Factors that increase the risk of mortality for patients diagnosed with bacterial pneumonia include alcoholism, very young or elderly patients, and multiple lobar involvement.

Sources of legal risk for the nurse practitioner would include all of the following except: A) Invasive procedures B) Electronic medical record entries C) Prescribing medication D) In-service training

D) In-service training Legal risks for the NP include invasive procedures, electronic medical record entries, and prescribing of medications. Inservice training does not directly affect the patient; therefore, legal risks are not a problem.

The following clinical signs are seen in Parkinson's disease except: A) Pill-rolling tremor B) Difficulty initiating involuntary movement C) Shuffling gait with cogwheel rigidity D) Increased facial movements due to tics

D) Increased facial movements due to tics Clinical signs of Parkinson's dis- ease include pill-rolling tremors, difficulty initiating involuntary movements, and shuffling gait with cogwheel rigidity. Facial movement decreases and is known as generalized rigidity with masked facies.

A lipid profile done on a newly diagnosed hypertensive patient shows a triglyceride level of 650 mg/dL, total cholesterol 240 mg/dL, LDL 145 mg/dL, and an HDL of 35 mg/dL. What is the best intervention for this patient? A) Educate the patient about lifestyle changes that will help lower cholesterol levels B) Initiate a prescription of pravastatin (Pravachol) C) Recommend that the patient exercise at least every other day and avoid eating fatty or fried foods D) Initiate a prescription of nicotinic acid (Niacin, Niaspan)

D) Initiate a prescription of nicotinic acid (Niacin, Niaspan) Niacin is recommended for treatment of high triglyceride and cholesterol levels. Exercise and healthy eating lifestyle is also recommended, but with the high level of triglycerides at 650 mg/dL and total cholesterol 240 mg/dL, niacin is recommended.

All of the following are considered risk factors for UTIs in women except: A) Diabetes mellitus B) Diaphragms and spermicide use C) Pregnancy D) Intrauterine device

D) Intrauterine device Risk factors for UTIs include diabetes mellitus, pregnancy, and use of diaphragms and spermicide.

Koilonychia is associated with which of the following conditions? A) Lead poisoning B) Beta thalassemia trait C) B12 deficiency anemia D) Iron-deficiency anemia

D) Iron-deficiency anemia Koilonychia is an abnormal shape of the nail. The nailbed is thin, with irregular edges, and curves inward. Koilonychia is commonly associated with iron-deficiency anemia.

What is the best description of a variable? A) It is an important part of every research study B) It is the probability that a factor is important for the research data C) It is the value or number that occurs the most frequently D) It is a condition, characteristic, or factor that is being measured

D) It is a condition, characteristic, or factor that is being measured A variable is a condition, characteristic, or factor that is being measured. An independent variable is the one being manipulated that is not affected by the others. A dependent variable changes depending on the manipulation of the independent variable.

All of the following statements are correct regarding licensure for nurse practitioners except: A) It ensures a minimum level of professional competency B) It grants permission for an individual to practice in a profession C) It requires verification of educational training from an accredited graduate program D) It reviews information via a nongovernmental agency

D) It reviews information via a nongovernmental agency Licensure for nurse practitioners includes a minimum level of professional competency, grants per- mission for one to practice in the profession, and verifies completion of an accred- ited graduate program.

The following are treatment plans related to migraine headaches. Which one of the following would not be considered effective therapy? A) Propranolol (Inderal) B) Cold packs to the forehead C) Trimethobenzamide (Tigan) D) Moderate sodium restriction

D) Moderate sodium restriction Effective treatments for migraine headaches include propranolol, trimethobenzamide, cold packs to the forehead/neck, and hydration.

An 18-year-old male is found to have a 47, XXY karyotype and is diagnosed with Klinefelter's syndrome. The patient is most likely to have all of the following physical characteristics except: A) Gynecomastia B) Long limbs C) Lack of secondary sexual characteristics D) Large testes

D) Large testes Signs/symptoms of Klinefelter's syndrome include gynecomastia, long limbs, and lack of secondary sexual characteristics. Testes are usually small. Infertility is a major concern for these boys. If treated early, they may have a normal sexual/reproductive system in the future.

Which of the following findings is associated with the chronic use of chewing tobacco? A) Cheilosis and xerostomia B) Glossitis C) A geographic tongue D) Leukoplakia and oral cancer

D) Leukoplakia and oral cancer The chronic use of tobacco increases the risk of oral cancer and leukoplakia. Cheilosis is skin fissures/maceration in the corner of the mouth, most commonly caused by anemia, bacterial infection, vitamin deficiencies, or oversalivation.

The first nurse practitioner program was started by: A) Alfred Bandura B) President John F. Kennedy C) Federal government D) Loretta Ford, PhD

D) Loretta Ford, PhD The first NP program was started by Loretta Ford, PhD, at the University of Colorado; it was a certificate program for NPs.

The following are acceptable methods of birth control for breastfeeding mothers except: A) Diaphragm with spermicidal gel B) Progesterone-only pills (Micronor) C) Condoms D) Low-dose oral contraceptives with at least 20 mcg of estradiol (Alesse, Lo-estrin)

D) Low-dose oral contraceptives with at least 20 mcg of estradiol (Alesse, Lo-estrin) Low-dose oral contraceptives that contain estradiol are contraindi- cated for breastfeeding mothers.

Which of the following findings are seen in a patient with folate-deficiency anemia? A) Microcytic and hypochromic RBCs B) Microcytic and normochromic RBCs C) Normal size and color of the RBCs D) Macrocytic and normocytic RBCs

D) Macrocytic and normocytic RBCs Folate-deficiency anemia is diagnosed by macrocytic, normocytic red blood cell.

Which of the following physical exam findings is most specific for systemic lupus erythematosus (SLE)? A) Swollen and painful joint involvement B) Fatigue and myalgia C) Stiffness and swelling of multiple joints D) Malar rash

D) Malar rash A malar rash is the butterfly-shaped rash on the middle of the face that is caused by a type of photosensitivity reaction. It is associated with SLE. The other answer options are found with other diseases, such as rheumatoid arthritis, polymyalgia rheumatica, and so on.

Carpal tunnel syndrome is due to inflammation of the: A) Ulnar nerve B) Radial nerve C) Brachial nerve D) Median nerve

D) Median nerve Carpal tunnel syndrome is due to inflammation of the median nerve.

Systemic lupus erythematosus (SLE) is more common among the following racial backgrounds except: A) AfricanAmericans B) Asians C) Hispanics D) Mediterranean

D) Mediterraneans Lupus is more common among women of AfricanAmerican, Asian, and Hispanic racial backgrounds compared with those with Mediterranean background (Italians, Greeks, etc.).

Patients with Down syndrome are at higher risk for all of the following except: A) Atlantoaxial instability B) Congenital heart disease C) Early onset of Alzheimer's disease D) Melanoma

D) Melanoma Patients with the diagnosis of Down syndrome are at higher risk for atlantoaxial instability, congenital heart defects, and early onset of Alzheimer 's disease. Children with Down syndrome who participate in sport activities must be carefully examined for these conditions prior to participation to prevent injury.

All of the following are considered Category X drugs except: A) Misoprostol (Cytotec) B) Isotretinoin (Accutane) C) Finasteride (Proscar) D) Meperidine (Demerol)

D) Meperidine (Demerol) Demerol is in drug Category C and and treated by the FDA as a controlled drug Schedule II.

All of the following are clinical eye findings found in some patients with chronic uncontrolled hypertension. Which of the following findings is not associated with this disorder? A) AV nicking B) Copper wire arterioles C) Flame-shaped hemorrhages D) Microaneurysms

D) Microaneurysms "Keith Wagener Barker (KWB) Grades" for uncontrolled hypertension: Grade 1: generalized arteriolar constriction seen as "silver wiring" and vascular tortuosities; Grade 2: grade 1 plus irregularly located, tight constric- tions known as "AV nicking" or "AV nipping"; Grade 3: grade 2 plus cotton wool spots and flame-shaped hemorrhages; Grade 4, grade 3 but with swelling of the optic disk (papilledema). Microaneurysms occur with diabetic retinopathy.

Three of the following are eye findings associated with chronic uncontrolled hypertension. Which one of the following is associated with diabetic retinopathy? A) AV nicking B) Copper wire arterioles C) Flame hemorrhages D) Microaneurysms

D) Microaneurysms Microaneurysms are seen with diabetic retinopathy. AV nicking, copper wire arterioles, and flame hemorrhages are seen with uncontrolled hypertension.

A 70-year-old male with open-angle glaucoma is prescribed Betimol (timolol) ophthalmic drops. All of the following are contraindications to Betimol ophthalmic drops except: A) Overt heart failure or sinus bradycardia B) Asthmatic patients C) Second- or third-degree AV block D) Migraine headaches

D) Migraine headaches Contraindications with Betimol (timolol) include heart failure or sinus bradycardia, asthmatic patients, and second- or third-degree AV block

Potential complications of mitral valve prolapse (MVP) include all of the following except: A) Severe mitral regurgitation B) Endocarditis C) Increased risk of stroke and TIA D) Mitral stenosis

D) Mitral stenosis Complications of MVP include mitral regurgitation, endocarditis, and increased risk of stroke and TIAs.

A 40-year-old woman is in the office complaining of palpitations and some light headedness for the past 6 months. These are random episodes. The nurse practitioner notices a mid-systolic click with a late systolic murmur that is best heard in the apical area during auscultation of the chest. The NP would suspect: A) Atrial fibrillation B) Sinus arrhythmia C) Mitral stenosis D) Mitral valve prolapse

D) Mitral valve prolapse Mitral valve prolapse occurs when the mitral valve does not close all the way, causing a late systolic murmur heard best in the apical area during auscultation of the chest. Symptoms patients may experience at times include palpitations and dizziness.

The differential diagnosis for genital ulceration includes all of the following except: A) Syphilis B) Genital herpes C) Chancroid D) Molluscum contagiosum

D) Molluscum contagiosum Genital ulcers may occur with syphilis, genital herpes, and chancroid. Molluscum contagiosum is a viral infection that causes smooth, round tiny papules, approximately 5 mm or less, that have a central umbilication with a white plug present.

Which of the following findings is associated with thyroid hypofunction? A) Graves disease B) Eye disorder C) Thyroid storm D) Myxedema

D) Myxedema Myxedema is a rare, and sometimes fatal, disease in which the thyroid is severely underactive and causes life-threatening symptoms. These symptoms include low blood pressure, decreased breathing, decreased body temperature, unresponsiveness, and even coma. Graves' disease, thyroid storm, and eye disorder are seen with an overactive thyroid disease known as hyperthyroidism.

An adult patient is being evaluated for tuberculosis infection with a Mantoux test. The PPD result is 10.5 mm. The patient denies weight loss, cough, and night sweats and has a negative chest x-ray. The patient reports that he is in the United States illegally and is fearful about discovery. What is the most appropriate action for the nurse practitioner? A) The patient is an illegal alien/migrant and the nurse practitioner has the legal duty to report the patient to the local State Department responsible for illegal migrants B) Health care workers are legally mandated to report illegal migrants to state authorities. C) The nurse practitioner should call the state health department and report that the patient has a TB infection D) Nurse practitioners have the ethical duty to provide quality health care to patients

D) Nurse practitioners have the ethical duty to provide quality health care to patients Currently, health caregivers are not legally required to report illegal aliens to the state or local authorities. This patient does not have the signs and symptoms of active TB disease (cough, weight loss, night sweats) and has a negative chest x-ray. Therefore, he has latent TB infection and is not contagious. Only patients with active TB disease (has signs/symptoms) must be reported to the state public health department.

A college student is seen as a walk-in appointment in a college health clinic. She complains of the abrupt onset of sore throat, nasal congestion, runny nose, and malaise. Vital signs are a temperature of 99.8 degrees Fahrenheit, pulse 84, and respiratory rate 14 breaths/minute. The physical exam reveals an erythematous throat, swollen nasal turbinates, and rhinitis. The NP suspects viral URI. All of the following treatments are appropriate except: A) Saline nasal spray (Ocean nasal spray) B) Pseudoephedrine (Sudafed) C) Ibuprofen (Advil) D) Oral prednisone (Medrol Dose Pack)

D) Oral prednisone (Medrol Dose Pack) Symptomatic treatment for viral URI are saline nasal sprays (Ocean spray), decongestants (pseudoephedrine), NSAIDs (Advil), increased fluid intake, and alternative herbal remedies (echinachea, astralagus, elderberry syrup, high doses of Vitamin C).

All of the following clinical findings are classified as major criteria that are necessary to diagnose pelvic inflammatory disease (PID). Which of the following is classified as a minor criterion? A) Cervical motion tenderness B) Adnexal tenderness C) Uterine tenderness D) Oral temperature of more than 101°F (more than 38°C)

D) Oral temperature of more than 101°F (more than 38°C) PID is a clinical diagnosis. The presence of at least 1 of the major criteria (cervical motion tenderness, adnexal tenderness, uterine tenderness) when combined with the history is highly suggestive of PID. Minor criteria are not necessary, but they help to support the diagnosis of PID (oral temperature of more than 101°F or more than 38°C), mucopurulent cervical or vaginal discharge, elevated sedimentation rate, elevated C-reactive protein, large amount of WBCs on saline microscopy of the vaginal fluid, or laboratory documentation of cervical infection with N. gonorrhoeae or C. trachomatis).

Women with a history of pelvic inflammatory disease (PID) have an increased risk for all of the following complications except: A) Ectopic pregnancy B) Scarring of the fallopian tube(s) C) Infertility D) Ovarian cysts

D) Ovarian cysts Women with a history of pelvic inflammatory disease (PID) have a higher risk of ectopic pregnancy, scarring of the fallopian tubes, and infertility due to the scarring and trauma caused by the pelvic inflammation.

A woman at 32 weeks gestation has a positive throat culture for strep pyogenes. She denies allergies but gets very nauseated with erythromycin. Which of the fol- lowing is the best choice for this pregnant patient? A) Clarithromycin (Biaxin) B) Trimethoprim/sulfamethoxazole (Bactrim DS) C) Ofloxacin (Floxin) D) Penicillin (Pen VK)

D) Penicillin (Pen VK) Pen VK is safe to use for strep throat during pregnancy. Pen VK is a category B medication for pregnancy and lactation.

A nurse practitioner is teaching a 54-year-old woman with stress urinary incontinence about Kegel exercises. The patient is instructed to tighten her pelvic floor muscles for a count of 10 and then to relax them for a count of 10. The nurse practitioner instructs the patient that Kegel exercises should be done consistently every day at what frequency? A) Perform 30 exercises each time in the morning and the evening B) Perform 20 exercises each time 2 times a day C) Perform 15 exercises each time 3 to 4 times a day D) Perform 10 exercises each time 3 times a day

D) Perform 10 exercises each time 3 times a day Weak pelvic floor muscles increase the risk of urinary and fecal incontinence. Educate a female patient that the pelvic floor muscles are the ones that she uses when she consciously holds/ stops the flow of urine when she urinates. Warn her that the anal sphincter will also tighten with the vaginal muscles. Advise the patient to relax the abdomen and the thighs when doing the exercises. Kegel exercises are also recommended as conservative treatment for patients (male and female) with fecal incontinence.

All of the following conditions are associated with an increased risk for normocytic anemia except: A) Rheumatoid arthritis B) Lupus C) Chronic autoimmune disorders D) Pregnancy

D) Pregnancy During pregnancy, women may experience microcytic, hypochromic anemia due to the dilutional effect of the increased blood volume during the pregnancy.

A 74-year-old man presents with recurrent abdominal cramping associated with diarrhea that occurs from 4 to 5 times per day. He reports that currently he is having an exacerbation. The stools are bloody with mucus and pus. The patient reports that he has lost a little weight and is always fatigued. The patient denies recent travel or outdoor camping. Which of the following conditions is most likely? A) Giardiasis B) Irritable bowel syndrome (IBS) C) Diverticulitis D) Ulcerative colitis

D) Ulcerative colitis The most important clue for ulcerative colitis is bloody stools that are covered with mucus and pus along with the systemic symptoms (fatigue, low-grade fever).

A patient with a history of mitral valve prolapse (MVP) is requesting prophylaxis before her dental surgery. Which of the following would you prescribe this patient? A) Amoxicillin a half hour before and 2 hours after the procedure B) Amoxicillin 1 hour before the procedure C) Amoxicillin 1 hour before and 3 hours after the procedure D) Prophylaxis is not recommended for this patient

D) Prophylaxis is not recommended for this patient The American Heart Association does not recommend routine antibiotic prophylaxis prior to dental procedures except for patients who are at high risk for bacterial endocarditis (a prosthetic cardiac valve, previous history of bacterial endocarditis, congenital heart defects, etc.).

A skilled nursing facility (SNF) can provide all of the following except: A) Physical therapy and other types of rehabilitation B) Skilled nursing care and medical care C) Reimbursement by Medicare D) Provision of custodial care

D) Provision of custodial care Custodial care is done by nursing homes. SNFs are reimbursed by Medicare and can provide skilled nursing and medical care. If a patient is discharged from a hospital, but needs therapy and skilled care, he/she is usually transferred to an SNF. SNFs have transfer agreements with local hospitals. Nursing home patients are usually medically stable and need skilled or medical care. These patients are unable to perform from 2 to 6 ADLs. Patients with Alzheimer's and other types of dementia are usually cared for in nursing homes.

Which of the following conditions is associated with a positive Auspitz sign? A) Contact dermatitis B) Seborrheic dermatitis C) Systemic lupus erythematosus D) Psoriasis

D) Psoriasis Auspitz sign is the presence of pinpoint bleeding spots from psoriasis where the skin is scraped off.

Mary, who has recently been diagnosed with lupus, complains that her hands and feet always feel cold even in the summertime. Sometimes her fingertips become numb and turn a blue color. The fingertips eventually turn to a dark red color. Which of the following is most likely? A) Chronic arterial insufficiency B) This is a normal reaction when one feels very cold C) Peripheral vascular disease D) Raynaud's syndrome

D) Raynaud's syndrome Raynaud's syndrome occurs from vasospasms of the blood vessels, leading to decreased blood supply to the hands/feet, which causes bluish discoloration, with fingertips turning a dark red color if severe. Stress and cold weather are classic triggers for this.

The mother of an 11-year-old boy with sickle cell anemia calls on the phone complaining that her son woke up because of a painful penile erection that will not go away. The nurse practitioner's most appropriate intervention is: A) Insert a Foley catheter and measure the child's intake and output for the next 24 hours B) Insert a Foley catheter to obtain a specimen for a urinalysis and urine for C&S (culture and sensitivity) C) Recommend an increase in the child's fluid intake D) Recommend immediate referral to the ER

D) Recommend immediate referral to the ED Priapism (painful penile erection not related to sexual activity) is a true urologic emergency that may lead to permanent erectile dysfunction and penile necrosis if not treated appropriately. It can be associated with a number of medical conditions (sickle cell anemia, leukemia, or spinal cord injury) and/or some pharmacologic agents

A patient with chronic obstructive pulmonary disease (COPD) is referred for pulmonary function testing. Which of the following pulmonary function tests are abnormal in patients with COPD? A) Reduction of the TLC (total lung capacity) and the RV (residual volume) B) Complaints of mild to severe dyspnea with hypoxemia C) Normal forced vital capacity (FVC) with no changes in the FEV1 (forced expiratory volume in 1 second) D) Reduction of the FEV1 (forced expiratory volume in 1 second) with increase in the TLC (total lung capacity) and RV (residual volume)

D) Reduction of the FEV1 (forced expiratory volume in 1 second) with increase in the TLC (total lung capacity) and RV (residual volume) COPD findings during pulmonary function testing are the reduction of the FEV1 (forced expiratory volume in 1 second) and reduction of the FVC (forced vital capacity). There is an increase in the TLC (total lung capacity) and RV (residual volume). The lungs of patients with emphysema have lost their recoil (decreases FEV1). The lungs are always full of air that is hard to "squeeze out" of the lungs (increases residual volume and total lung capacity). To summarize, COPD=reduction in FEV1 and FVC with increased RV and TLC.

Your 35-year-old patient is being worked up for microscopic hematuria. All of the following are differential diagnoses of microscopic hematuria except: A) Kidney stones B) Bladder cancer C) Acute pyelonephritis D) Renal stenosis

D) Renal stenosis Renal stenosis is a narrowing of the renal artery. No blood would be noted on exam. Evidence of blood in the urine can be seen with kidney stones, bladder cancer, and acute pyelonephritis.

An elderly woman with a history of rheumatoid arthritis reports to the nurse practitioner that she had been taking ibuprofen BID for many years. Which of the following organ systems has the highest risk of damage from chronic non- steroidal anti-inflammatory drug (NSAID) use? A) Cardiovascular system B) Neurological system C) Gastrointestinal system D) Renal system

D) Renal system The 2 main adverse drug reactions associated with NSAIDs relate to gastrointestinal (GI) effects and renal effects of the agents. The main adverse drug reactions associated with use of NSAIDs relate to direct and indirect irritation of the GI tract. Ulceration risk increases with therapy duration and with higher doses. NSAIDs can induce 2 different forms of acute kidney injury: hemodynamically mediated, and acute interstitial nephritis, which is often accompanied by nephrotic syndrome.

An 18-year-old waitress is diagnosed with pelvic inflammatory disease (PID). The cervical Gen-Probe result is positive for Neisseria gonorrhoeae and negative for Chlamydia trachomatis. All of the following statements are true regarding the man- agement of this patient except: A) This patient should be treated for chlamydia even though the Gen-Probe for chlamydia is negative B) Ceftriaxone 250 mg IM and doxycycline 100 mg PO BID x 14 days are appropriate treatment for this patient C) Advise the patient to return to the clinic for a repeat pelvic exam in 48 hours D) Repeat the Gen-Probe test for Chlamydia trachomatis to ensure that the previous test was not a false-negative result

D) Repeat the Gen-Probe test for Chlamydia trachomatis to ensure that the previous test was not a false-negative result Treatment for both gonorrhea and chlamydia are recommended for the diagnosis of PID, regardless if the chlamydia test was negative.

A new patient is complaining of severe pruritus that is worse at night. Several family members also have the same symptoms. Upon examination, areas of excoriated papules are noted on some of the interdigital webs of both hands and on the axillae. This finding is most consistent with: A) Contact dermatitis B) Impetigo C) Larva migrans D) Scabies

D) Scabies The classic symptom of scabies is severe pruritis that worsens at night. The pruritic rash is commonly seen on the trunk and between the fingers, and appears as excoriated, erythemic papules. Scabies is contagious and can affect other family members.

A 22-year old is going on a 5-day cruise for her honeymoon. She reports a history of severe motion sickness. Which of the following medicines can be prescribed for motion sickness? A) Dimenhydrinate (Dramamine) B) Metochlopramide (Reglan) C) Ondansetron (Zofran) _ D) Scopolamine patch (Transderm Scop)

D) Scopolamine patch (Transderm Scop) Scopolamine patch (Transderm Scop) is a prescription medicine that is used for motion/sea sickness. It is a small, circular patch that is placed behind the ear and is effective for 3 days. Advise the patient to apply it 4 hours before the trip to be effective. Because the question is asking about a "prescribed" med, answers with OTCs such as Dramamine are incorrect. Zofran is indicated for cancer-related nausea and vomiting (chemo, radiation, surgery).

At level of prevention would you classify screening for lung cancer? A) Primary prevention B) Secondary prevention C) Tertiary prevention D) Screening for lung cancer is not currently recommended

D) Screening for lung cancer is not currently recommended

The nurse practitioner would refer all of the following to a physician except: A) Severe facial burns B) Electrical burns C) Burns that involve the cartilage of the ear D) Second-degree burns on the lower arm

D) Second-degree burns on the lower arm First- and second-degree burns are appropriately treated by a nurse practitioner. Third-degree burns should be referred to a physician. Examples of third-degree burns include electrical burns, severe burns on the face, and burns involving cartilage, such as the ear and nose.

Acute prostatitis can present with all of the following signs and symptoms except: A) Fever and chills B) Tenderness of the scrotum on the affected side C) Perineal pain D) Slow onset of symptoms

D) Slow onset of symptoms Acute prostatitis symptoms include fever, chills, tenderness of the scrotum on the affected side, and abrupt onset of symptoms.

A faun tail nevus is a sign of which of the following? A) Down syndrome B) Infantile scoliosis C) Congenital heart disease D) Spina bifida

D) Spina bifida A faun tail nevus is an abnormal tuft of hair in the lumbosacral area, which can be a sign of spina bifida.

A nurse practitioner is giving dietary counseling to a male alcoholic who has recently been diagnosed with folic acid deficiency anemia. Which of the following foods should the nurse practitioner recommend to this patient? A) Tomatoes, oranges, and bananas B) Cheese, yogurt, and milk C) Lettuce, beef, and dairy products D) Spinach, liver, and whole wheat bread

D) Spinach, liver, and whole wheat bread Foods that are higher in folic acid include spinach, liver, and whole wheat bread.

What would you advise him regarding his fluvastatin (Lescol) prescription? A) Continue taking the medicine until the lab results are available B) Take half the usual daily dose until the lab results are available C) Take the medicine every other day instead of daily until the lab results are available D) Stop taking the medicine until the lab results are available

D) Stop taking the medicine until the lab results are available When liver enzymes are affected by medications, such as statin medications, the patient is advised to stop taking the medicine until the lab results are available.

A 68-year-old woman complains of leaking a small amount of urine whenever she sneezes, laughs, and/or strains. The problem has been present for many months. The patient denies dysuria, frequency, and nocturia. The urine dipstick test is neg- ative for white blood cells, red blood cells, ketones, and urobilinogen. What is the name of this condition? A) Urge incontinence B) Overflow incontinence C) Urinary incontinence D) Stress incontinence

D) Stress incontinence The signs and symptoms of stress incontinence occur when the stress caused by sneezing, laughing, and/or straining results in the leaking of a small amount of urine through a weakened sphincter.

While performing a routine physical exam on a 60-year-old male, the nurse practitioner notices a soft bruit over the carotid area on the left side of the neck. The patient has a history of hypertension. The patient is at higher risk for: A) Temporal arteritis and brain aneurysms B) Dizziness and headaches C) Abdominal aneurysm and congestive heart failure D) Stroke and coronary heart disease

D) Stroke and coronary heart disease Carotid stenosis puts the patient at risk for stroke and coronary heart disease

The cremasteric reflex is elicited by: A) Asking the patient to open his or her mouth and touching the back of the pharynx with a tongue blade B) Hitting the biceps tendon briskly with a reflex hammer and watching the lower arm for movement C) Hitting the patellar tendon briskly with a reflex hammer and watching the lower leg for movement D) Stroking the inner thigh of a male client and watching the testicle on the ipsilateral side rise up toward the body

D) Stroking the inner thigh of a male client and watching the testicle on the ipsilateral side rise up toward the body The cremasteric reflex test is done by stroking the inner thigh of a male client and watching for the testicle on the ipsi- lateral side to rise up toward the body.

A 70-year-old male patient complains of a bright red spot in his left eye for 2 days. He denies eye pain, visual changes, or headaches. He has a new onset of cough from a recent viral upper respiratory infection. The only medicine he is on is Bayer aspirin 1 tablet a day. Which of the following is most likely? A) Corneal abrasion B) Acute bacterial conjunctivitis C) Acute uveitis D) Subconjunctival hemorrhage

D) Subconjunctival hemorrhage Subconjunctival hemorrhage is a benign disorder that occurs from an increase in intraocular pressure that may be caused by coughing, vomiting, forceful exerrtion in labor during childbirth, straining while having a bowel movement, weight lifting, or lifting a heavy object.

Ken has type 2 diabetes mellitus and a "sensitive stomach." Which medication is least likely to cause him gastrointestinal distress? A) Naproxen sodium (Anaprox) B) Aspirin (Bayer's aspirin) C) Erythromycin (E-mycin) D) Sucralfate (Carafate)

D) Sucralfate (Carafate) Naproxen sodium, aspirin, and erythromycin all have gastrointestinal side effects. Sucralfate is used to protect the stomach lining by building a protective layer over the stomach lining; it allows healing to occur

All of the following statements about common health beliefs of many traditional Asian cultures are true except: A) An imbalance of the hot and cold (yin/yang) vital forces can cause illness, and treating a hot disease with a "cold" treatment (i.e., certain foods/herbs) can help to restore balance and cure the illness B) If the patient is very ill or dying, immediate family and extended family members will visit the patient daily in "shifts" to provide emotional support C) Babies and small children may wear an amulet such as a red string on the wrist or a piece of cloth on the neck or the wrist D) Surgical procedures are regarded as important treatment for many illnesses

D) Surgical procedures are regarded as important treatment for many illnesses The Vietnamese regard surgery as a last resort and consider loss of blood as depleting the vital forces of the body and causing illness. Western medicine is considered "hot," and patients may discontinue or reduce the doses of their medicine without asking. An imbalance of the hot and cold (yin/yang) is believed to cause illness. Treating a "yin" disease (common cold) means avoiding eating yin foods (melons, cucumbers) because they will worsen it. Instead, yin diseases are treated with yang foods (meat, spicy foods) so that the body becomes more balanced. If the patient is very ill or dying, immediate family and extended family members will visit the patient daily in "shifts" to provide emotional support. Babies and small children may wear an amulet such as a red string on the wrist or a piece of cloth that ties on the neck or the wrist.

When an adolescent female's areola and nipples elevate above the level of the breasts to form a secondary mound, which of the following is the correct Tanner stage for this phase of breast development? A) Tanner Stage I B) Tanner Stage II C) Tanner Stage III D) Tanner Stage IV

D) Tanner Stage IV At Tanner Stage IV, the breast tissue is still growing. This is the stage in which the areola and the nipple separate to form a distinct mound. The most important clue is "secondary mound." During Tanner Stage III, the breast and areola and nipples grow together in one mound. There is no separation yet.

A 45-year-old gardener is seen as a walk-in patient in a private clinic. He reports stepping on a nail that morning. His last tetanus vaccine was 7 years ago. Which of the following vaccines is recommended? A) DTaP B) DT C) Td D) Tdap

D) Tdap The CDC recommends that 1 of the tetanus boosters be replaced with the Tdap (once in a lifetime). Thereafter, the Td form of the vaccine is indicated every 10 years. The DTaP (diphtheria-tetanus-acellullar pertussis) and DT (diphtheria-tetanus) forms of the tetanus vaccine are not given after the age of 7 years. Puncture wounds are at higher risk for tetanus because Clostridium tetani bacteria are anaerobes (deep puncture wounds are not exposed to air compared with superficial wounds).

In addition to surgical repair of a compound fracture that has broken through the skin, which of the following treatment plans is important to consider in this patient? A) Application of a topical antibiotic BID until the wound is healed B) Wound irrigation C) Tdap vaccine D) Tetanus vaccine and systemic antibiotics

D) Tetanus vaccine and systemic antibiotics A break in the skin with a compound fracture is an indication for use of a tetanus vaccine (if last dose is more than 5 years ago) and systemic antibiotics.

A nurse practitioner is evaluating an 80-year-old woman from a nursing home. She is instructing the patient to remember the words "orange," "house," and "world." A few minutes later, the patient is told to recall these 3 words. Which of the following is being described? A) The Lachman test B) A neurologic exam C) The Romberg test D) The Mini Mental Status Exam

D) The Mini Mental Status Exam The Lachman test is performed on the knee to assess stability; the Romberg test is performed to assess for balance; a neurologic exam is performed by assessing the 12 cranial nerves; the MMSE is performed to assess for cognitive impairment.

The Romberg test is done to check for problems with balance. Which area of the brain is responsible for balance? A) Frontal lobe B) Temporal lobe C) The midbrain D) The cerebellum

D) The cerebellum The Romberg test evaluates the cerebellum, which is responsible for balance. This test is performed by having the client stand up straight with feet together and then having him/her close the eyes and stand still for at least 20 seconds. A positive Romberg exam would demonstrate loss of balance.

Which of the following is a true statement regarding the effect of aspirin on platelet function? A) The effect on platelets is reversible B) The effect on platelets is reversible and lasts only 1 week C) It has a minimal effect on platelet function D) The effect on platelet function is irreversible and lasts 15 to 20 days

D) The effect on platelet function is irreversible and lasts 15 to 20 days The use of aspirin affects the platelet function, is irreversible, and can last up to 15-20 days

The apex of the heart is located at: A) Second ICS (intercostal space) to the right of the sternal border B) Second ICS to the left of the sternal border C) The left lower sternal border D) The left side of the sternum at the fifth ICS by the midclavicular line

D) The left side of the sternum at the fifth ICS by the midclavicular line The apex of the heart is located at the left side of the sternum at the fifth ICS by the midclavicular line.

An adult female presents with complaints of "bad burns" that are very painful. A large pot of boiling water tipped over and spilled on her arms and her anterior chest and abdomen. During the physical exam, the nurse practitioner notices bright red skin with numerous bullae on the left arm and hand and large patches of bright red skin on the anterior chest and abdominal area. On a pain scale of 1-10, she reports the pain as 9. Her vital signs are stable with tachycardia (pulse is 100/minute). She does not appear to be in shock. Using the Rule of Nines, what is the total body surface area (TBSA) and the depth of the burns in this patient? A) The patient has a TBSA of 15% with full-thickness burns of the left arm and left hand, and partial-thickness burns of the anterior chest and abdominal area B) The patient has a TBSA of 20% with partial-thickness burns on the left arm, left hand, and mild burns on the anterior chest and abdominal area C) The patient has a TBSA of 27% with partial-thickness burns on the left arm and left hand with superficial burns on the anterior chest and abdominal area D) The patient has a TBSA of 18% with full-thickness burns of the left arm and left hand, and superficial burns on the anterior chest

D) The patient has a TBSA of 18% with full-thickness burns of the left arm and left hand, and superficial burns on the anterior chest

A 72-year-old female complains to you of a crusty and non-healing small ulcer on her upper lip that she has had for several years. Which of the following would you recommend? A) Triamcinolone acetonide (Kenalog) cream BID for 2 weeks B) Triple antibiotic ointment BID x 2 weeks C) Hydrocortisone 1% cream BID for 2 weeks D) The patient needs to be evaluated by a dermatologist

D) The patient needs to be evaluated by a dermatologist Non healing ulcers of the skin are a risk for skin cancer and should be evaluated by a dermatologist for treatment.

All of the following are correct statements regarding the role of the person named in a durable power of attorney except: A) The agent's decisions are legally binding B) The agent can make decisions in other areas of the patient's life such as financial issues C) The agent can decide for the patient who is on life support when that life support can be terminated D) The patient's spouse has a right to override the agent's decisions

D) The patient's spouse has a right to override the agent's decisions The person named in a durable power of attorney (the agent) is designated by the patient to make all medical decisions, as well as any decisions regarding the patient's private affairs in the event that the patient becomes incompetent and unable to make his/her own decisions. No one has the ability to override the agent's decision.

The following statements are true about Wilms tumor except: A) The most frequent clinical sign is a palpable abdominal mass B) It is a congenital tumor of the kidney C) Microscopic or gross hematuria is sometimes present D) The tumor commonly crosses the midline of the abdomen when it is discovered

D) The tumor commonly crosses the midline of the abdomen when it is discovered A Wilm's tumor is a congential tumor of the kidney that should never be palpated, once diagnosed, to avoid spread of the tumor cells. Microscopic or gross hematuria may be present.

A fracture on the navicular area of the wrist is usually caused by falling forward and landing on the hands. The affected wrist is hyperextended to break the fall. The nurse practitioner is aware that all of the following statements are true except: A) It has higher rate of nonunion compared with the other bones in the wrist when it is fractured B) The fracture frequently does not show up on an x-ray film when it is taken immediately after the injury C) The x-ray film will show the fracture if the film is repeated in 2 weeks D) These fractures always require surgical intervention to stabilize the joint

D) These fractures always require surgical intervention to stabilize the joint A fracture of the navicular area of the wrist occurs when one tries to break a fall with the hands, thus hyperextending the wrist. Immediately following the injury, the fracture may not show on x-ray; however, fracture is usually visible on x-ray if the x-ray is repeated in 2 weeks. The need for surgery is dependent on the extent of the fracture.

Acanthosis nigricans is associated with all of the following disorders except: A) Obesity B) Diabetes C) Colon cancer D) Tinea versicolor

D) Tinea versicolor Acanthosis nigricans is a benign skin condition. It appears as hyperpigmented velvety areas of skin that are usually located on the neck and the axillae. It is a sign of insulin resistance. It is rarely associated with some types of adenocarcinoma of the gastrointestinal tract. Tinea versicolor is a superficial infection of the skin (stratum corneum layer) that is caused by dermatophytes (fungi) of the tinea family. Another name for it is "sunspots."

Which of the following symptoms is associated with B12 deficiency anemia? A) Spoon-shaped nails and pica B) An abnormal neurological exam C) A vegan diet D) Tingling and numbness of both feet

D) Tingling and numbness of both feet Vitamin B12 deficiency anemia can cause nerve cell damage if not treated. Symptoms of B12 deficiency anemia may include tingling or numbness in fingers and toes, difficulty walking, mood changes or depression, memory loss, disorientation, and dementia.

A 17-year-old boy reports feeling something on his left scrotum. On palpation, soft and movable blood vessels that feel like a "bag of worms" are noted underneath the scrotal skin. It is not swollen or reddened. The most likely diagnosis is: A) Chronic orchitis B) Chronic epididymitis C) Testicular torsion D) Varicocele

D) Varicocele Palpation of varicose veins, known as "bag of worms" in the scrotum, is a classic symptom of a varicocele. Chronic epididymitis and chronic orchi- tis are caused by a bacterial infection and commonly have burning, frequency, and pain. Testicular torsion is an emergent condition in which the testicle becomes twisted, interrupting the blood supply to the testis; to avoid damage, the condition must be corrected within 6 hours.

Auscultation of normal breath sounds of the chest will reveal: A) Bronchial breath sounds heard at the lower bases B) High-pitched vesicular breath sounds heard over the upper lobes C) Vesicular breath sounds heard over the trachea D) Vesicular breath sounds in the lower lobe

D) Vesicular breath sounds in the lower lobe Normal sounds of the chest wall include vesicular breath sounds in the lower lobes. Bronchial breath sounds are heard best at the second and third intercostal spaces. Tracheal breath sounds are heard over the trachea.

Peak expiratory flow (PEF) meters are used to monitor asthma by using personal best measurements. All of the following factors are used to determine the PEF except: A) Age B) Gender C) Height D) Weight

D) Weight Peak expiratory flow volume is determined by using height, gender, and age. Weight is not used in the formula.

An Rh-negative pregnant woman with negative rubella titers should be vaccinated at what time period in pregnancy? A) She can be vaccinated at any time in her pregnancy B) During the second trimester C) During the third trimester D) During the postpartum period

D)During the post partum period Rubella should be administered to the woman during the postpartum period. Rubella is contraindicated during pregnancy.


Kaugnay na mga set ng pag-aaral

Intro to Film: Chapter 6: Cinematography

View Set

Cybersecurity Level 2 Study Guide

View Set

OEA Study guide - Social Studies

View Set

SUA Honors Chemistry 2014 Final exam

View Set

Physiology: Immune System Part 2

View Set

MKTG 300 - Exam 1 Review (Ch 1-3)

View Set

CHAPTER 1 TEST QUESTIONS AND ANSWERS!!!!

View Set